0% found this document useful (0 votes)
958 views164 pages

Vector Calculus

This document provides information about multiple products in vector calculus. It discusses scalar triple products and vector triple products. A scalar triple product is the dot product of one vector with the cross product of two other vectors, resulting in a scalar. A vector triple product is the cross product of a vector with the cross product of two other vectors, resulting in another vector. The document defines scalar triple products and provides a geometric interpretation involving volumes of parallelepipeds. It also discusses properties of scalar triple products, including that their value depends only on the cyclic order of the vectors and is independent of the positions of dot and cross products. The document proves the distributive law for vector products.

Uploaded by

wecharri
Copyright
© © All Rights Reserved
We take content rights seriously. If you suspect this is your content, claim it here.
Available Formats
Download as PDF, TXT or read online on Scribd
0% found this document useful (0 votes)
958 views164 pages

Vector Calculus

This document provides information about multiple products in vector calculus. It discusses scalar triple products and vector triple products. A scalar triple product is the dot product of one vector with the cross product of two other vectors, resulting in a scalar. A vector triple product is the cross product of a vector with the cross product of two other vectors, resulting in another vector. The document defines scalar triple products and provides a geometric interpretation involving volumes of parallelepipeds. It also discusses properties of scalar triple products, including that their value depends only on the cyclic order of the vectors and is independent of the positions of dot and cross products. The document proves the distributive law for vector products.

Uploaded by

wecharri
Copyright
© © All Rights Reserved
We take content rights seriously. If you suspect this is your content, claim it here.
Available Formats
Download as PDF, TXT or read online on Scribd
You are on page 1/ 164

sh na's

Kri TEXT BOOK on

V ector C alculus
(For B.A./B.Sc. (Mathematics) IIIrd/IVth Semester Students of HNB Garhwal University, U.K.)

As per CHOICE BASED CREDIT SYSTEM (w.e.f. 2016-2017)

By

A. R. Vasishtha
Retired Head, Dep’t. of Mathematics
Meerut College, Meerut (U.P.)

Rajesh Dangwal Vipin Kumar Ravi


M.Sc. (Gold Medalist), M. Phil., Ph.D. CSIR-NET (Mathematical Sciences)
Associate Prof. & Head Dehradun (U.K.)
Dep’t. of Mathematics
H.N.B. Garhwal University Campus
Pauri Garhwal (U.K.)

KRISHNA Prakashan Media (P) Ltd.


KRISHNA HOUSE, 11, Shivaji Road, Meerut-250 001 (U.P.), India
Jai Shri Radhey Shyam

Dedicated
to

Lord

Krishna
Authors & Publishers
P reface
This book on VECTOR CALCULUS has been specially written according to
the latest Syllabus to meet the requirements of the B.A. and B.Sc. IVth
Semester Students of all colleges affiliated to Garhwal University (U.K.) .

The subject matter has been discussed in such a simple way that the students
will find no difficulty to understand it. The proofs of various theorems and
examples have been given with minute details. Each chapter of this book
contains complete theory and a fairly large number of solved examples.
Sufficient problems have also been selected from various university examination
papers. At the end of each chapter an exercise containing objective questions has
been given.

We have tried our best to keep the book free from misprints. The authors
shall be grateful to the readers who point out errors and omissions which, inspite
of all care, might have been there.

The authors, in general, hope that the present book will be warmly received
by the students and teachers. We shall indeed be very thankful to our colleagues
for recommending this book to the students.

The authors wish to express their thanks to Mr. S.K. Rastogi, M.D.,
Mr. Sugam Rastogi, Executive Director, Mrs. Kanupriya Rastogi, Director and
entire team of KRISHNA Prakashan Media (P) Ltd., Meerut for bringing
out this book in the present nice form.

The authors will feel amply rewarded if the book serves the purpose for
which it is meant. Suggestions for the improvement of the book are always
welcome.

— Authors
Syllabus
Vector Calculus
H.N.B. Garhwal University, U.K.
Choice Based Credit System (w.e.f. 2016-2017)

B.A./B.Sc.—IVth Semester M.M. : 33 / 65

Multiple products, Reciprocal vector, Vector differentiation and integration,


differential vector operator Del, Gradient, Divergence, Curl, Vector identities. Vector
integration along line, surface and volume, Simple applications of Green’s theorem,
Gauss’s divergence and Stokes theorem (without proof)
B rief C ontents
Dedication.........................................................................(v)
Preface ...........................................................................(vi)
Syllabus ........................................................................(vii)
Brief Contents ...............................................................(viii)

VECTOR CALCULUS...............................................V -01—V -160


1. Multiple Products...............................................................................................V-03—V-28

2. Differentiation of Vectors................................................................................V-29—V-52

3. Gradient, Divergence and Curl.......................................................................V-53—V-96

4. Integration of Vectors......................................................................................V-97—V-104

5. Line Integrals....................................................................................................V-105—V-120

6. Green's, Gauss's and Stoke's Theorems.....................................................V-121—V-160


Krishna's

VECTOR ANALYSIS
C hapters

1. Multiple Products
1.

1. Differentiation of Vectors
2.

1. Gradient, Divergence and Curl


3.

1. Integration of Vectors
4.

1. Line Integrals
5.

1. Green's, Gauss's and Stoke's Theorems


6.
V-3

1
M ultiple P roducts

1 Triple Products
e know that the vector product a × b of two vectors a and b is itself a vector
W quantity. Therefore we can multiply it by another vector c both scalarly and
vectorially. The product (a × b) • c is called scalar triple product, which is a pure
number. On the other hand the product (a × b) × c is called vector triple
product, which is again a vector quantity.
Note. Since a • b is a scalar quantity, therefore the products (a • b) • c and
(a • b) × c are meaningless. Moreover in the product (a × b) • c we can omit the
parentheses and we can simply write it as a × b • c. Obviously the product a × b • c
has meaning only if we regard it as the product (a × b) • c .

2 Scalar Triple Product


The scalar product of two vectors one of which is itself the vector product of two vectors is a
scalar quantity called “Scalar Triple Product”. Thus if a , b and c be three vectors, then
(a × b) • c is called the scalar triple product of these three vectors.
Since the scalar triple product involves both the signs of ‘cross’ and ‘dot’ therefore
it is sometimes also called the mixed product.
V-4

Geometrical Interpretation of Scalar Triple Product.


Let us consider a parallelopiped whose coterminous edges OA , OB , OC have the
lengths and directions of the vectors a, b, c respectively. Let V be the volume of this
parallelopiped. We shall regard V, as necessarily n C
F
positive.
G
φ E
Let a × b = n. Then from our definition of vector
product, the vector n is perpendicular to the face O B
OADB , and its modulus n is the measure of the area of θ
A
the parallelogram OADB . Also, by definition, the D

vectors a, b and n form a right handed triad.



Let φ denote the angle between the directions of the vectors OC and n. Then the
vectors a, b and c will form a right handed or a left handed triad according as φ is
acute or obtuse.
Now (a × b) • c = |(a × b)||c|cos φ = |n||c|cos φ
= (area of the parallelogram OADB).(OC cos φ)
[∵ |c| = OC ]
Now OC cos φ will be positive or negative according as φ is acute or obtuse. Its
absolute value will give us the length of the perpendicular from C to the plane of the
parallelogram OADB .

Now the volume V of the parallelopiped = (Area of the parallelogram OADB) ×


length of the perpendicular from C on this parallelogram. Therefore
(a × b) • c = + V , if φ is acute i. e., if a, b, c form a right handed triad and
(a × b) • c = − V, if φ is obtuse i. e., if a, b, c form a left handed triad.

Now we know that if the vectors a, b, c form a right handed triad, then the vector
triads b, c, a and c, a, b are also right handed. Hence each of the products
(b × c) • a and (c × a) • b will have the same value + V or − V according as a, b, c
form a right handed or a left handed triad. Thus we conclude that in all the cases
(a × b) • c = (b × c) • a = (c × a) • b.
Now a • b = b • a and a × b = − b × a.
∴ (a × b) • c = c • (a × b) = (b × c) • a
= a • (b × c) = (c × a) • b = b • (c × a)
= − (b × a) • c = – c • (b × a) = – (c × b) • a
= – a • (c × b) = – (a × c) • b = – b • (a × c).

From this we conclude that the value of a scalar triple product depends on the cyclic order of
the factors and is independent of the position of the dot and cross. These may be interchanged at
pleasure. However, an anticyclic permutation of the three factors changes the value of the
product in sign but not in magnitude. (Important)
V-5

Notation: In view of the properties discussed above, the scalar triple product is
usually written as (a × b) • c = [abc] or [a, b, c]. This notation takes into consideration
only the cyclic order of the three vectors and disregards the unimportant positions of dot and
cross. Thus [abc] = [bca] = [cab] = – [cba] etc.
The signs of dot and cross can be inserted at pleasure i. e.,
[a, b, c] = a • (b × c) or = (a × b) • c .
Note 1: If i , j , k constitute an orthogonal right handed triad of unit vectors,
then [i, j, k] = (i × j) • k = k • k = 1.
Note 2: The scalar triple product [abc] is positive or negative according as a, b, c
form a right handed or a left handed triad of vectors.

3 Distributive Law for Vector Product


To prove that a × (b + c) = a × b + a × c , where a , b , c are any three vectors.
Let r ≡ a × (b + c) – a × b – a × c …(1)
Now forming the scalar product of both sides of (1) with an arbitrary vector d, we
get
d • r = d • [a × (b + c) – a × b – a × c] …(2)
or d • r = d • [a × (b + c)] – d • (a × b) – d • (a × c)
[Since scalar product is distributive]
Now in a scalar triple product the positions of dot and cross can be interchanged
without affecting its value. Therefore from (2), we get
d • r = (d × a) • (b + c) – (d × a) • b – (d × a) • c
= (d × a) • b + (d × a) • c – (d × a) • b – (d × a) • c
[Since scalar product is distributive]
= 0.
Therefore either d = 0, or r = 0 or d is perpendicular to r. But the vector d is
arbitrary. Therefore we can take it to be non-zero and not perpendicular to r.
Hence r = 0 i. e., a × (b + c) – a × b – a × c = 0
i. e., a × (b + c) = a × b + a × c.

4 Properties of Scalar Triple Product


(i) The value of a scalar triple product, if two of its vectors are equal, is zero.
We have [aab] = a • (a × b).
Now a × b is a vector perpendicular to the plane of a and b.
Therefore a • (a × b) = 0.
(ii) The value of a scalar triple product, if two of its vectors are parallel, is zero.
Let a, b, c be three vectors such that a and b are parallel i. e., b = ta, where t is
some scalar.
V-6

Now [abc] = (a × b) • c = (a × ta) • c = t (a × a) • c = t (0 • c)


[∵ a × a = 0]
= 0.
(iii) The necessary and sufficient condition that three non-parallel and non-zero vectors
a, b, c be coplanar is that [abc] = 0.
Let a, b, c be three coplanar vectors. Now a × b is a vector perpendicular to
the plane of a and b. Since a, b, c are coplanar, therefore a × b is also
perpendicular to c. Now the dot product of two perpendicular vectors is
equal to zero. Hence (a × b) • c = 0 i. e., [abc] = 0. Therefore the condition
is necessary.
The condition is also sufficient: Because if [abc] = 0 i. e., (a × b) • c = 0,
then c is perpendicular to a × b. But a × b is a vector perpendicular to the
plane of a and b. Since c is perpendicular to a × b , therefore c is parallel to
the plane of a and b.
Hence a , b , c are coplanar.
(iv) Since the distributive law holds for both scalar and vector products, it holds also for
the scalar triple product.
Thus [a , b + d , c + r] = [abc] + [abr] + [adc] + [adr], the cyclic order of the
factors being maintained in each term.

5 To Express The Value of The Scalar Triple Product [abc]


In Terms of Rectangular Components of The Vectors
Let a = a1 i + a2 j + a3 k , b = b1 i + b2 j + b3 k ,
c = c1 i + c 2 j + c 3 k .
Now b × c = (b1 i + b2 j + b3 k) × (c1 i + c 2 j + c 3 k)
i j k
= b1 b2 b3
 
c1 c2 c 3

= (b2 c 3 − b3 c 2 ) i − (b1 c 3 − b3 c1 ) j + (b1 c 2 − b2 c1 ) k .


∴ a • (b × c) = (a1 i + a2 j + a3 k) • [(b2 c 3 − b3 c 2 ) i
− (b1 c 3 − b3 c1 ) j + (b1 c 2 − b2 c1 ) k]
= a1 (b2 c 3 − b3 c 2 ) − a2 (b1 c 3 − b3 c1 ) + a3 (b1 c 2 − b2 c1 )
[∵ i • i = j • j = k • k = 1 and i • j = j • k = k • i = 0]
a1 a2 a3
∴ [abc] = b1 b2 b3 …(1)
 
c1 c2 c 3
V-7

 c1 c2 c 3  a1 a2 a3
Also (a × b) • c = c • (a × b) = a1 a2 a3 =  b1 b2 b3
   
 b1 b2 b3  c1 c2 c 3

showing that the value of a scalar triple product is independent of the positions of
dot and cross.

Note: If OA , OB , OC be three concurrent edges of a parallelopiped and if


(a1 , a2 , a3 ), (b1 , b2 , b3 ), (c1 , c 2 , c 3 ) be the rectangular coordinates of A , B , C
referred to O as origin, then the determinant (1) gives the volume of that
parallelopiped.

6 To Express the Scalar Triple Product [a, b, c] in Terms


of any Three non-Coplanar Vectors l, m, n
Let a = a1 l + a2 m + a3 n , b = b1 l + b2 m + b3 n ,
and c = c1 l + c 2 m + c 3 n .
Now b × c = (b1 l + b2 m + b3 n) × (c1 l + c 2 m + c 3 n)
= b1 c1 l × l + b1 c 2 l × m + b1 c 3 l × n + b2 c1 m × l
+ b2 c 2 m × m + b2 c 3 m × n + b3 c1 n × l
+ b3 c 2 n × m + b3 c 3 n × n
= (b2 c 3 − b3 c 2 ) m × n − (b1 c 3 − b3 c1 ) n × l
+ (b1 c 2 − b2 c1 ) l × m
[∵ l × l = 0 and l × m = – m × l etc.]
∴ a • (b × c) = (a1 l + a2 m + a3 n) • [(b2 c 3 − b3 c 2 ) m × n
− (b1 c 3 − b3 c1 ) n × l + (b1 c 2 − b2 c1 ) l × m]
= a1 (b2 c 3 − b3 c 2 ) [lmn] − a2 (b1 c 3 − b3 c1 ) [lmn]
+ a3 (b1 c 2 − b2 c1 ) [lmn] .
[∵ [lmn] = [mnl] = [nlm ] and all the scalar
triple products of the type [lml] in
which two vectors are equal vanish]
a1 a2 a3
Hence, [abc] = b1 b2 b3 [lmn ].
 
c1 c2 c 3

Note: Since [i , j, k] = 1, therefore article 5 is particular case of article 6.


V-8

Example 1: Find the volume of the parallelopiped whose edges are represented by
a = 2 i − 4 j+ 5 k, b = i − j+ k, c = 3 i − 5 j + 2 k.
Solution: The required volume of the parallelopiped is equal to the absolute
value of [a b c].
2 −4 5
Now 
[a b c] = 1 −1 1
 
3 −5 2
= 2 (−2 + 5) + 4 (2 − 3) + 5 (−5 + 3)= 6 − 4 − 10 = − 8.
Neglecting the negative sign, we get the volume of the parallelopiped = 8 cubic
units.

Example 2: Find the constant p such that the vectors a = 2 i − j + k , b = i + 2 j − 3 k ,


c = 3 i + p j + 5 k are coplanar. (Kumaun 2013)
Solution: If the vectors a, b, c are coplanar, then we should have [abc] = 0.
2 −1 1
Now [a b c] =  1 2 −3
 
3 p 5
= 2 (10 + 3 p) + 1 (5 + 9) + 1 ( p − 6) = 7 p + 28.
∴ [abc] will be zero if 7 p + 28 = 0 or p = − 4.
Hence for the given vectors to be coplanar, we should have p = − 4.

Example 3: Prove that the four points 4 i + 5 j + k, – ( j + k), (3 i + 9 j + 4 k) and


4 ( – i + j + k) are coplanar. (Kumaun 2012)
Solution: Let A , B , C , D be the four given points whose position vectors
referred to some origin O are
4 i + 5 j + k , − ( j + k), (3 i + 9 j + 4 k) and 4 (− i + j + k).
→ → →
If the four points A, B, C, D are coplanar, then the vectors AB, AC and AD
should also be coplanar.

We have AB = position vector of B − position vector of A
= − ( j + k) − (4 i + 5 j + k) = − 4 i − 6 j − 2 k = a (say).

Similarly AC = (3 i + 9 j + 4 k) − (4 i + 5 j + k) = − i + 4 j + 3 k = b (say),

and AD = 4 (− i + j + k) − (4 i + 5 j + k) = − 8 i − j + 3 k = c (say).
Now the vectors a , b , c will be coplanar if [a b c] = 0.
V-9

 −4 −6 −2
Now [a b c] =  −1 4 3
 
 −8 −1 3
= − 4 (12 + 3) + 6 (− 3 + 24) − 2 (1 + 32)
= − 60 + 126 − 66 = 0.
∴ The points A, B, C, D are coplanar.

Example 4: Show that the four points − a + 4b − 3c, 3a + 2b − 5c, − 3a + 8b − 5c


and − 3a + 2b + c are coplanar.
Solution: Let A, B, C and D be the points whose position vectors are respectively
− a + 4b − 3c , 3a + 2b − 5c , − 3a + 8b − 5c and −3a + 2b + c .

We have AB = position vector of B − position vector of A
= 3a + 2b − 5c − (− a + 4b − 3c) = 4a − 2b − 2c ,

AC = position vector of C − position vector of A
= (− 3a + 8b − 5c) − (− a + 4b − 3c) = − 2a + 4b − 2c ,

and AD = (− 3a + 2b + c) − (− a + 4b − 3c) = − 2a − 2b + 4c .
→ → →
Now the scalar triple product of the vectors AB, AC and AD
 4 −2 −2
→ → →
= [ AB, AC , AD] =  −2 4 −2 [a bc] [Refer article 6]
 
 −2 −2 4
= {4 (16 − 4) + 2 (− 8 − 4) − 2 (4 + 8)} [a b c]
= (48 − 24 − 24) [a b c] = 0 [a b c] = 0.
→ → →
Since the scalar triple product of the vectors AB, AC and AD is zero, therefore
these vectors are coplanar. Hence the points A, B, C and D are coplanar.

Example 5: If a, b, c are the position vectors of A, B, C prove that a × b + b × c + c × a


is a vector perpendicular to the plane of ABC .
→ → →
Solution: We have AB = b – a , BC = c – b and CA = a – c .
Let d= a × b+b × c +c × a.

Now d • AB = d • (b – a) = (a × b + b × c + c × a) • (b – a)
= (a × b) • b − (a × b) • a + (b × c) • b – (b × c) • a
+ (c × a) • b – (c × a) • a
= [abb] – [aba] + [bcb] – [bca] + [cab] – [caa]
= – [bca] + [cab], since [abb] = 0 etc.
V-10

= – [bca] + [bca], since [cab] = [bca]


= 0.

Therefore vector d is perpendicular to AB. Similarly, we can show that d is

perpendicular to BC.
Now since d is perpendicular to two lines in the plane ABC , hence it is
perpendicular to the plane ABC .

Example 6: Prove that [a + b , b + c , c + a] = 2 [abc]. (Kumaun 2010)


Solution: L.H.S.
= (a + b) • [(b + c) × (c + a)]
= (a + b) • [b × c + b × a + c × c + c × a]
= (a + b) • [b × c + c × a + b × a] , since c × c = 0
= a • (b × c) + a • (c × a) + a • (b × a) + b • (b × c)
+ b • (c × a) + b • (b × a)
= [abc] + [aca] + [aba] + [bbc] + [bca] + [bba] = [abc] + [bca],
since all the scalar triple products in which two vectors are equal vanish.
But [abc] = [bca].
Hence the L.H.S. = 2 [abc].

l • a l•b l • c
Example 7: Prove that [lmn] [abc]=m • a m• b m • c⋅
 
n • a n• b n • c
Solution: Let l = l1 i + l2 j + l3 k , m = m1 i + m2 j + m3 k ,
n = n1 i + n2 j + n3 k ; a = a1 i + a2 j + a3 k ,
b = b1 i + b2 j + b3 k , c = c1 i + c 2 j + c 3 k .

Now L.H.S. = [lmn] [abc]

 l1 l2 l3  a1 a2 a3
=  m1 m2 m3 b1 b2 b3
  
 n1 n2 n3  c1 c2 c 3
l1 a1 + l2 a2 + l3 a3 l1 b1 + l2 b2 + l3 b3
= m1 a1 + m2 a2 + m3 a3 m1 b1 + m2 b2 + m3 b3
n1 a1 + n2 a2 + n3 a3 n1 b1 + n2 b2 + n3 b3
l1 c1 + l2 c 2 + l3 c 3
m1 c1 + m2 c 2 + m3 c 3
n1 c1 + n2 c 2 + n3 c 3
by the rule for the multiplication of determinants of the same order.
V-11

Now l • a = (l1 i + l2 j + l3 k) • (a1 i + a2 j + a3 k)


= l1 a1 + l2 a2 + l3 a3 , etc.

 l •a l•b l•c 
Hence the L.H.S. = m • a m• b m • c ⋅
 
 n•a n• b n•c 

Example 8: Prove that if l , m , n be three non-coplanar vectors, then


 l•a l•b l 
[lmn] (a × b) = m • a m• b m ⋅
 
 n•a n• b n
Solution: Let
l = l1 i + l2 j + l3 k , m = m1 i + m2 j + m3 k , n = n1 i + n2 j + n3 k ,
and a = a1 i + a2 j + a3 k , b = b1 i + b2 j + b3 k .
 l1 l2 l3  i j k
Now [lmn ] =  m1 m2 m3 and ( a × b) =  a1 a2 a3⋅
   
 n1 n2 n3   b1 b2 b3
 l1 l2 l3  i j k
∴ [lmn ] ( a × b) =  m1 m2 m3 a1 a2 a3
  
 n1 n2 n3  b1 b2 b3
 l1 i + l2 j + l3 k l1 a1 + l2 a2 + l3 a3 l1 b1 + l2 b2 + l3 b3 
= m1 i + m2 j + m3 k m1 a1 + m2 a2 + m3 a3 m1 b1 + m2 b2 + m3 b3.
 
 n1 i + n2 j + n3 k n1 a1 + n2 a2 + n3 a3 n1 b1 + n2 b2 + n3 b3 
Now l • a = (l1 i + l2 j + l3 k) • (a1 i + a2 j + a3 k)
= l1 a1 + l2 a2 + l3 a3 etc.
l l•a l • b  l • a l•b l
∴ 
[lmn] (a × b) = m m • a m • b=m • a m• b m⋅
   
n n•a n • b n • a n• b n

Comprehensive Exercise 1

1. Define scalar triple product of three vectors a , b , c and interpret the same
geometrically.

2. Define scalar triple product of u , v , w . Prove that the value of the scalar triple
product of u , v , w remains unchanged if the cyclic order of the vectors is
maintained.
V-12

3. If a, b, c are any three vectors, prove that


a • (b × c) = b • (c × a) = c • (a × b) .
4. Prove that a • b × c = a × b • c .
5. Show that i • j × k = 1.
6. Show that [λa + µb , c , d] = λ [a , c , d] + µ [b, c, d].
7. Prove that [i – j, j – k , k – i] = 0.
8. Find the volume of the parallelopiped whose edges are represented by
(i) a = 2i − 3 j + 4k , b = i + 2 j – k , c = 3i – j + 2k .
(ii) a = i − 2 j + 3k , b = 2i + j − k , c = j + k .
9. Show that the vectors i − 2 j + 3k , − 2i + 3 j − 4k , i − 3 j + 5k are coplanar.
10. Show that the vectors 2a − b + 3c , a + b − 2c and a + b − 3c are
non-coplanar where a , b , c are non-coplanar vectors.
[Hint: Show that the scalar triple product of the three given vectors is not
zero.]
11. Prove that the four points 6a − 4b + 10 c , − 5a + 3b − 10 c , 4a − 6b − 10 c
and 2b + 10 c are coplanar.
12. Show that the vectors a , b , c are coplanar if b + c , c + a , a + b are coplanar.
13. Prove that four points with position vectors a , b , c , d are coplanar if and only
if [b , c , d] + [c , a , d] + [a , b , d] = [a , b , c].

A nswers 1
8. (i) 7 cubic units, (ii) 12 cubic units.

7 Vector Triple Product


The vector product of two vectors one of which is itself the vector product of two vectors is a vector
quantity called a “Vector Triple Product”. Thus if a, b and c be three vectors,the
products of the form a × (b × c) and (a × b) × c etc. are called “Vector Triple
Products”.
Theorem: To prove that a × (b × c) = (a • c) b – (a • b) c .

Let r = a × (b × c) and b × c = d .
Since b × c = d , therefore d is a vector perpendicular to the plane containing b and
c . Also r = a × d . Therefore r is a vector perpendicular to both a and d. Now the
vector r is perpendicular to the vector d, whereas the vector d is perpendicular to
the plane containing b and c .Therefore the vector r must lie in the plane containing
V-13

b and c . Hence the vector r can be expressed linearly in terms of b and c in the form
r = l b + mc , …(1)
where l and m are scalars.
Since r is perpendicular to a , therefore r • a = 0.
∴ (l b + mc) • a = 0 or l (b • a) + m (c • a) = 0.
l −m
∴ = = λ (say).
c•a b•a
Putting the values of l and m in (1), we get
r = λ (c • a) b − λ (b • a) c = λ [(c • a) b – (b • a) c] …(2)
Now we are to find the value of λ.
Consider unit vectors j and k,the first parallel to b and the second perpendicular to
it in the plane containing b and c . Then we may write b = b2 j and
c = c2 j + c3 k .
In terms of j and k and the other unit vector i of the right handed system, the
remaining vector a may be written as
a = a1 i + a2 j + a3 k .
Now b × c = b2 j × (c 2 j + c 3 k) = b2 c 2 j × j + b2 c 3 j × k = b2 c 3 i
[∵ j × j = 0 and j × k = i]
∴ r = a × (b × c) = (a1 i + a2 j + a3 k) × (b2 c 3 i)
= a1 b2 c 3 i × i + a2 b2 c 3 j × i + a3 b2 c 3 k × i
= a3 b2 c 3 j − a2 b2 c 3 k …(3)
[∵ i × i = 0, j × i = – k and k × i = j ]
Also r = λ [(c • a) b – (b • a) c]
= λ [(c 2 j + c 3 k) • (a1 i + a2 j + a3 k) b2 j
− (b2 j) • (a1 i + a2 j + a3 k) (c 2 j + c 3 k)]
= λ [c 2 a2 b2 j + c 3 a3 b2 j − b2 a2 c 2 j − b2 a2 c 3 k]
[∵ i • i = 1, i • j = 0 etc.]
= λ [a3 b2 c 3 j − a2 b2 c 3 k]. …(4)
Now from (3) and (4) we conclude that λ = 1.
Hence a × (b × c) = (c • a) b – (b • a) c = (a • c) b – (a • b) c
[∵ c • a = a • c]
Corollary: (a × b) × c = – [c × (a × b)]
= – [(c • b) a – (c • a) b] = (c • a) b – (c • b) a .
Rule to remember a × (b × c) . It is a vector to be expressed linearly in terms of b
and c which are the vectors within the brackets. Also
a × (b × c) = [Dot product of a and c] b
− [Dot product of a and b] c .
Similarly we may remember (a × b) × c .
V-14

8 Vector Triple Product is not Associative


If a , b , c be three vectors, then a × (b × c) gives a vector which lies in the plane of b
and c and which is perpendicular to a . Moreover (a × b) × c gives a vector which
lies in the plane of a and b and which is perpendicular to c . Hence, in general,
a × (b × c) ≠ (a × b) × c . Thus in the case of vector triple product the position of
brackets cannot be, in general, changed without altering the value of the product.

Example 9: Prove that a × (b × c) + b × (c × a) + c × (a × b) = 0 .


Solution: We have
a × (b × c) = (a • c) b – (a • b) c , b × (c × a) = (b • a) c – (b • c) a ,
and c × (a × b) = (c • b) a – (c • a) b .
Adding these three expressions, we get
a × (b × c) + b × (c × a) + c × (a × b)
= (a • c) b – (a • b) c + (b • a) c – (b • c) a + (c • b) a – (c • a) b
= 0. [∵ a • c = c • a , a • b = b • a , b • c = c • b]

Example 10: Show that the vectors a × (b × c) , b × (c × a) , c × (a × b) are coplanar.


Solution: Let r1 = a × (b × c) , r 2 = b × (c × a) , r 3 = c × (a × b) .
Now first prove that r1 + r 2 + r 3 = 0, as we have done in the previous exercise.
Since there exists a linear relation between the vectors r1 , r 2 , r 3 therefore any of
these vectors can be expressed as a linear combination of the other two. Hence
these three vectors are coplanar.

Example 11: If a = i − 2 j + k , b = 2i + j + k , c = i + 2 j – k , find a × (b × c) .


Solution: We have
a × (b × c) = (a • c) b – (a • b) c
= [(i − 2 j + k) • (i + 2 j – k)] (2i + j + k)
− [(i − 2 j + k) • (2i + j + k)] (i + 2 j – k)
= (1 − 4 − 1) (2i + j + k) − (2 − 2 + 1) (i + 2 j – k)
= (− 8i − 4 j − 4k) − (i + 2 j − k) = − 9i − 6 j − 3k .

Example 12: Show that i × (a × i) + j × (a × j) + k × (a × k) = 2a .


(Kumaun 2009)
Solution: We have
i × (a × i) = (i • i) a – (i • a) i = a – (i • a) i [∵ i • i = 1]
j × (a × j) = ( j • j) a – ( j • a) j = a – ( j • a) j [∵ j • j = 1]
V-15

and k × (a × k) = (k • k) a – (k • a) k = a – (k • a) k . [∵ k • k = 1]
Adding these three expressions, we get
i × (a × i) + j × (a × j) + k × (a × k)
= 3a − (i • a) i – ( j • a) j – (k • a) k
= 3a – [(a • i) i + (a • j) j + (a • k) k]. [∵ a • j = j • a etc.]
Now we shall show that
a = (a • i) i + (a • j) j + (a • k) k .
Let a = xi + yj + zk .
Taking dot product of both sides with i , j and k successively, we get
x = a • i , y = a • j, z = a • k .
∴ a = (a • i) i + (a • j) j + (a • k) k .
Hence i × (a × i) + j × (a × j) + k × (a × k) = 3a – a = 2a .

Example 13: Show that i × ( j × k) = 0.


Solution: We have i × ( j × k) = i × i [∵ j × k = i ]
= 0. [∵ i × i = 0 ]
2
Example 14: Show that [a × b , b × c , c × a] = [abc] , and express the result by
means of determinants. (Kumaun 2011)
Solution: We have
[a × b , b × c , c × a] = (a × b) • [(b × c) × (c × a)].
Let us first find the value of (b × c) × (c × a).
Let b × c = d.
Then (b × c) × (c × a) = d × (c × a) = (d • a) c – (d • c) a
= [(b × c) • a] • c − [(b × c) • c] a
= [bca] c – [bcc] a = [abc] c ,
since [bcc] = 0 and [bca] = [abc].
∴ [a × b , b × c , c × a] = (a × b) • [abc] c
= [abc] (a × b) • c = [abc] [abc] = [abc] 2 .
Second part: Let
a = a1 i + a2 j + a3 k , b = b1 i + b2 j + b3 k ,
c = c1 i + c 2 j + c 3 k.
 a1 a2 a3
We have 
[abc ] = b1 b2 b3⋅
 
 c1 c2 c 3
i j k
Again a × b =  a1 a2 a3
 
 b1 b2 b3
V-16

= (a2 b3 − b2 a3 ) i + (b1 a3 − a1 b3 ) j + (a1 b2 − a2 b1 ) k .


i j k
Similarly b × c =  b1 b2 b3
 
 c1 c2 c 3

= (b2 c 3 − b3 c 2 ) i + (c1 b3 − b1 c 3 ) j + (b1 c 2 − c1 b2 ) k


i j k
and c × a =  c1 c2 c 3
 
 a1 a2 a3

= (c 2 a3 − a2 c 3 ) i + (a1 c 3 − a3 c1 ) j + (c1 a2 − a1 c 2 ) k .
∴ [a × b, b × c, c × a]
 a2 b3 − b2 a3 b1 a3 − a1 b3 a1 b2 − a2 b1
=  b2 c 3 − b3 c 2 c1 b3 − b1 c 3 b1 c 2 − b2 c1
 
 c 2 a3 − c 3 a2 a1 c 3 − a3 c1 c1 a2 − c 2 a1

C1 C2 C3 A1 A2 A3


= A1 A2 A3 = B1 B2 B3,
   
B1 B2 B3 C1 C2 C3
where the capital letters A1 , A2 , A3 etc. denote the cofactors of the corresponding
a1 a2 a3

small letters a1 , a2 , a3 etc. in the determinant b1 b2 b3⋅
 
c1 c2 c 3

Since [abc] 2 = [a × b , b × c , c × a],


2
a1 a2 a3 A1 A2 A3
∴ b b2 b3 = B1 B2 B3⋅
1   
c1 c2 c 3 C1 C2 C3

Example 15: Prove that (a × b) × (a × c) • d = (a • d) [abc] .

Solution: Let a × b = r .
Then (a × b) × (a × c) = r × (a × c) = (r • c) a – (r • a) c
= [(a × b) • c] a – [(a × b) • a] c
= [abc] a – [aba] c = [abc] a , since [aba] = 0.
Therefore (a × b) × (a × c) • d = [abc] a • d = (a • d) [abc] .

1
Example 16: If a , b , c be three unit vectors such that a × (b × c) = b , find the angles
2
which a makes with b and c , b and c being non-parallel. (Kumaun 2009)
1
Solution: It is given that a × (b × c) = b .
2
V-17

1
∴ (a • c) b – (a • b) c = b,
2
 a c – 1 b – (a b) c = 0.
or  •  • …(1)
 2
Since b and c are non-parallel, therefore for the existence of the relation (1) the
coefficients of b and c should vanish separately. Therefore, we get
1 1
a • c − = 0, i. e., a • c = and a • b = 0.
2 2
Let θ and φ be the angles which a makes with b and c respectively. Since a , b , c are
unit vectors, we have
a • b = cos θ = 0 ⇒ θ = 90 ° ,
1
and a • c = cos φ = ⇒ φ = 60 ° .
2

Example 17: Prove that (a × b) × c = a × (b × c) , if and only if (c × a) × b = 0 .


Solution: We have (a × b) × c = a × (b × c)
if and only if (c • a) b – (c • b) a = (a • c) b – (a • b) c
i. e., if and only if – (c • b) a = – (a • b) c, since c • a = a • c
i. e., if and only if (c • b) a – (a • b) c = 0
i. e., if and only if (b • c) a – (b • a) c = 0, since c • b = b • c anda • b = b • a
i. e., if and only if (c × a) × b = 0.
Note: (c × a) × b = 0 is possible when (i) a and c are collinear because then
c × a = 0 or (ii) b is parallel to c × a i. e., b is perpendicular to both c and a or (iii) at
least one of the vectors a, b, c is a null vector.

Comprehensive Exercise 2

1. Evaluate (b × c) × a,
where a = 2i + 3 j − 5k , b = – i + j + k , and c = 4i + 2 j + 6k.
2. (i) Verify the formula for vector triple product
a × (b × c) = (a • c) b – (a • b) c
by taking a = i + j, b = – i + 2k , c = j + k.
(ii) Verify a × (b × c) = (a • c) b – ( a • b) c for a = i – 2j+ k , b = 2i + j – k
and c = 3i – 2j + 3k . (Kumaun 2014)
3. Prove that (b × c) × (c × a) = [abc] c .
4. Prove that for any three vectors A , B and C,
(A × B) • (B × C) × (C × A) = (A • B × C) 2 .
V-18

5. Prove that for any three vectors a , b and c , [a × b , b × c , c × a] = [abc] 2 .


Hence show that the vectors a , b , c are non-coplanar if and only if the
vectors a × b , b × c , c × a are non-coplanar.
a • a a•b a • c
6. 
Show that [a × b, b × c, c × a]= b • a b• b b • c  = [abc ]2 .
 
c • a c•b c • c
[Hint: First prove that [a × b , b × c , c × a] = [abc]2 .
For proof see Ex. 6. For the next part proceed as in Ex. 7.]
7. Prove that a × (b × a) = (a × b) × a .

A nswers 2
1. 8 (−4i + j – k).

9 Scalar Product of Four Vectors


If a , b , c , d are four vectors, the products (a × b) • (c × d) , (a × d) • (b × c) etc. are
called scalar products of four vectors.
Theorem: To prove that
a • c b • c
(a × b) • (c × d) = ⋅
a • d b • d
Let a × b = r.
Then (a × b) • (c × d) = r • (c × d) .
Now in a scalar triple product the position of dot and cross may be interchanged
without altering the value of the product.
Therefore r • (c × d) = (r × c) • d.
∴ (a × b) • (c × d) = [(a × b) × c] • d = [(c • a) b – (c • b) a] • d
= (c • a) (b • d) – (c • b) (a • d)
a • c b • c
= (a • c) (b • d) – (b • c) (a • d) = ⋅
a • d b • d
This relation is known as Lagrange’s Identity.

10 Vector Product of Four Vectors


Let a , b , c , d be four vectors. Consider the vector product of the vectors a × b and
c × d . This product can be written as (a × b) × (c × d) and is called the vector
V-19

product of four vectors. It is a vector perpendicular to a × b and, therefore coplanar


with a and b. Similarly it is a vector coplanar with c and d. Hence this vector must
be parallel to the line of intersection of a plane parallel to a and b with another
plane parallel to c and d.
Theorem: To prove that
(i) (a × b) × (c × d) = [abd] c – [abc] d
(ii) (a × b) × (c × d) = [acd] b – [bcd] a .
Proof:
(i) (a × b) × (c × d) is a vector which can be either expressed in terms of c and d or
in terms of a and b.To express it in terms of c and d, let us put a × b = l . Then
(a × b) × (c × d) = l × (c × d) = (l • d) c – (l • c) d
= [(a × b) • d] c – [(a × b) • c] d
= [abd] c – [abc] d .
(ii) Again to express (a × b) × (c × d) in terms of a and b, let us put c × d = m .
Then (a × b) × (c × d) = (a × b) × m = – m × (a × b)
= – [(m • b) a – (m • a) b] = (m • a) b – (m • b) a
= [(c × d) • a] b – [(c × d) • b] a
= [cda] b – [cdb] a = [acd] b – [bcd] a .

Linear Relation connecting four vectors: Equating the above two expressions
for the value of (a × b) × (c × d) , we get
[abd] c – [abc] d = [acd] b – [bcd] a
or [bcd] a – [acd] b + [abd] c – [abc] d = 0 , …(1)
which is the required linear relation connecting the four vectors a , b , c , d .
To find an expression for any vector r, in space, as a linear combination of
three non-coplanar vectors a , b , c .
Replacing d by r in the relation (1) just established, we get
[bcr] a – [acr] b + [abr] c – [abc] r = 0
or [abc] r = [bcr] a – [acr] b + [abr] c . …(2)
Since a , b , c are non-coplanar, therefore [abc] ≠ 0.
Therefore dividing both sides of (2) by [abc], we get
[bcr] a – [acr] b + [abr] c ,
r=
[abc]
[bcr] a + [car] b + [abr] c
or r= , since [acr] = – [car]
[abc]
[rbc] a + [rca] b + [rab] c
or r= , …(3)
[abc]
which is the required expression for r .
V-20

11 Reciprocal System of Vectors


If a , b , c be any three non-coplanar vectors so that [abc] ≠ 0, then the three vectors a ′ , b ′ , c ′
defined by the equations
b×c c×a a ×b
a′ = , b′ = , c′ =
[abc] [abc] [abc]

are called reciprocal system of vectors to the vectors a , b , c .


(i) To show that a • a ′ = b • b ′ = c • c ′ = 1.
b×c a • (b × c) [abc]
We have a • a′ = a • = = = 1.
[abc] [abc] [abc]
c×a b • (c × a) [bca] [abc]
Similarly b • b′ = b • = = = =1
[abc] [abc] [abc] [abc]
a ×b c • (a × b) [cab] [abc]
and c • c′ = c • = = = = 1.
[abc] [abc] [abc] [abc]
Note: The reason for the name reciprocal lies in the relations
a • a ′ = b • b ′ = c • c ′ = 1.
(ii) The scalar product of any other pair of vectors, one from each system, is zero i. e.,
a • b ′ = a • c ′ = b • a ′ = b • c ′ = c • a ′ = c • b ′ = 0.
We have
c×a a • (c × a) [aca]
a • b′ = a • = = = 0, since [aca] = 0.
[abc] [abc] [abc]
Similarly we can prove the other results.
(iii) The scalar triple product [abc] formed from three non-coplanar vectors a , b , c is the
reciprocal of the scalar triple product [a ′ b ′ c ′ ] formed from the reciprocal system
a ′ , b ′ , c ′ i.e., [abc] [a ′ b ′ c ′ ] = 1.
b × c  c × a (a × b) 
We have [a ′ b′ c ′ ]= a ′ • (b′ × c ′ ) = • × 
[abc]  [abc] [abc] 
(b × c) • [(c × a) × (a × b)]
= ⋅
[abc] 3
Now expanding (c × a) × (a × b) by vector triple product treating c × a as one
vector, we get
(c × a) × (a × b) = [(c × a) • b] a – [(c × a) • a] b = [cab] a – [caa] b
= [abc] a , since [caa] = 0
and [cab] = [abc].
(b × c) • [abc] a [(b × c) • a] [abc]
∴ [a ′ b ′ c ′ ] = =
[abc] 3 [abc] 3
V-21

[bca] [abc] [abc] 2 1


= 3
= 3
= ⋅
[abc] [abc] [abc]
∴ [a ′ b ′ c ′ ] [abc] = 1.
Note 1: Since [abc] ≠ 0, therefore from the relation, [a ′ b ′ c ′ ] [abc] = 1, we
conclude that [a ′ b ′ c ′ ] ≠ 0.
Hence the vectors a ′ , b ′ , c ′ are also non-coplanar.
Note 2: The symmetry of results proved in properties (i), (ii) and (iii) suggest that
if a ′ , b ′ , c ′ is the reciprocal system to a , b , c then a , b , c is also the reciprocal
system to a ′ , b ′ , c ′ .
Note 3: The relation [abc] [a ′ b ′ c ′ ] = 1shows that the scalar triple products [abc]
and [a ′ b ′ c ′ ] are either both positive or both negative. Hence the two systems of
vectors a , b , c and a ′ , b ′ , c ′ are either both right handed or both left handed.
(iv) The orthonormal vector triads i , j, k form a self reciprocal system.
Let i ′ , j′ , k ′ be the system of vectors reciprocal to the system i , j, k .
j× k i
Then by definition i ′ = = = i.
[i j k] 1
Similarly, j′ = j and k ′ = k .
Hence the result.

Theorem: If a , b , c be three non-coplanar vectors and a ′ , b ′ , c ′ constitute the reciprocal


system of vectors, then any vector r can be expressed as
r = (r • a ′ ) a + (r • b ′ ) b + (r • c ′ ) c .
Proof: Let r be expressed as a linear combination of the non-coplanar vectors
a , b , c in the form
r = xa + yb + zc …(1)
where x, y, z are some scalars.
Multiplying both sides of (1) scalarly with b × c , we get
r • (b × c) = xa • (b × c) + yb • (b × c) + zc • (b × c)
= x [abc] + y [bbc] + z [cbc] = x [abc],
since [bbc] = 0 = [cbc] .
r • (b × c) (b × c) b×c
∴ x= = r• = r • a ′ , since a ′ = ⋅
[abc] [abc] [abc]
Similarly multiplying both sides of (1) scalarly with c × a and a × b , we can show
that
y = r • b ′ and z = r • c ′ .
Putting the values of x, y and z in (1), we get
r = (r • a ′ ) a + (r • b ′ ) b + (r • c ′ ) c . …(2)
V-22

Note 1: In a similar manner, we can prove that


r = (r • a) a ′ + (r • b) b ′ + (r • c) c ′ .
Note 2: Since the system of vectors i , j , k is self-reciprocal, therefore from (2) we
conclude that r = (r • i) i + (r • j) j + (r • k) k .

Example 16: Find a set of vectors reciprocal to the set 2i + 3 j – k , i – j − 2k ,


– i + 2 j + 2k .
Solution: Let a = 2i + 3 j – k , b = i – j − 2k , c = – i + 2 j + 2k .
Let a ′ , b ′ , c ′ be the set of vectors reciprocal to the set a , b , c .Then by definition
b×c c×a a ×b
a′ = , b′ = , c′ = ⋅
[abc] [abc] [abc]
 2 3 −1
Now [abc] =  1 −1 −2 = 2 (2) − 3 (0) − 1 (1) = 3
 
 −1 2 2

 i j k
and b × c = 1 −1 −2 = 2i + 0 j + k = 2 i + k.
 
 −1 2 2
b × c 2i + k  2
k ⋅
1
∴ a′ = = = i+
[abc] 3 3 3 

 i j k
c×a 1 − 8i + 3j − 7k
Similarly b′ = = −1 2 2 =
[abc] 3  3
 2 3 −1

i j k
a×b 1 − 7i + 3j − 5k
and c′ = = 2 3 −1 = ⋅
[abc] 3  3
1 −1 −2

Example 17: Prove that a × {b × (c × d)} = (b • d) (a × c) − (b • c) (a × d) .


Hence expand a × [b × {c × (d × e)}].
Solution: First part: We have
a × {b × (c × d)} = a × {(b • d) c – (b • c) d}
= (b • d) (a × c) – (b • c) (a × d) .
Second part: We have
b × {c × (d × e)} = b × {(c • e) d – (c • d) e}
= (c • e) (b × d) – (c • d) (b × e) .
V-23

∴ a × [b × {c × (d × e)}] = a × [(c • e) (b × d) – (c • d) (b × e)]


= (c • e) [a × (b × d)] – (c • d) [a × (b × e)]
= (c • e) [(a • d) b – (a • b) d]
– (c • d) [(a • e) b – (a • b) e] .

Example 18: Prove that d • [a × {b × (c × d)}] = (b • d) [acd] .


Solution: a × {b × (c × d)} = a × {(b • d) c – (b • c) d}
= (b • d) (a × c) – (b • c) (a × d) .
∴ d • [a × {b × (c × d)}] = d • [(b • d) (a × c) – (b • c) (a × d)]
= (b • d) [d • (a × c)] – (b • c) [d • (a × d)]
= (b • d) [dac] – (b • c) [dad]
= (b • d) [acd],
since [dad] = 0 and [dac] = [acd].

Example 19: If the four vectors a , b , c , d are coplanar, show that (a × b) × (c × d) = 0 .


Solution: a × b is a vector perpendicular to the plane containing a and b.
Similarly c × d is a vector perpendicular to the plane containing c and d.
Since a , b , c , d are all coplanar, therefore the vectors a × b and c × d are
perpendicular to the same plane. Therefore a × b and c × d are parallel.
Now we know that the vector product of two parallel vectors is equal to a zero
vector, therefore (a × b) × (c × d) = 0 .

Example 20: Prove that


(a × b) × (c × d) + (a × c) × (d × b) + (a × d) × (b × c) = − 2 [bcd] a .
Solution: (a × b) × (c × d) = l × (c × d), where l = a × b
= (l • d) c – (l • c) d
= [(a × b) • d)] c – [(a × b) • c] d
= [abd] c – [abc] d . …(1)
Again (a × c) × (d × b) = (a × c) × m, where m = d × b
= (m • a) c – (m • c) a
= [(d × b) • a] c – [(d × b) • c] a
= [dba] c – [dbc] a = – [abd] c – [bcd] a , …(2)
since [dba] = – [abd], as we have changed the cyclic order of the vectors and
[dbc] = [bcd], as the cyclic order has been maintained.
Also (a × d) × (b × c) = (a × d) × n , where n = b × c
= (n • a) d – (n • d) a
= [(b × c) • a] d – [(b × c) • d] a
= [bca] d – [bcd] a = [abc] d – [bcd] a . …(3)
V-24

Adding (1), (2) and (3), we get


(a × b) × (c × d) + (a × c) × (d × b) + (a × d) × (b × c) = − 2 [bcd] a .

Example 21: Prove that


[a × p , b × q , c × r] + [a × q , b × r , c × p] + [a × r , b × p , c × q] = 0.
(Kumaun 2012)
Solution: We have
[a × p , b × q , c × r] = (a × p) • [(b × q) × (c × r)]
= (a × p) • [{(b × q) • r} c – {(b × q) • c} r ]
= (a × p) • {[ bqr ] c – [ bqc ] r }
= [apc] [bqr] – [apr] [bqc] . …(1)
Again [a × q , b × r , c × p] = [b × r , c × p , a × q ]
= (b × r) • [(c × p) × (a × q)]
= (b × r) • [{(c × p) • q } a – {(c × p) • a} q ]
= [bra] [cpq] – [brq] [cpa]. …(2)
and [a × r , b × p , c × q] = [c × q , a × r, b × p]
= (c × q) • [(a × r) × (b × p)]
= (c × q) • [{(a × r) • p } b − {(a × r) • b } p ]
= [cqb] [arp] – [cqp] [arb]. …(3)
Adding (1), (2) and (3) we get
[a × p , b × q , c × r] + [a × q , b × r , c × p] + [a × r , b × p , c × q]
= [apc] [bqr] – [apr] [bqc] + [bra] [cpq]
– [brq] [cpa] + [cqb] [arp] – [cqp] [arb]
= [apc] [bqr] – [apr] [bqc] + [bra] [cpq]
– [bqr] [apc] + [bqc] [apr] – [cpq] [bra]
= 0, since [brq] = – [bqr], [cpa] = – [apc] etc.

Example 22: Prove that [a × b , c × d , e × f ] = [abd] [cef ] – [abc] [def ]


= [abe] [fcd] – [abf ] [ecd] = [cda] [bef ] – [cdb] [aef ].
(Kumaun 2012, 15)
Solution: We have [a × b , c × d , e × f ] = (a × b) • [(c × d) × (e × f )]
= (a × b) • [l × (e × f )], where l = c × d
= (a × b) • [(l • f ) e – (l • e) f ]
= (a × b) • [{(c × d) • f } e – {(c × d) • e} f ]
= [cdf ] [abe] – [cde] [abf ]
= [abe] [fcd] – [abf ] [ecd], since [cdf ] = [fcd] etc.
V-25

Again [a × b , c × d , e × f ] = [c × d , e × f , a × b]
= (c × d) • [(e × f ) × (a × b)]
= (c × d) • [{(e × f ) • b} a – {(e × f ) • a} b]
= [cda] [efb] – [cd b] [efa]
= [cda] [bef ] – [cdb] [aef ].
and [a × b , c × d , e × f ] = [e × f , a × b , c × d]
= (e × f ) • [(a × b) × (c × d)]
= (e × f ) • [{(a × b) • d} c – {(a × b) • c} d]
= [efc] [abd] – [efd] [abc]
= [abd] [cef ] – [abc] [def ].

Example 23: Prove that (b × c) • (a × d) + (c × a) • (b × d) + (a × b) • (c × d) = 0.


Solution: We have
b • a b • d
(b × c) • (a × d) = 
c • a c • d
= (b • a) (c • d) – (c • a) (b • d) …(1)
c • b c • d
Similarly, (c × a) • (b × d) = 
a • b a • d
= (c • b) (a • d) – (a • b) (c • d) …(2)
a • c a • d
and (a × b) • (c × d) = 
b • c b • d
= (a • c) (b • d) – (b • c) (a • d) …(3)
Adding (1), (2) and (3), we get
(b × c) • (a × d) + (c × a) • (b × d) + (a × b) • (c × d) = 0,
since a • b = b • a etc.

Comprehensive Exercise 3

1. Prove the identity a × [a × (a × b)] = (a • a) (b × a). (Kumaun 2011)

2. Establish the identity [a b c] d = [b c d] a + [c a d] b + [a b d] c for any


four vectors a , b , c , d . Hence show that any vector r can always be expressed
as a linear combination of three given non-coplanar vectors.

3. Obtain a set of vectors reciprocal to the three vectors


– i + j + k, i – j + k, i + j + k.
V-26

4. If a , b , c be a set of non-coplanar vectors and


b×c c×a a ×c
a′ = , b′ = , c′ = ,
[abc] [abc] [abc]
then prove that
b′ × c ′ c′ × a′ a ′ × b′
a = ,b = and c = ⋅
[a ′ b ′ c ′ ] [a ′ b ′ c ′ ] [a ′ b ′ c ′ ]
5. If a , b , c and a ′ , b ′ , c ′ are reciprocal system of vectors, prove that
(i) a × a ′ + b × b ′ + c × c ′ = 0
a +b+c
(ii) a ′ × b ′ + b ′ × c ′ + c ′ × a ′ =
[a b c]
(iii) a • a ′ + b • b ′ + c • c ′ = 3.

A nswers 3
1 1 1 1 1 1
3. − i + k, − j + k, i + j.
2 2 2 2 2 2

O bjective T ype Q uestions

Multiple Choice Questions


Indicate the correct answer for each question by writing the corresponding letter from
(a), (b), (c) and (d).
1. The value of i • ( j × k) + j • (k × i) + k • (i × j) is
(a) 0 (b) 1
(c) 2 (d) 3
2. The volume of the parallelopiped whose edges are given by
→ → →
OA = 2i − 3 j, OB = i + j – k , OC = 3i − k is
(a) 1 (b) 4
(c) 2/7 (d) None of these
3. If [a b c] is the scalar triple product of three vectors a , b and c , then [a b c] is
equal to
(a) [b a c] (b) [c b a]
(c) [b c a] (d) [a c b]
V-27

4. If i , j, k constitute an orthogonal right handed triad of unit vectors and a is


any vector, then i × (a × i) + j × (a × j) + k × (a × k) is equal to
(a) a (b) 2a
(c) 3a (d) 0

5. [a ′, b ′, c ′] is qual to
(a) [a, b , c ] (b) [a, b , c ]2
1
(c) (d) None of these
[a, b , c ] (Kumaun 2009)

Fill in the Blank(s)


Fill in the blanks “……” so that the following statements are complete and correct.

1. If { i , j, k} be a set of orthonormal unit vectors, then [i j k] = …… .


→ → →
2. If A , B , C be three non-coplanar vectors, then
→ → → → → →
A• B × C B• A × C
+ = …… .
→ → → → → →
C × A• B C• A × B
3. For any three vectors a , b , c , a × (b × c) + b × (c × a) + c × (a × b) = … .
4. If a , b , c and a ′ , b ′ , c ′ are reciprocal system of vectors, then
a • a ′ + b • b ′ + c • c ′ = …… .
5. If a , b , c are any three coplanar vectors, then (a × b) • c = …… .
(Kumaun 2008)
6. (a × b ) × c = ……… (Kumaun 2015)
7. If i, j, k are unit vectors, then (i × j) • k = ……… (Kumaun 2009)
8. Value of vector [(b+c) × (c +a)] = ………… (Kumaun 2013)

True or False
Write ‘T’ for true and ‘F’ for false statement.

1. If x • a = x • b = x • c = 0, for some non-zero vector x, then [a b c] = 0.

2. If { i , j, k} be a set of orthonormal unit vectors, then i × ( j × k) ≠ 0 .

3. The orthonormal vector triads i , j, k form a self reciprocal system.


V-28

A nswers

Multiple Choice Questions


1. (d) 2. (b) 3. (c) 4. (b) 5. (c)

Fill in the Blank(s)


1. 1 2. 0 3. 0 4. 3 5. 0
6. (c • a) b – (c • b) a 7. 1 8. b × c +c × a + b × a

True or False
1. T 2. F 3. T

¨
V-29

D ifferentiation of V ectors

1 Vector Function
e know that a scalar quantity possesses only magnitude and has no concern
W with direction. A single real number gives us a complete representation of a
scalar quantity. Thus a scalar quantity is nothing but a real number.
Let D be any subset of the set of all real numbers. If to each element t of D , we
associate by some rule a unique real number f (t), then this rule defines a scalar
function of the scalar variable t . Here f (t) is a scalar quantity and thus f is a scalar
function.
In a similar manner we define a vector function.
Let D be any subset of the set of all real numbers. If to each element t of D, we associate by some
rule a unique vector f (t), then this rule defines a vector function of the scalar variable t.
Here f (t) is a vector quantity and thus f is a vector function.
We know that every vector can be uniquely expressed as a linear combination of
three fixed non-coplanar vectors. Therefore we may write
f (t) = f1 (t) i + f 2 (t) j + f 3 (t) k
V-30

where i , j , k denote a fixed right handed triad of three mutually perpendicular


non-coplanar unit vectors.

2 Scalar Fields and Vector Fields


If to each point P( x , y , z ) of a region R in space there corresponds a unique scalar
f ( P), then f is called a scalar point function and we say that a scalar field f has
been defined in R .

Examples: (1) The temperature at any point within or on the surface of earth at a
certain time defines a scalar field.
(2) f ( x , y , z ) = x 2 − y 3 − 3z 2 defines a scalar field.
If to each point P ( x , y , z ) of a region R in space there corresponds a unique vector
f ( P), then f is called a vector point function and we say that a vector field f has
been defined in R.

Examples: (1) If the velocity at any point ( x, y, z ) of a particle moving in a curve


is known at a certain time, then a vector field is defined.
(2) f ( x , y , z ) = xy 2 i + 3 yz 3 j − 2 x 2 zk defines a vector field.

3 Limit and Continuity of a Vector Function


Definition 1: A vector function f (t) is said to tend to a limit l, when t tends to t0 , if for any
given positive number ε , however small, there corresponds a positive number δ such that
|f (t) − l | < ε whenever 0 < |t − t0 | < δ.

If f (t) tends to a limit l as t tends to t0 , we write lim f (t) = l .


t → t0
Definition 2: A vector function f (t) is said to be continuous for a value t0 of t if
(i) f (t0 ) is defined and
(ii) for any given positive number ε, however small, there corresponds a positive number δ
such that |f (t) − f (t0 )| < ε , whenever |t − t0| < δ.
Further a vector function f (t) is said to be continuous if it is continuous for every value of t for
which it has been defined.
We shall give here (without proof ) some important results about the limits and
continuity of a vector function.

Theorem 1: The necessary and sufficient condition for a vector function f (t) to be

continuous at t = t0 is that lim f (t) = f (t0 ).


t → t0 (Purvanchal 2014)
V-31

Theorem 2: If f (t) = f1 (t)i + f 2 (t) j + f 3 (t)k , then f (t) is continuous if and only if
f1 (t), f 2 (t), f 3 (t) are continuous.
Theorem 3: Let f (t) = f1 (t)i + f 2 (t) j + f 3 (t)k and l = l1 i + l2 j + l3 k .

Then the necessary and sufficient conditions that lim f (t) = l are
t → t0
lim f (t) = l , lim f 2 (t) = l2 and lim f 3 (t) = l3 .
t → t0 1 1
t → t0 t → t0
Theorem 4: If f (t), g (t) are vector functions of scalar variable t and φ (t) is a scalar
function of scalar variable t , then
(i) lim [f (t) ± g(t)] = lim f (t) ± lim g(t)
t → t0 t → t0 t → t0

(ii) lim [f (t) • g(t)] =  lim f (t) •  lim g(t)


t → t0  t→ t   t→ t 
 0   0 

(iii) lim [f (t) × g(t)] =  lim f (t) ×  lim 


t → t0  t→ t   t → t g(t)
 0   0 

(iv) lim [φ (t) f (t)] =  lim φ (t)  lim f (t)


t → t0  t→ t   t→ t 
 0  0 

(v) lim |f (t) | = 


 lim f (t) 
⋅
t → t0  t → t0 

4 Derivative of a Vector Function with Respect to a Scalar


Definition: Let r = f (t) be a vector function of the scalar variable t. We define
r + δr = f (t + δt).
∴ δr = f (t + δt) − f (t).
δr f (t + δt) − f (t)
Consider the vector = ⋅
δt δt
δr f (t + δt) − f (t)
If lim = lim exists, then the value of this limit, which we shall
δt → 0 δt δt → 0 δt
dr
denote by , is called the derivative of the vector function r with respect to the scalar t.
dt
Symbolically
dr (r + δr) − r f (t + δt) − f (t)
= lim = lim ⋅
dt δt → 0 δt δt → 0 δt
dr δr
If exists, then r is said to be differentiable. Since is a vector quantity,
dt δt
dr
therefore is also a vector quantity.
dt
V-32

dr d2 r
Successive Derivatives: If we differentiate again, we get 2 which is called
dt dt
the second derivative of r w.r.t. t , and so on.
Thus, we continue differentiating successively upto n times and get
dr d2 r d3 r d nr
, , , …… , ,
dt dt 2 dt 3 dt n
d nr
where n
is called the n th diff. coeff. (or derivative) of r w.r.t. t. (Here diff. coeff.
dt
means differential coefficient.)
dr d2 r . ..
We often represent , 2
, …… by r , r, …… respectively.
dt dt
A scalar or vector function of t is called differentiable of order n if its n th order
derivative exists.

5 Differentiation Formulae
Theorem: If a, b and c are differentiable vector functions of a scalar t and φ is a
differentiable scalar function of the same variable t, then
d da db
1. (a + b) = +
dt dt dt
d db da
2. (a • b) = a • + •b
dt dt dt
d db da
3. (a × b) = a × + × b
dt dt dt
d da dφ
4. (φa) = φ + a
dt dt dt
[a b c] =  b c + a
db  
c + a b 
d da dc
5.
dt 
 dt  
  dt     dt  (Meerut 2013B; Purvanchal 08)

6.
d
{a × (b × c)} =
da
× (b × c) + a ×  db × c + a ×  b × dc ⋅
dt dt  dt   dt 
(Meerut 2000; Purvanchal 08)

d {( a + δa) + ( b + δb)} − (a + b)
Proof: 1. (a + b) = lim
dt δ t → 0 δt
δa + δb  δa δb
= lim = lim  + 
δt → 0 δt δt → 0  δ t dt 
δa δb d a d b
= lim + lim = + ⋅
δt → 0 δ t δt → 0 dt dt dt
V-33

Thus the derivative of the sum of two vectors is equal to the sum of their derivatives, as it is also
in Scalar Calculus.
d da d b
Similarly we can prove that (a – b) = − ⋅
dt dt dt
In general if r1 , r2 ,……, r n are vector functions of a scalar t, then
d dr dr dr
(r1 + r2 + …… + r n ) = 1 + 2 + …… + n ⋅
dt dt dt dt
d (a + δa) • ( b + δb) − a • b
2. (a • b) = lim
dt δt → 0 δt

= lim a • b + a • δb + δa • b + δa • δb − a • b
δt → 0 δt

= lim a • δb + δa • b + δa • δb
δt → 0 δt

lim  δb δa δa 
= a • + • b+ • δb
δt → 0  δ t δ t δ t 
lim a • δb + lim δa • b + lim δa • δb
=
δt → 0 δt δt → 0 δt δt → 0 δt
db da da
= a• + • b+ • 0, since δb → zero vector as δt → 0
dt dt dt
db da
= a• + •b
dt dt
d
Note: We know that a • b = b • a. Therefore while evaluating (a • b), we should
dt
not bother about the order of the factors.

d (a + δa) × ( b + δb) − a × b
3. (a × b) = lim
dt δt → 0 δt

= lim a × b + a × δb + δa × b + δa × δb − a × b
δt → 0 δt

= lim a × δb + δa × b + δa × δb
δt → 0 δt

= lim  a × δb + δa × b + δa × δb
 
δt → 0  δt δt δt 

= lim a × δb + lim δa × b + lim δa × δb


δt → 0 δt δt → 0 δt δt → 0 δt
db da da
=a× + × b+ × 0 , since δb → zero vector as δt → 0
dt dt dt
db da db da
=a× + × b+ 0 = a × + × b.
dt dt dt dt
V-34

Note: We know that cross product of two vectors is not commutative because
d
a × b = – b × a . Therefore while evaluating (a × b), we must maintain the order
dt
of the factors a and b .

d (φ + δφ)(a + δa) − φa
4. (φa) = lim
dt δ t → 0 δt

= lim φa + φδa + δφa + δφδa − φa


δt → 0 δt

= lim φδa + δφa + δφδa


δt → 0 δt

= lim  φ δa + δφ a + δφ δa
 
δt → 0  δt δt δt 

= lim φ δa + lim δφ a + lim δφ δa


δt → 0 δt δt → 0 δt δt → 0 δt
da dφ dφ
=φ + a+ 0 , since δa → zero vector as δt → 0
dt dt dt
da dφ da dφ
=φ + a+0=φ + a.
dt dt dt dt
Note: φa is the multiplication of a vector by a scalar. In the case of such
multiplication we usually write the scalar in the first position and the vector in the
second position.
d d d da
5. [a b c] = {a • (b × c)} = a • ( b × c) + • ( b × c) [by rule (2)]
dt dt dt dt

= a •  b × × c +
dc db da
+ • ( b × c) [by rule (3)]
 dt dt  dt

= a •  b ×  + a •  × c +
dc db da
• ( b × c)
 dt   dt  dt

= a b  + a
db   da
bc 
dc
c +
 dt   dt   dt 

= b c  + a
db  
c + a b ⋅
da dc
 dt   dt   dt 
Note: Here [abc] is the scalar triple product of three vectors a , band c. Therefore
d
while evaluating [abc] we must maintain the cyclic order of each factor.
dt
d d da
6. {a × (b × c)} = a × (b × c) + × (b × c) [by rule (3)]
dt dt dt

= a ×  × c + b ×  +
db dc da
× (b × c)
 dt dt  dt
V- 35

= a ×  × c + a ×  b ×  +
db dc da
× (b × c)
 dt   dt  dt

× (b × c) + a ×  × c + a ×  b ×  ⋅
da db dc
=
dt  dt   dt 

6 Derivative of a Function of a Function


Suppose r is a differentiable vector function of a scalar variable s and s is a
differentiable scalar function of another scalar variable t.Then r is a function of t.
An increment δt in t produces an increment δr in r and an increment δs in s. When
δt → 0, δr → 0 and δs → 0.
dr δr  δs δr 
We have = lim = lim  
dt δt → 0 δt δt → 0  δt δs 
 δs  lim δr  ds dr
=  lim    = ⋅
δ t → 0 δt  δt → 0 δs  dt ds

7 Derivative of a Constant Vector


A vector is said to be constant only if both its magnitude and direction are fixed. If
either of these changes then the vector will change and thus it will not be constant.
Let r be a constant vector function of the scalar variable t. Let r = c , where c is a
constant vector. Then r + δr = c .
δr 0
∴ δr = 0 (zero vector). ∴ = = 0.
δt δt
∴ lim δr = lim 0 = 0 . ∴
dr
= 0 (zero vector).
δt → 0 δt δt → 0 dt
Thus the derivative of a constant vector is equal to the null vector.

8 Derivative of a Vector Function in Terms


of its Components
Let r be a vector function of the scalar variable t . (Avadh 2014)
Let r = xi + yj + zk where the components x , y , z are scalar functions of the
scalar variable t and i, j, k are fixed unit vectors.
We have r + δr = ( x + δx) i + ( y + δy) j + (z + δz ) k .
∴ δr = (r + δr) − r = δx i + δyj + δzk .
δr δx δy δz
∴ = i+ j+ k.
δt δt δt δt
δy
∴ lim δr = lim  δx i + j+
δz 
k ⋅

δt → 0 δt δt → 0  δt δt δt 
V-36

dr dx dy dz
∴ = i+ j+ k.
dt dt dt dt
Thus in order to differentiate a vector we should differentiate its components.
Note: If r = xi + yj + zk , then sometimes we also write it as r = ( x, y, z ). In this
dr  dx dy dz  d 2 r  d2 x d y d2 z 
2
notation = , , , 2 = , , 2 , and so on.
dt  dt dt dt  dt  dt 2 dt 2 dt 

Alternative Method:
We have r = xi + yj + zk , where i , j, k are constant vectors and so their
derivatives will be zero.
dr d d d d
Now, = ( xi + yj + zk ) = ( xi ) + ( yj) + (zk )
dt dt dt dt dt
dx di dy dj dz dk
= i+ x + j+ y + k +z
dt dt dt dt dt dt
dx dy dz di
= i+ j+ k , since , etc. vanish.
dt dt dt dt

9 Some Important Results


Theorem 1: The necessary and sufficient condition for the vector function a (t) to be
da
constant is that = 0.
dt (Purvanchal 2014)
Proof: The condition is necessary. Let a (t) be a constant vector function of
the scalar variable t . Then a (t + δt) = a (t). We have
da a (t + δt) − a (t) 0
= lim = lim = 0.
dt δt → 0 δt δt → 0 δt
Therefore the condition is necessary.
da
The condition is sufficient. Let = 0. Then to prove that a is a constant
dt
vector. Let
a (t) = a1 (t) i + a2 (t) j + a3 (t) k .
da da1 da da
Then = i + 2 j+ 3 k.
dt dt dt dt
da da1 da2 da
Therefore = 0 gives, i+ j + 3 k = 0.
dt dt dt dt
Equating to zero the coefficients of i, j and k , we get
da1 da da
= 0, 2 = 0, 3 = 0.
dt dt dt
Hence a1 , a2 , a3 are constant scalars i. e., they are independent of t . Therefore a (t)
is a constant vector function.
V-37

Theorem 2: If a is a differentiable vector function of the scalar variable t and if | a | = a,


then
d 2 da da da
(i) (a ) = 2a ; (ii) a • =a ⋅
dt dt dt dt
Proof: (i) We have a2 = a • a = (a) (a) cos 0 = a2 .
d 2 d 2 da
Therefore (a ) = (a ) = 2a ⋅
dt dt dt
d 2 d da da da
(ii) We have (a ) = (a • a) = •a+a• = 2a • ⋅
dt dt dt dt dt
d 2 d 2 da
Also (a ) = (a ) = 2a ⋅
dt dt dt
da da da da
∴ 2a • = 2a or a• =a ⋅
dt dt dt dt
da
Theorem 3: If a has constant length (fixed magnitude), then a and are
dt
da
perpendicular provided  ≠ 0.
 dt 
Proof: Let |a | = a = constant. Then a • a = a2 = constant.
d da da
∴ (a • a) = 0 or •a+a• =0
dt dt dt
da da
or 2a • =0 or a• = 0.
dt dt
da
Thus the scalar product of two vectors a and is zero.
dt
da da
Therefore a is perpendicular to provided is not null vector i. e., provided
dt dt
da ≠ 0.
dt
Thus the derivative of a vector of constant length is perpendicular to the vector provided the
vector itself is not constant.
Theorem 4: The necessary and sufficient condition for the vector a (t) to have constant
da
magnitude is a • = 0. (Meerut 2001, 04B, 06, 07, 11, 13; Avadh 13;
dt
Kashi 10, 12; Purvanchal 11)
Proof: Let a be a vector function of the scalar variable t . Let|a| = a = constant.
Then a • a = a2 = constant.
d da da
∴ (a • a) = 0 or a• + •a=0
dt dt dt
da da
or 2a • =0 or a• = 0.
dt dt
Therefore the condition is necessary.
V-38

da
Condition is sufficient. If a • = 0, then
dt
da da d
a• + •a=0 or (a • a) = 0
dt dt dt
or a • a = constant or a2 = constant
or a2 = constant or |a | = constant
Theorem 5: If a is a differentiable vector function of the scalar variable t, then
d  d2 a
 a ×  = a ×
da

dt  dt  dt 2
Proof: We have
d  d2 a d2 a
 a ×  =
da da da
× +a× 2 =0+a× ,
dt  dt  dt dt dt dt 2
da d2 a
since the cross product of two equal vectors is zero = a × ⋅
dt dt 2
Theorem 6: The necessary and sufficient condition for the vector a (t) to have constant
da
direction is a × = 0.
dt (Meerut 2004B, 06, 07, 10, 11, 13B; Avadh 14)
Proof: Let a be a vector function of the scalar variable t . Let A be a unit vector in
the direction of a . If a be the magnitude of a , then a = aA .
da dA da
∴ =a + A.
dt dt dt

= (a A) ×  a
da dA da 
Hence a× + A
dt  dt dt 
dA da
= a2 A × +a A×A
dt dt
dA
= a2 A × [ ∵ A × A = 0] …(1)
dt
The condition is necessary. Suppose a has a constant direction. Then A is a
constant vector because it has constant direction as well as constant magnitude.
dA
Therefore = 0.
dt
da
∴ From (1), we get a × = a2 A × 0 = 0.
dt
Therefore the condition is necessary.
da
The condition is sufficient. Suppose that a × = 0.
dt
Then from (1), we get
dA dA
a2 A × =0 or A× = 0. …(2)
dt dt
V-39

dA
Since A is of constant length, therefore A • = 0. …(3)
dt
dA
From (2) and (3), we get = 0.
dt
Hence A is a constant vector i. e., the direction of a is constant.

10 Curves in Space
A curve in a three dimensional Euclidean space may be regarded as the intersection
of two surfaces represented by two equations of the form
F1 ( x, y, z ) = 0, F2 ( x, y, z ) = 0.
It can be easily seen that the parametric equations of the form
x = f1 (t), y = f 2 (t), z = f 3 (t)
where x , y , z are scalar functions of the scalar t, also represent a curve in three
dimensional space. Here ( x, y, z ) are the coordinates of a current point on the
curve. The scalar variable t may range over a set of values a ≤ t ≤ b .
In vector notation an equation of the form r = f (t), represents a curve in three
dimensional space if r is the position vector of a current point on the curve. As t
changes, r will give position vectors of different points on the curve. The vector f (t)
can be expressed as
f1 (t)i + f 2 (t) j + f 3 (t)k .
Also if ( x, y, z ) are the coordinates of a current point on the curve whose position
vector is r, then r = xi + yj + zk .
Therefore the single vector equation r = f (t)
i. e., xi + yj + zk = f1 (t)i + f 2 (t) j + f 3 (t)k
is equivalent to the three parametric equations
x = f1 (t), y = f 2 (t), z = f 3 (t).
Thus a curve in a space may be defined as the locus of a point whose coordinates may be
expressed as a function of a single parameter.
For example, the two equations
x2 y2 z
2
− 2
= 1, x = a cosh
a b a
specify a curve in three dimensional space. The parametric equations of this curve
are x = a cosh u, y = b sinh u, z = au.
And its vectorial equation is
r = a cosh u i + b sinh u j + au k .
The vector equation r = a cos ti + b sin tj + 0 k represents an ellipse, as for different
values of t, the end point of r describes an ellipse.
Similarly r = at 2 i + 2atj + 0 k is the vector equation of a parabola.
The terms skew, twisted or tortuous are often used for curves in a space.
V-40

11 Geometrical Significance of dr / dt and Unit Tangent


Vector to a Curve
Geometrical Significance of dr / dt :
Let r = f (t) be the vector equation of a curve
in space. Let r and r + δr be the position
vectors of two neighbouring points P and Q
on this curve. Thus we have

OP = r = f (t)

and OQ = r + δr = f (t + δt).
→ → →
∴ PQ = OQ − OP
= (r + δr) − r= δr.
δr
Thus is a vector parallel to the chord PQ .
dt
As Q → P i. e., as δt → 0, chord PQ → tangent at P to the curve.

∴ lim δr = dr is a vector parallel to the tangent at P to the curve r = f (t).


δt → 0 δt dt
Unit tangent vector to a curve:
Suppose in place of the scalar parameter t , we take the parameter as s where s
denotes the arc length measured along the curve from any convenient fixed point C
on the curve. Thus arc CP = s and arc CQ = s + δs .
dr
In this case will be a vector along the tangent at P to the curve and in the
ds
direction of s increasing. Also we have
δr |δr |
  = lim 
d r     = lim = lim
chord PQ
= 1.
 ds  δ t → 0  δs  δ t → 0 arc PQ Q → P arc PQ
dr
Thus is a unit vector along the tangent at P in the direction of s increasing. We
ds
denote it by t. Note that t always points in the direction of motion along the curve.

12 Application to Velocity and Acceleration


Velocity: If the scalar variable t be the time and r be the position vector of a
moving particle P with respect to the origin O, then δr is the displacement of the
particle in time δt .
δr
The vector is the average velocity of the particle during the interval δt . If v
δt
δr dr
represents the velocity vector of the particle at P, then v = lim = ⋅
δt → 0 δ t dt
V-41

dr
Since is a vector along the tangent at P to the curve in which the particle is
dt
moving, therefore the direction of velocity is along the tangent.
δv
Acceleration: If δv be the change in the velocity v during the time δt , then is
δt
the average acceleration during that interval. If a represents the acceleration of the
particle at time t , then
δv dv d  dr  d2 r
a = lim = =   = ⋅
δt → 0 δt dt dt  dt  dt 2

Example 1: If r = sin t i + cos t j + tk , find


dr d2 r
(i) , (Kashi 2013) (ii) ,
dt dt 2 (Meerut 2010B)
dr d r
2
(iii)  , (Kashi 2013) (iv)  2  .
dt dt  (Meerut 2009)
di
Solution: Since i , j , k are constant vectors, therefore = 0 etc. Therefore
dt
dr d d d
(i) = (sin t) i + (cos t) j + (t) k = cos t i − sin t j + k .
dt dt dt dt
d2 r d  dr  = d (cos t) i − d (sin t) j + dk
(ii) =
dt 2 dt  dt  dt dt dt
= − sin t i − cos t j + 0 = − sin t i − cos t j.

(iii) dr = √ [(cos t)2 + (− sin t)2 + (1)2 ] = √ 2 .


dt
d2 r
(iv)   = √ [(− sin t)2 + (− cos t)2 ] = 1.
2
dt 
Example 2: If a, b are constant vectors, ω is a constant, and r is a vector function of the
scalar variable t given by r = cos ωt a + sin ωt b , show that
d2 r
(i) 2
+ ω2 r = 0
dt (Rohilkhand 2006)
dr
(ii) r× = ωa × b .
dt (Bundelkhand 2004; Kanpur 05)
Solution: Since a, b are constant vectors, therefore
da db
= 0, = 0.
dt dt
V-42

dr d d
(i) = (cos ωt) a + (sin ωt) b
dt dt dt
= − ω sin ωt a + ω cos ωt b .
2
d r
∴ 2
= − ω 2 cos ωt a − ω 2 sin ωt b
dt
= − ω 2 (cos ωta + sin ωtb) = − ω 2 r.
d2 r
∴ 2
+ ω 2 r = 0.
dt
dr
(ii) r× = (cos ωta + sin ωt b) × (− ω sin ωta + ω cos ωtb)
dt
= ω cos 2 ωt a × b − ω sin2 ωt b × a [ ∵ a × a = 0, b × b = 0]
2 2
= ω cos ωt a × b + ω sin ωt a × b
= ω (cos 2 ωt + sin2 ωt) a × b = ωa × b .
Example 3: If r = a cos t i + a sin t j + at tan α k , find
dr d2 r  dr d2 r d3 r 
 ×  and  , , 3⋅
dt dt 2   dt dt
2
dt  (Purvanchal 07, 10; Agra 14)
dr
Solution: We have = − a sin t i + a cos t j + a tan α k
dt
d2 r  dk 
2
= − a cos t i − a sin t j ,  ∵ dt = 0
dt
d3 r
= a sin t i − a cos t j .
dt 3
 i j k 
dr d2 r 
∴ × 2 = − a sin t a cos t a tan α 
dt dt  
 − a cos t − a sin t 0 

= a2 sin t tan α i − a2 cos t tan α j + a2 k .


dr d2 r
∴  ×  = √ (a4 sin2 t tan2 α + a4 cos 2 t tan2 α + a4 )
dt 2
 dt 
= a2 sec α.
 dr d2 r d3 r   dr d2 r  d3 r
Also  , , = × 2 • 3
 dt dt
2
dt 3   dt dt  dt
= (a2 sin t tan α i − a2 cos t tan α j + a2 k ) • (a sin t i − a cos t j)
= a3 sin2 t tan α i • i + a3 cos 2 t tan α j • j [∵ i • j = 0 etc.]
= a3 tan α (sin2 t + cos 2 t) [∵ i • i = 1 = j • j]
= a3 tan α.
V-43

du dv d
Example 4: If = w × u, = w × v, show that (u × v) = w × (u × v).
dt dt dt
(Garhwal 2003; Bundelkhand 09)

Solution: We have
d du dv
(u × v) = × v+ u × = ( w × u) × v + u × ( w × v)
dt dt dt
= ( v • w) u − ( v • u) w + (u • v) w − (u • w) v
= ( v • w) u − (u • w) v [∵ u • v = v • u]
= (w • v) u – (w • u) v = w × (u × v).
Example 5: If R be a unit vector in the direction of r, prove that
dR 1 dr
R× = 2 r× , where r = | r |.
dt r dt (Bundelkhand 2008, 11)
1
Solution: We have r = rR ; so that R = r.
r
dR 1 dr 1 dr
∴ = − r.
dt r dt r 2 dt
dR 1 1 d r 1 dr 
Hence R× = r× − 2 r
dt r  r dt r dt 
1 dr 1 dr
= 2
r× − r×r
r dt r 3 dt
1 dr
= r× ⋅ [∵ r × r = 0]
r2 dt
Example 6: If r is a vector function of a scalar t and a is a constant vector, m a constant,
differentiate the following with respect to t :
(i) r • a , (ii) r × a ,
dr dr
(iii) r × , (iv) r • ,
dt dt
dr 2
(vi) m   ,
1
(v) r 2 + 2 ,
r  dt 
r+a r×a
(vii) 2 , (viii) ⋅
r +a 2 r •a

Solution: (i) Let R = r • a. [Note that r • a is a scalar]


dR dr da
Then = •a+r •
dt dt dt
=
dr
•a+r •0 ∵ da
= 0, as a is constant 
dt  dt 
dr
= •a+0
dt
dr
= • a.
dt
V-44

(ii) Let R = r × a .
dR dr da
Then = ×a+r×
dt dt dt

=
dr
×a+r×0 ∵ da
= 0
dt  dt 
dr dr
= ×a+0= × a.
dt dt
dr
(iii) Let R = r × ⋅
dt

dR dr dr d2 r
Then = × +r× 2
dt dt dt dt
d2 r ∵ = 0
dr dr
=0+r× ×
dt 2  dt dt 
2
d r
=r× ⋅
dt 2
dr
(iv) Let R = r • ⋅
dt
dR dr dr d2 r
Then = • +r• 2
dt dt dt dt
2
d2 r
=  
dr
+r• ⋅
 dt  dt 2
1
(v) Let R = r2 + ⋅
r2
dR d 2 d  1
Then = (r ) +  
dt dt dt  r 2 
d 2 d  1
= (r ) +   , where r = | r |
dt dt  r 2 
dr 2 dr
= 2r − ⋅
dt r 3 dt
dr 2
(vi) Let R = m   ⋅
 dt 
2
Then
dR
=m
d  dr 
dt dt  dt 

dr d2 r  dr 2 dr 
=2m • Note : =2r • 
dt dt 2  dt dt 

dr d2 r
=2m • ⋅
dt dt 2
V-45

r+a
(vii) Let R = ⋅
r 2 + a2

dR 1 d  d  1  
= 2 (r + a) +    (r + a)
2 
Then 2 dt 2
dt (r + a )  dt  r + a  

[Note that r 2 + a2 is a scalar]

=
1  dr + da  −  1 d 2 
(r + a2 ) (r + a)
 dt   2
r 2
+a 2 dt  (r + a ) dt
2 2

dr
2r•
1 dr dt (r + a)
= −
r 2 + a2 dt (r 2 + a2 )2

 ∵ da = 0, d r 2 = 2r • dr , d a2 = 0
 dt dt dt dt 
r×a
(viii) Let R = ⋅
r •a

dR 1 d  d  1 
Then = (r × a) +     (r × a)
dt r • a dt  dt  r • a  
[Note that r • a is a scalar quantity]
1 d r d a   1 d 
=  ×a+r×  − (r • a) (r × a)
r •a  dt dt   (r • a) dt
2

dr
×a 
= dt

−
1  dr • a + r • da   (r × a)

r •a  (r • a)  dt
2 dt  
dr dr
×a •a
= dt − dt 2 (r × a). ∵ da
= 0
r •a (r • a)  dt 

Example 7: Find
d  dr d2 r  d2  dr d2 r 
(i) r, , ; (ii) r, , ;
dt  dt dt 2  dt 2  dt dt 2 

d   dr d2 r  
(iii) r ×  × 2  ⋅
dt   dt dt   (Kumaun 2010)
 dr d r  2
Solution: (i) Let R = r, , 2  ⋅ Then R is the scalar triple product of three
 dt dt 
dr d2 r
vectors r, and 2 ⋅ Therefore using the rule for finding the derivative of a scalar
dt dt
triple product, we have
V-46

dR  dr dr d2 r   d2 r d2 r   dr d3 r 
= , ,  + r, ,  + r, , 
dt  dt dt dt 2   dt 2 dt 2   dt dt 3 

 dr d3 r 
= r, , 3 ,
 dt dt 
since scalar triple products having two equal vectors vanish.
 dr d2 r 
(ii) Let R = r, , 2  ⋅ Then as in part (i)
 dt dt 
dR  dr d3 r 
= r, , ⋅
dt  dt dt 3 
Differentiating again, we get
d2 R  dr dr d3 r   d2 r d3 r   dr d4 r 
= , , 3  + r, 2 , 3  + r, , 
dt 2  dt dt dt   dt dt   dt dt 4 
 d2 r d3 r   dr d4 r 
= r, 2 , 3  + r, , ⋅
 dt dt   dt dt 4 
 dr d2 r 
(iii) Let R = r ×  × 2⋅
 dt dt 
Then R is the vector triple product of three vectors. Therefore using the rule for
finding the derivative of a vector triple product, we have

dR dr  dr d2 r   d2 r d2 r   dr d3 r 
= × × 2 +r × 2 × 2 +r × × 3
dt dt  dt dt   dt dt   dt dt 
dr  dr d2 r   dr d3 r 
= × × 2 +r × × 3 ,
dt  dt dt   dt dt 
d2 r d2 r
since × = 0 , being vector product of two equal vectors.
dt 2 dt 2

Example 8: If a = sin θ i + cos θ j + θk , b = cos θ i − sin θ j − 3k , and


d π
c = 2i + 3 j − 3k , find {a × ( b × c)} at θ = ⋅
dθ 2 (Kumaun 2010)
 i j k 
Solution: We have b × c = cos θ − sin θ − 3
 
 2 3 − 3

= (3 sin θ + 9) i + (3 cos θ − 6) j + (3 cos θ + 2 sin θ) k .

 i j k 
∴ a × (b × c) =  sin θ cos θ θ 
 
3 sin θ + 9 3 cos θ − 6 3 cos θ + 2 sin θ 
V-47

= (3 cos 2 θ + 2 sin θ cos θ − 3θ cos θ + 6θ) i


+ (3θ sin θ + 9θ − 3 sin θ cos θ − 2 sin2 θ) j
+ (− 6 sin θ − 9 cos θ) k .
d
∴ {a × (b × c)} = (−6 cos θ sin θ + 2 cos θ − 2 sin2 θ
2

− 3 cos θ + 3θ sin θ + 6) i + (3 sin θ + 3θ cos θ + 9
− 3 cos 2 θ + 3 sin2 θ − 4 sin θ cos θ) j
+ (−6 cos θ + 9 sin θ) k .
Putting θ = π / 2 , we get the required derivative

= 4 + π i + 15 j + 9k .
3
 2 
Example 9: A particle moves along the curve x = 4 cos t , y = 4 sin t , z = 6t . Find the
1
velocity and acceleration at time t = 0 and t = π . Find also the magnitudes of the velocity
2
and acceleration at any time t. (Bundelkhand 2007, 14)
Solution: Let r be the position vector of the particle at time t .
Then r = xi + yj + zk = 4 cos t i + 4 sin t j + 6t k . If v is the velocity of the
particle at time t and a its acceleration at that time, then
dr
v= = − 4 sin t i + 4 cos t j + 6k
dt
d2 r
and a = 2 = − 4 cos t i − 4 sin t j.
dt
Magnitude of the velocity at time t = | v|
= √ (16 sin2 t + 16 cos 2 t + 36) = √ (52) = 2 √ (13).
Magnitude of the acceleration = |a | = √ (16 cos 2 t + 16 sin2 t) = 4.
At t = 0, v = 4 j + 6 k , a = − 4 i.
1
At t = π , v = − 4 i + 6 k , a = − 4 j.
2

Ex am ple 10: A par ti cle moves along the curve x = t 3 + 1, y = t 2 , z = 2t + 5, where t


is the time. Find the com po nents of its ve loc ity and ac cel er a tion at t = 1in the di rec tion
i + j + 3k . (Purvanchal 2012)
Solution: If r is the position vector of any point ( x , y , z ) on the given curve, then
r = xi + yj + zk = (t 3 + 1) i + t 2 j + (2t + 5) k .
dr
Velocity = v = = 3t 2 i + 2tj + 2k = 3i + 2 j + 2k at t = 1.
dt
d2 r d  dr  = 6ti + 2 j = 6i + 2 j at t = 1.
Acceleration = a = =
dt 2 dt  dt 
Now the unit vector in the given direction i + j + 3k
V-48

i + j + 3k i + j + 3k
= = = b , say.
|i + j + 3k| √ (11)
∴ the component of velocity in the given direction
(3i + 2 j + 2k ) • (i + j + 3k ) 11
= v• b= = = √ (11);
√ (11) √ (11)
and the component of acceleration in the given direction
(6i + 2 j) • (i + j + 3k ) 8
= a • b= = ⋅
√ (11) √ (11)

Comprehensive Exercise 1

1. (i) If r is the position vector of a moving point and r is the modulus of r ,


dr dr dr dr
show that r • =r ⋅ Interpret the relations r • = 0 and r × = 0.
dt dt dt dt
(ii) If r × dr = 0, show that rɵ = constant. (Garhwal 2001; Meerut 05B;
Purvanchal 12; Bundelkhand 12; Avadh 13)
dr d2 r
2. (i) If r = (t + 1) i + (t 2 + t + 1) j + (t 3 + t 2 + t + 1) k , find and 2 ⋅
dt dt
(ii) If r = t 2 i − t j + (2t + 1) k , find at t = 0, the values of
dr d2 r dr d2 r
, , ,  ⋅
dt dt 2 dt dt 2  (Kanpur 2002; Bundelkhand 2010, 13)

3. Show that ^ r = (r × d r) / r 2 , where r = r ^


r × d^ r.
4. (i) The position vector of a moving particle at time t is given by
r = (3t − 4) i + (t 2 − 2) j + 4t 3 k. Find its velocity and acceleration at
time t = 2. (Kanpur 2005; Bundelkhand 13)
 1  dr
(ii) If r = t 3 i + 2t 3 −  j , show that r × = k.
 5t 2  dt (Agra 2007)

(iii) If r = (cos nt) i + (sin nt) j , where n is a constant and t varies, show that
dr
r× = nk . (Garhwal 2002)
dt
5. (i) If r = (sinh t) a + (cosh t) b, where a and b are constant vectors, then
d2 r
show that = r.
dt 2
(ii) If u = t 2 i − tj + (2t + 1) k and v = (2t − 3) i + j − tk , find
d
(u • v), when t = 1.
dt (Bundelkhand 2007)
V-49

6. (i) If r = e n t a + e − n t b , where a , b are constant vectors, show that


d2 r
2
− n2 r = 0 . (Kumaun 2000; Rohilkhand 07; Agra 07; Kashi 14)
dt
(ii) Show that r = a e mt + b e nt , where a and b are the constant vectors, is the
d2 r dr
solution of the differential equation 2
− (m + n) + mn r = 0.
dt dt
Hence solve the equation
d2 r dr dr
2
− − 2r = 0, where r = i and = j for t = 0.
dt dt dt
ct
(iii) If r = a sin ωt + b co s ωt + 2 sin ωt, prove that
ω
d2 r 2c
+ ω2 r = cos ωt,
dt 2 ω
where a , b, c are constant vectors and ω is a constant scalar.
7. (i) A particle moves along the curve x = e − t , y = 2 cos 3t, z = 2 sin 3t.
Determine the velocity and acceleration at any time t and their
magnitudes at t = 0. (Bundelkhand 2005; Rohilkhand 08)
(ii) Show that if a , b, c are constant vectors, then r = at 2 + bt + c is the
path of a particle moving with constant acceleration.
8. A particle moves so that its position vector is given by r = cos ωt i + sin ωt j
where ω is a constant; show that
(i) the velocity of the particle is perpendicular to r,
(ii) the acceleration is directed towards the origin and has magnitude
proportional to the distance from the origin,
dr
(iii) r × is a constant vector.
dt
9. If A = 5t 2 i + t j − t 3 k and B = sin t i − cos t j , find
d d d
(i) (A • B) ; (ii) (A × B) ; (iii) (A • A). (Agra 2005)
dt dt dt
10. Prove the following :
d  d2 b d2 a
• b = a • 2 − 2 • b
db da
(i) a• −
dt  dt dt  dt dt
d  d2 b d 2 a
× b = a × 2 − 2 × b
db da
(ii) a× −
dt  dt dt  dt dt
11. If r is a unit vector, then prove that
r × dr  = dr ⋅
 dt  dt (Purvanchal 2006)
V-50

12. If r is a vector function of a scalar t, r its module, and a , b are constant vectors,
differentiate the following with respect to t :
dr
(i) r 3 r + a × , (ii) r 2 r + (a  r) b ,
dt
(iii) r n r , (iv) (ar + r b)2 .
13. Find the unit tangent vector to any point on the curve
x = a cos t , y = a sin t , z = bt .
14. If the direction of a differentiable vector function r (t) is constant, show that
 d r
r×  = 0. Or
 dt 
If r (t) is a vector of constant direction, show that its derivative is collinear
with it.
15. If e is the unit vector making an angle θ with x-axis, show that de / dθ is a unit
vector obtained by rotating e through a right angle in the direction of θ
increasing.

A nswers 1
2. (i) i + (2t + 1) j + (3t 2 + 2t + 1) k ; 2 j + (6t + 2) k
(ii) − j + 2k ; 2i; √ 5 ; 2
4. (i) V = 3 i + 4 j + 48 k , a = 2 j + 48 k
5. (i) r (ii) 6t 2 − 10 t − 2 ; − 6
7. (i) √ 37; √ (325)
9. (i) (5t 2 − 1) cos t + 11t sin t
(ii) t 2 (t sin t − 3 cos t) i − t 2 (t cos t + 3 sin t) j
− (11t cos t − 5t 2 sin t + sin t) k
3 5
(iii) 100 t + 2 t + 6t
dr dr d2 r
12. (i) 3r 2 r+r3 +a ×
dt dt dt 2
dr dr  d r
(ii) 2 r r +r2 + a •  b
dt dt  dt 
dr
(iii)  nr n−1 dr 
 r+r n
 dt  dt
 d r dr 
(iv) 2 (a r + r b) •  a + b
 dt dt 
1
13. (− a sin t i + a cos t j + b k)
√ (a + b 2 )
2
V-51

O bjective T ype Q uestions

Multiple Choice Questions


Indicate the correct answer for each question by writing the corresponding letter from
(a), (b), (c) and (d).
d2 r
1. If r = a e ωt + b e − ωt , where a, b are constant vectors then 2 − ω 2 r is
dt
equal to
(a) 1 (b) 0
(c) 2 (d) none of these
(Bundelkhand 2001; Agra 06)
−t −t −t
2. A particle moves along the curve r = e cos t i + e sin t j + e k . The
magnitude of its velocity at t = 0 is
(a) 2 3 (b) 3 /2
(c) 3 (d) none of these
2
3. d r
If r = x i + y j + z k , then = …… .
dt 2
dx dy dz
(a) i x + j y + k z (b) i + j +k
dt dt dt
d2 x d2 y d2 z
(c) i + j +k (d) none of these
dt 2 dt 2 dt 2 (Kumaun 2011)

Fill in the Blank(s)


Fill in the blanks “……”, so that the following statements are complete and correct.
dr d2 r
1. If r = 3i − 6t 2 j + 4t k , then = …… ; = …… .
dt dt 2 (Bundelkhand 2008)
d
2. If u = t 2 i − t j + (2t + 1) k , v = (2t − 3) i + j − t k , then (u • v) = …… .
dt
d2 r
3. If r = (cos ωt) i + (sin ωt) j , then r × 2 = …… .
dt
4. The necessary and sufficient condition for the vector a (t) to have constant
direction is ……
dr d2 r
5. If r = 5i + 3t 2 j + 2t k , then = …… ; = …… . (Bundelkhand 2010)
dt dt 2
d
6. (a • b) = …… .
dt (Kumaun 2009)
V-52

True or False
Write ‘T’ for true and ‘F’ for false statement.
1. A vector is said to be constant only if its magnitude is fixed and direction
changes.
2. The necessary and sufficient condition for the vector a (t) to have constant
da
magnitude is a • = 0.
dt

A nswers

Multiple Choice Questions


1. (b) 2. (c) 3. (c)

Fill in the Blank(s)


1. − 12 t j + 4 k ; − 12 j 2. 6t 2 − 10 t − 2 3. 0
da
4. a × =0 5. 6 t j + 2 k ;6 j
dt
db da
6. a • + •b
dt dt

True or False
1. F 2. T

¨
V-53

3
G radient, D ivergence
and C url

1 Partial Derivatives of Vectors


uppose r is a vector depending on more than one scalar variable. Let
S r = f ( x, y, z ) i. e., let r be a function of three scalar variables x , y and z . The
partial derivative of r with respect to x is defined as
∂r f ( x + δx, y, z ) − f ( x, y, z )
= lim
∂x δx → 0 δx
if this limit exists. Thus ∂r / ∂x is nothing but the ordinary derivative of r with
respect to x provided the other variables y and z are regarded as constants.
∂r ∂r
Similarly we may define the partial derivatives and ⋅
∂y ∂z
Higher partial derivatives can also be defined as in Scalar Calculus. Thus, for
example,
∂2 r ∂  ∂r  ∂2 r ∂  ∂r  ∂2 r ∂  ∂r 
=   , =   , =  ,
∂x 2 ∂ x  ∂ x  ∂y 2 ∂ y ∂
  y ∂z 2 ∂ z  ∂z 
∂2 r ∂  ∂r  ∂2 r ∂  ∂r 
=  , =  ⋅
∂x ∂y ∂x  ∂y  ∂y ∂x ∂y  ∂x 
V-54

If r has continuous partial derivatives of the second order at least, then,


∂2 r ∂2 r
= i. e., the order of differentiation is immaterial. If r = f ( x, y, z ), the
∂x ∂y ∂y ∂x
∂r ∂r ∂r
total differential dr of r is given by dr = dx + dy + dz .
∂x ∂y ∂z

2 The Vector Differential Operator Del (∇ )


The vector differential operator ∇ (read as del or nabla) is defined as
∂ ∂ ∂ ∂ ∂ ∂
∇≡ i+ j+ k ≡i + j +k
∂x ∂y ∂z ∂x ∂y ∂z
and operates distributively.
The vector operator ∇ can generally be treated to behave as an ordinary vector. It
∂ ∂ ∂
possesses properties like ordinary vectors. The symbols , , can be treated
∂x ∂y ∂z
as its components along i , j , k .

3 Gradient of a Scalar Field


(Agra 2005)
Definition: Let f ( x, y, z ) be defined and differentiable at each point ( x, y, z ) in a
certain region of space (i.e., defines a differentiable scalar field). Then the gradient of f, written
as ∇f or grad f, is defined as
∂ ∂ ∂  ∂f ∂f ∂f
∇f =  i + j+ k f = i+ j+ k.
 ∂x ∂y ∂z  ∂x ∂y ∂z
∂f ∂f
It should be noted that ∇f is a vector whose three successive components are ,
∂x ∂y
∂f
and ⋅ Thus the gradient of a scalar field defines a vector field. If f is a scalar point
∂z
function, then ∇f is a vector point function.

4 Formulas Involving Gradient


Theorem 1: Gradient of the sum of two scalar point functions. If f and g are
two scalar point functions, then
grad ( f + g) = grad f + grad g or ∇ ( f + g) = ∇f + ∇g .
Proof: We have
 ∂ ∂ ∂
grad ( f + g) = ∇ ( f + g) =  i + j +k  ( f + g)
 ∂x ∂y ∂z 
V-55

∂ ∂ ∂
=i ( f + g) + j ( f + g) + k ( f + g)
∂x ∂y ∂z
∂f ∂g ∂f ∂g ∂f ∂g
=i +i + j + j +k +k
∂x ∂x ∂y ∂y ∂z ∂z
 ∂f ∂f ∂f   ∂g ∂g ∂g 
= i + j +k  + i + j +k 
 ∂x ∂y ∂z   ∂x ∂y ∂z 
 ∂ ∂ ∂  ∂ ∂ ∂
= i + j +k  f + i + j +k  g
 ∂x ∂y ∂z   ∂x ∂y ∂z 
= ∇f + ∇g = grad f + grad g .
Similarly, we can prove that ∇ ( f − g) = ∇f − ∇g.
Theorem 2: Gradient of a constant. The necessary and sufficient condition for a
scalar point function to be constant is that ∇f = 0.
∂f ∂f ∂f
Proof: If f ( x, y, z ) is constant, then = 0, = 0, = 0.
∂x ∂y ∂z
∂f ∂f ∂f
Therefore, grad f = i + j +k = 0 i + 0 j + 0 k = 0.
∂x ∂y ∂z
Hence the condition is necessary.
∂f ∂f ∂f
Conversely, let grad f = 0. Then i + j +k = 0.
∂x ∂y ∂z
∂f ∂f ∂f
Therefore, = 0, = 0, = 0.
∂x ∂y ∂z
∴ f must be independent of x, y and z .
∴ f must be a constant. Hence the condition is sufficient.

Theorem 3: Gradient of the product of two scalar point functions.


If f and g are scalar point functions, then grad ( fg) = f grad g + g grad f
or ∇ ( fg) = f ∇g + g ∇f . (Kumaun 2014)
 ∂ ∂ ∂
Proof: We have ∇ ( fg) =  i + j +k  ( fg)
 ∂x ∂y ∂z 
∂ ∂ ∂
=i ( fg) + j ( fg) + k ( fg)
∂x ∂y ∂z
 ∂g ∂f   ∂g ∂f   ∂g ∂f 
=i f + g  + jf + g  +k f + g 
 ∂x ∂x   ∂ y ∂ y   ∂ z ∂z 
 ∂g ∂g ∂g   ∂f ∂f ∂f 
= f i + j +k  + g i + j +k 
 ∂x ∂y ∂z   ∂x ∂y ∂z 

= f ∇g + g ∇f = f grad g + g grad f .
In particular, if c is a constant, then
∇ (c f ) = c ∇f + f ∇c = c ∇f + 0 = c ∇f .
V-56

Theorem 4: Gradient of the quotient of two scalar functions. If f and g are two
f  g ∇f − f ∇g
scalar point functions, then ∇   = ⋅
 g g2 (Kumaun 2013)
Proof: We have
f   ∂ ∂ ∂ f 
∇   = i + j +k   
 g   ∂x ∂y ∂z   g 

∂ f  ∂ f  ∂ f 
=i   + j   +k  ⋅
∂x  g  ∂y  g  ∂z  g 

∂f ∂g ∂f ∂g
g − f g − f
∂  f ∂x ∂x , ∂ f  ∂y ∂y
But   =   = ,
∂x  g  g 2 ∂y  g  g 2

∂f ∂g
g − f
∂ f  ∂z ∂z ⋅
and   =
∂z  g  g2
f  1   ∂f ∂g   ∂f ∂g   ∂f ∂g  
∴ ∇   = 2  i g − f  + jg − f  + k g − f 
 g g   ∂x ∂x   ∂y ∂y   ∂z ∂z  

1   ∂f ∂f ∂f   ∂g ∂g ∂g 
= 2 
g i + j +k  − f i + j +k 
g   ∂x ∂y ∂z   ∂x ∂y ∂z 
1
= 2 { g ∇f − f ∇g }.
g

Example 1: If A = x 2 yz i − 2 xz 3 j + xz 2 k , B = 2zi + yj − x 2 k , find the value of


∂2
(A × B) at (1 , 0, − 2).
∂x ∂y
 i j k 
Solution: We have A × B =  x 2 yz −2 xz 3
xz 2 

 

2z y − x2 

= (2 x 3 z 3 − xyz 2 ) i + (2 xz 3 + x 4 yz ) j
+ ( x 2 y 2 z + 4 xz 4 ) k .

∴ (A × B) = − xz 2 i + x 4 z j + 2 x 2 yzk .
∂y
∂2 ∂ ∂ 
Again (A × B) =  (A × B) = − z 2 i + 4 x 3 zj + 4 xyz k . …(1)
∂x ∂y ∂x  ∂y 
V-57

Putting x = 1, y = 0 and z = − 2 in (1), we get the required derivative at the point


(1, 0, − 2) = − 4i − 8 j .

Example 2: If f ( x , y , z ) = 3 x 2 y − y 3 z 2 , find grad f at the point (1, − 2 , − 1).


(Rohilkhand 2009)
Solution: We have
 ∂ ∂ ∂ 2 3 2
grad f = ∇f =  i + j +k  (3 x y − y z )
 ∂x ∂y ∂z 
∂ ∂
=i (3 x 2 y − y 3 z 2 ) + j (3 x 2 y − y 3 z 2 )
∂x ∂y

+k (3 x 2 y − y 3 z 2 )
∂z
= i (6 xy) + j (3 x 2 − 3 y 2 z 2 ) + k (−2 y 3 z )
= 6 x y i + (3 x 2 − 3 y 2 z 2 ) j − 2 y 3 z k .
Putting x = 1, y = − 2 , z = − 1, we get
∇f = 6 (1) (−2) i + {3 (1)2 − 3 (−2)2 (−1)2 } j − 2 (−2)3 (−1) k
= − 12 i − 9 j − 16k .

Example 3: If r = | r | where r = xi + yj + zk , prove that


(i) ∇ f (r) = f ′ (r) ∇r , (Rohilkhand 2008; Meerut 11)
1
(ii) ∇r = r ,
r
(iii) ∇ f (r) × r = 0,
∇   = − 3 ,
1 r
(iv) (Rohilkhand 2008, 09B)
 r r
r
(v) ∇ log | r | = 2 ,
r (Garhwal 2000; Kumaun 07; Rohilkhand 08;
Bundelkhand 08; Meerut 11)
n−2
(vi) ∇r n
= nr r. (Rohilkhand 2008, 11; Bundelkhand 11)
Solution: If r = x i + y j + z k , then r = | r | = √ ( x 2 + y 2 + z 2 ).
∴ r 2 = x 2 + y 2 + z 2.
 ∂ ∂ ∂ ∂ ∂ ∂
(i) ∇f (r) =  i + j +k  f (r) = i f (r) + j f (r) + k f (r)
 ∂x ∂y ∂z  ∂x ∂y ∂z
∂r ∂r ∂r  ∂r ∂r ∂r 
= i f ′ (r) + j f ′ (r) + k f ′ (r) = f ′ (r)  i + j +k 
∂x ∂y ∂z  ∂x ∂y ∂z 

= f ′ (r) ∇r .
∂r ∂r ∂r
(ii) We have ∇r = i + j +k ⋅
∂x ∂y ∂z
V-58

Now r 2 = x2 + y2 + z 2 .
∂r ∂r x
∴ 2r = 2 x i. e., = ⋅
∂x ∂x r
∂r y ∂r z
Similarly, = and = ⋅
∂y r ∂z r
x y z 1 1 ∧
∴ ∇r = i+ j + k = ( xi + yj + zk ) = r = r .
r r r r r

(iii) We have as in part (i), ∇f (r) = f ′ (r) ∇r.


1
But as in part (ii), ∇r = r .
r
1
∴ ∇f (r) = f ′ (r) r .
r
∇f (r) × r =  f ′ (r) r × r =  f ′ (r) (r × r) = 0 .
1 1

 r  r 
[ ∵ r × r = 0]
∂  1 + j ∂  1 + k ∂
(iv) We have ∇   = i  1
1
 r ∂x  r ∂y  r ∂z  r

 1 ∂r   1 ∂r   1 ∂r 
= i − 2  + j − 2  + k − 2 
 r ∂x   r ∂ y   r ∂z 

1  ∂r ∂r ∂r 
=−  i+ j+ k
r  ∂x
2 ∂y ∂z 

1 x y z  1 1
=−  i + j + k  = − 3 ( xi + yj + zk ) = − 3 r .
r r
2 r r  r r
[see part (ii)]
(v) We have ∇ log |r | = ∇ log r
∂ ∂ ∂
=i log r + j log r + k log r
∂x ∂y ∂z
1 ∂r 1 ∂r 1 ∂r 1 x y z 
= i+ j+ k =  i+ j + k
r ∂x r ∂y r ∂z r r r r 
1 1
= 2
( xi + yj + zk ) = r.
r r2
∂ n ∂ n ∂ n
(vi) We have ∇r n = i r + j r +k r
∂x ∂y ∂z
∂r ∂r ∂r
= i nr n − 1 + j nr n − 1 + k nr n − 1
∂x ∂y ∂z
 ∂r ∂r ∂r 
= nr n − 1  i + j +k  = nr
n −1
∇r
 ∂ x ∂ y ∂ z 
V-59

= nr n − 1
1
r ∵ ∇r = r as in part (ii)
r  r 
= nr n − 2 r .
Example 4: (i) Interpret the symbol a • ∇ (ii) Show that (a • ∇) φ = a • ∇ φ
(iii) Show that (a • ∇) r = a . (Kumaun 2008; Purvanchal 14)
Solution: (i) Let a = a1 i + a2 j + a3 k . Then
 ∂ ∂ ∂
a • ∇ = (a1 i + a2 j + a3 k ) •  i + j +k 
 ∂x ∂y ∂z 
∂ ∂ ∂
= a1 + a2 + a3 ⋅
∂x ∂y ∂z
 ∂ ∂ ∂
(ii) (a • ∇) φ =  a1 + a2 + a3  φ.
 ∂x ∂y ∂z 
 ∂φ ∂φ ∂φ 
Also a • ∇φ = (a1 i + a2 j + a3 k ) •  i+ j+ k
 ∂x ∂y ∂z 
∂φ ∂φ ∂φ
= a1 + a2 + a3 ⋅
∂x ∂y ∂z
Hence (a • ∇) φ = a • ∇φ.
 ∂ ∂ ∂ ∂r ∂r ∂r
(iii) (a • ∇) r =  a1 + a2 + a3  r = a1 + a2 + a3 ⋅
 ∂x ∂y ∂z  ∂x ∂y ∂z
But r = xi + yj + zk .
∂r ∂r ∂r
∴ = i, = j, = k.
∂x ∂y ∂z
∴ (a • ∇) r = a1 i + a2 j + a3 k = a .

Comprehensive Exercise 1

1. If F = e x y
i + ( x − 2 y) j + x sin y k , calculate
∂F ∂F ∂2 F
(i) , (ii) , (iii) ,
∂x ∂y ∂x 2
∂2 F ∂2 F
(iv) , (v) ⋅
∂x ∂y ∂y 2
2. If f = (2 x 2 y − x 4 ) i + (e xy − y sin x) j + x 2 cos y k , verify that
∂2 f ∂2 f
= ⋅
∂y ∂x ∂x ∂y
V-60

2
3. If u = x y z i + x z j − y 3 k and v = x 3 i − x y z j + x 2 z k , calculate
∂2 u ∂2 v
× at the point (1, 1, 0).
∂y 2 ∂x 2
4. If φ ( x , y , z ) = x 2 y + y 2 x + z 2 , find ∇φ at the point (1, 1, 1).
5. Find grad f , where f is given by f = x 3 − y 3 + xz 2 , at the point (1, − 1, 2).
6. If φ ( x, y, z ) = xy 2 z and f = xzi − xyj + yz 2 k , show that
∂3
(φf ) at (2 , − 1, 1) is 4i + 2 j .
∂x 2 ∂z
7. If u = x + y + z , v = x 2 + y 2 + z 2 , w = yz + zx + xy, prove that
(grad u) • [(grad v) × (grad w)] = 0. (Meerut 2007B)
 ∂f ∂f   ∂f ∂f   ∂f ∂f 
8. If F =  y −z  i + z − x  j+ x − y  k , prove that
 ∂ z ∂ y   ∂ x ∂z   ∂ y ∂x 
(i) F = r × ∇f , (ii) F • r = 0, (iii) F • ∇f = 0 .
−1 /3 −3 /2 −7 /3
9. If φ = (3r 2 − 4r 1 /2 + 6r ), show that ∇φ = 2 (3 − r −r )r.
10. Prove that ∇φ • d r = dφ . (Meerut 2005, 06, 09B; Kumaun 08)
11. Prove that f (u) ∇u = ∇ ∫ f (u) du .
(Kumaun 2012, 13)
12. ρ and p are two scalar point functions such that ρ is a function of p; show

that ∇ρ = ∇p .
dp
dφ dr
13. Show that = ∇φ • , where r = x i + y j + z k and φ is a function of
ds ds
x , y and z .
A•r
14. Prove that A • ∇  = − 3 ⋅
1
 r r (Meerut 2010)
−3 −5
15. Prove that ∇r = − 3r r. (Meerut 2009, 12)
16. Show that
(i) grad (r • a) = a , (Avadh 2010)
(ii) grad [r , a , b] = a × b ,
where a and b are constant vectors.

A nswers 1
x y
1. (i) ye i + j + sin y k (ii) xe x y
i − 2 j + x cos y k
(iii) y 2 e x y
i (iv) e x y
( x y + 1) i + cos y k
V-61

(v) x2 e x y
i − x sin y k
3. − 36 j 4. 3i + 3 j + 2 k
5. 7i − 3 j + 4k

5 Equipotential Surfaces or Level Surfaces


Let f ( x , y , z ) be a scalar field over a region R .The points satisfying an equation of
the type
f ( x , y , z ) = c , (arbitrary constant )
constitute a family of surfaces in three dimensional space. The surfaces of this
family are called level surfaces. Any surface of this family is such that the value of
the function f at any point of it is the same. Therefore these surfaces are also called
iso-f-surfaces.

Theorem 1: Let f ( x , y , z ) be a scalar field over a region R . Then through any point of
R there passes one and only one level surface.
Proof: Let ( x1 , y1 , z1 ) be any point of the region R .
Then the level surface f ( x, y, z ) = f ( x1 , y1 , z1 ) passes through this point.
Now suppose the level surfaces f ( x , y , z ) = c1 and f ( x , y , z ) = c 2 pass through
the point ( x1 , y1 , z1 ). Then
f ( x1 , y1 , z1 ) = c1 and f ( x1 , y1 , z1 ) = c 2 .
Since f ( x, y, z ) has a unique value at ( x1 , y1 , z1 ), therefore we have c1 = c 2 .
Hence only one level surface passes through the point ( x1 , y1 , z1 ).
Theorem 2: ∇f is a vector normal to the surface f ( x , y , z ) = c where c is a constant.

Proof: Let r = xi + yj + zk be the position vector of any point P ( x , y , z ) on the


level surface f ( x, y, z ) = c . Let Q ( x + δx , y + δy , z + δz ) be a neighbouring point
on this surface.
Then the position vector of Q = r + δr = ( x + δx) i + ( y + δy) j + (z + δz ) k .

∴ PQ = (r + δr) − r = δr = δx i + δy j + δz k .
As Q → P , the line PQ tends to tangent at P to the level surface.
Therefore d r = dx i + dy j + dz k lies in the tangent plane to the surface at P .
From the differential calculus, we have
∂f ∂f ∂f
df = dx + dy + dz
∂x ∂y ∂z
 ∂f ∂f ∂f 
= i + j +k  • (dx i + dy j + dz k ) = ∇f • d r.
 ∂x ∂y ∂z 
V-62

Since f ( x , y , z ) = constant, therefore df = 0.


∴ ∇f • dr = 0 so that ∇f is a vector perpendicular to d r and therefore to the
tangent plane at P to the surface f ( x, y, z ) = c .
Hence ∇ f is a vector normal to the surface f ( x, y, z ) = c .
Thus if f ( x, y, z ) is a scalar field defined over a region R , then ∇f at any point
( x , y , z ) is a vector in the direction of normal at that point to the level surface
f ( x, y, z ) = c passing through that point.

6 Directional Derivative of a Scalar Point Function


Definition: Let f ( x, y, z ) define a scalar field in a region R and let P be any point in
this region. Suppose Q is a point in this region in the neighbourhood of P in the direction of a

given unit vector a .
lim f (Q) − f ( P )
Then Q → P , if it exists, is called the directional derivative of f at P in the
PQ

direction of a .

Interpretation of Directional Derivative: Let P be the point ( x , y , z ) and


let Q be the point ( x + δx, y + δy, z + δz ). Suppose PQ = δs . Then δs is a small

element at P in the direction of a. If
δf = f ( x + δx , y + δy , z + δz ) − f ( x , y , z ) = f (Q) − f ( P ),
δf
then represents the average rate of change of f per unit distance in the direction
δs
∧ ∧
of a . Now the directional derivative of f at P in the direction of a is
lim f (Q) − f ( P ) = lim δf = df ⋅ It represents the rate of change of f with
Q→ P
PQ δ s → 0 δs ds

respect to distance s at the point P in the direction of unit vector a .
Theorem 1: The directional derivative of a scalar field f at a point P ( x, y, z ) in the
∧ df ∧
direction of a unit vector a is given by = ∇ f • a.
ds
Proof: Let f ( x, y, z ) define a scalar field in the region R . Let r = xi + yj + zk
denote the position vector of any point P ( x, y, z ) in this region. If s denotes the

distance of P from some fixed point A in the direction of a, then δs denotes small
∧ dr
element at P in the direction of a . Therefore is a unit vector at P in this
ds
dr ∧
direction i. e., = a.
ds
But r = x i + y j+ z k.
V-63

dr dx dy dz ∧
∴ = i+ j+ k = a.
ds ds ds ds
∧  ∂f ∂f ∂f   dx dy dz 
Now ∇f • a =  i + j +k  • i + j+ k 
 ∂x ∂y ∂z   ds ds ds 
∂f dx ∂f dy ∂f dz d f
= + + =
dx ds dy ds dz ds ds

= directional derivative of f at P in the direction of a .

Alternative Proof: Let Q be a point in the neighbourhood of P in the direction of



the given unit vector a . If l, m, n are the direction cosines of the line PQ , then

l i + m j + n k = the unit vector in the direction of PQ = a . Further if PQ = δs, then
the co-ordinates of Q are ( x + lδs, y + mδs, z + nδs). Now the directional derivative

of f at P in the direction of a is
f (Q) − f ( P )
= lim
Q→ P PQ
f ( x + lδs , y + mδs , z + nδs) − f ( x , y , z )
= lim
δs → 0 δs

 ∂f ∂f ∂f 
f ( x , y , z ) +  lδs + mδs + nδs  +… − f ( x , y , z )
 ∂x ∂y ∂z 
= lim ,
δs → 0 δs

on expanding by Taylor’s theorem


∂f ∂f ∂f
=l +m +n
∂x ∂y ∂z
 ∂f ∂f ∂f  ∧
= i+ j+ k  • (l i + m j + n k ) = ∇f • a .
 ∂x ∂y ∂z 

Theorem 2: If n be a unit vector normal to the level surface f ( x , y , z ) = c at a point

P ( x , y , z ) and n be the distance of P from some fixed point A in the direction of n so that δn
∧ df ∧
represents element of normal at P in the direction of n , then grad f = n.
dn
∂f ∂f ∂f
Proof: We have grad f = ∇f = i+ j+ k.
∂x ∂y ∂z

Also grad f is a vector normal to the surface f ( x , y , z ) = c . Since n is a unit vector

normal to the surface f ( x , y , z ) = c , therefore let grad f = A n, where A is some
scalar to be determined.
V-64

df ∧
Now = directional derivative of f in the direction of n
dn
∧ ∧ ∧ ∧
= ∇f • n = A n • n = A. [∵ ∇ f = grad f = A n ]
df ∧
∴ grad f = ∇ f = n.
dn
∧ df
Note: If the vector n is in the direction of f increasing, then is positive.
dn
Therefore ∇f is a vector normal to the surface f ( x , y , z ) = c in the direction of f
increasing.
Theorem 3: Grad f is a vector in the direction of which the maximum value of the
df
directional derivative of f i.e., occurs.
ds

Proof: The directional derivative of f in the direction of a is given by
df ∧  df ∧  ∧  df ∧ 
=∇ f •a=  n  •a ∵ ∇f = dn n 
ds  dn 
df ∧ ∧ df ∧ ∧
= (n • a ) = cos θ, where θ is the angle between a and n .
dn dn
df df
Now is fixed. Therefore cos θ is maximum when cos θ is maximum i. e., when
dn dn
∧ ∧ ∧
cos θ = 1 . But cos θ will be 1 when the angle between a and n is 0 i. e., when a is along

the unit normal vector n .
Therefore the directional derivative is maximum along the normal to the surface.
df
Its maximum value is = = |grad f |.
dn

7 Tangent Plane and Normal to a Level Surface


Tangent plane to a surface: The tangent to any curve drawn on a surface is
called a tangent line to the surface. All the tangent lines to a surface at the point P lie
in a plane. This plane is called the tangent plane to the surface at the point P on it.
Normal to a surface: The normal to a surface at the point P is a straight line
passing through P and perpendicular to the tangent plane at P.
Angle between two surfaces: The angle between the two surfaces at a point P is
the angle between the normals to the two surfaces at that point.
Equations of the Tangent Plane and Normal to a Surface:
To find the equations of the tangent plane and normal to the surface
f (x , y , z) = c .
Let f ( x , y , z ) = c be the equation of a level surface. Let r = x i + y j + z k be the
position vector of any point P ( x , y , z ) on this surface.
V-65

∂f ∂f ∂f
Then ∇f = i+ k is a vector along the normal to the surface at P i. e.,∇f
j+
∂x ∂y ∂z
is perpendicular to the tangent plane at P .
Tangent plane at P: Let R = X i + Y j + Z k be the position vector of any
current point Q ( X , Y , Z ) on the tangent plane at P to the surface. The vector

PQ = R − r = ( X − x) i + (Y − y) j + (Z − z ) k
lies in the tangent plane at P . Therefore it is perpendicular to the vector ∇f .
∴ (R − r) • ∇f = 0
 ∂f ∂f ∂f 
or [( X − x) i + (Y − y) j + (Z − z ) k ] •  i+ j+ k = 0
 ∂x ∂y ∂z 
∂f ∂f ∂f
or ( X − x) + (Y − y) + (Z − z ) = 0, …(1)
∂x ∂y ∂z
is the equation of the tangent plane at P.
Normal at P: Let R = X i + Y j + Z k
be the position vector of any current point Q ( X , Y , Z ) on the normal at P to the

surface. The vector PQ = R − r = ( X − x) i + (Y − y) j + (Z − z ) k lies along the
normal at P to the surface. Therefore it is parallel to the vector ∇f .
∴ (R − r) × ∇f = 0 …(2)
is the vector equation of the normal at P to the given surface.
Cartesian form: The vectors
∂f ∂f ∂f
( X − x) i + (Y − y) j + (Z − z ) k and ∇ f = i+ j+ k,
∂x ∂y ∂z
will be parallel if
 ∂f ∂f ∂f 
( X − x) i + (Y − y) j + (Z − z ) k = p  i+ j+ k ,
 ∂x ∂y ∂z 
where p is some scalar.
Equating the coefficients of i , j, k , we get
∂f ∂f ∂f
X − x= p ,Y − y = p ,Z − z = p
∂x ∂y ∂z
X − x Y − y Z −z
or = = …(3)
∂f ∂f ∂f
∂x ∂y ∂z
which are the equations of the normal at P.
 ∂F ∂F ∂F 
Note: The vector  , ,  is along the normal to the surface
 ∂x ∂y ∂z 
F ( x, y, z ) = 0 at the point ( x, y, z ).
V-66

Example 5: Find a unit normal vector to the level surface x 2 y + 2 xz = 4 at the point
(2 , − 2 , 3). (Kashi 2014)
2
Solution: The equation of the level surface is f ( x, y, z ) ≡ x y + 2 xz = 4.
The vector grad f is along the normal to the surface at the point ( x , y , z ).
We have grad f = ∇ ( x 2 y + 2 xz ) = (2 x y + 2z ) i + x 2 j + 2 x k .
∴ at the point (2 , − 2 , 3), grad f = − 2 i + 4 j + 4k .
∴ −2 i + 4 j + 4 k is a vector along the normal to the given surface at the point
(2 , − 2 , 3).
Hence a unit normal vector to the surface at this point
−2 i + 4 j + 4 k −2 i + 4 j + 4 k 1 2 2
= = = − i + j+ k.
|−2 i + 4 j + 4 k| √ (4 + 16 + 16) 3 3 3
 1 2 2  1 2 2
The vector −  − i + j + k  i. e., i − j − k is also a unit normal vector to
 3 3 3  3 3 3
the given surface at the point (2 , − 2 , 3).
Alternate Solution: The given surface is
φ ( x, y, z ) ≡ x 2 y + 2 xz − 4 = 0.
∂φ ∂φ ∂φ
We have = 2 x y + 2z , = x2 , = 2z .
∂x ∂y ∂z
∴ At the point (2, − 2, 3), we have
∂φ ∂φ ∂φ
= − 2, = 4, = 4.
∂x ∂y ∂z
∴ A vector along the normal to the given surface at the point (2, − 2, 3)
 ∂φ ∂φ ∂φ
= , ,  = (− 2, 4, 4) or (− 1, 2, 2).
 ∂x ∂y ∂z 

We have |(− 1, 2, 2)| = (− 1)2 + 22 + 22 = 3.


Hence, a unit normal vector to the given surface at the point (2, − 2, 3)
= (− 1, 2, 2) =  − , ,  ⋅
1 1 2 2
3  3 3 3

Example 6: Find the directional derivative of f ( x, y, z ) = x 2 yz + 4 xz 2 at the point


(1, − 2 , − 1) in the direction of the vector 2 i − j − 2 k .
(Bundelkhand 2007; Rohilkhand 11; Kashi 13; Agra 14)
Solution: We have f ( x, y, z ) = x 2 yz + 4 xz 2 .
∴ grad f = (2 xyz + 4z 2 ) i + x 2 zj + ( x 2 y + 8 xz ) k
= 8i − j − 10 k at the point (1, − 2 , − 1).
V-67


If a be the unit vector in the direction of the vector 2i − j − 2k , then
∧ 2i − j−2k 2 1 2
a= = i − j − k.
√ (4 + 1 + 4) 3 3 3
Therefore the required directional derivative is
df ∧  2 1 2 
= grad f • a = (8i − j − 10 k ) •  i − j − k 
ds  3 3 3 
16 1 20 37
= + + = ⋅
3 3 3 3
Since this is positive, f is increasing in this direction.
Example 7: Find the directional derivative of the function f = x 2 − y 2 + 2z 2 at the
point P (1, 2 , 3) in the direction of the line PQ where Q is the point (5, 0, 4).
(Purvanchal 2007; Kashi 14)
Solution: Here
∂f ∂f ∂f
grad f = i+ j+ k
∂x ∂y ∂z
= 2 xi − 2 yj + 4zk = 2i − 4 j + 12k at the point (1, 2 , 3).

Also PQ = position vector of Q − position vector of P
= (5i + 0 j + 4k ) − (i + 2 j + 3k ) = 4i − 2 j + k .
∧ →
If a be the unit vector in the direction of the vector PQ , then
∧ 4i −2 j+ k 4i −2 j+ k
a= = ⋅
√ (16 + 4 + 1) √ (21)
∴ the required directional derivative
∧  4i − 2 j + k 
= (grad f ) • a = (2i − 4 j + 12 k ) •  
 √ (21) 
28 28 4
= = √ (21) = √ (21).
√ (21) 21 3

Example 8: In what direction from the point (1, 1, − 1) is the directional derivative of
f = x 2 − 2 y 2 + 4z 2 a maximum ? Also find the value of this maximum directional
derivative. (Kanpur 2008)
Solution: We have
grad f = 2 x i − 4 y j + 8z k
= 2 i − 4 j − 8 k at the point (1, 1, − 1).
The directional derivative of f is maximum in the direction of grad f
=2i −4 j−8k.
The maximum value of this directional derivative
= |grad f | = |2 i − 4 j − 8 k | = √ (4 + 16 + 64) = √ (84) = 2 √ (21).
V-68

Example 9: What is the greatest rate of increase of u = xyz 2 at the point (1, 0 , 3) ?

Solution: We have ∇u = yz 2 i + xz 2 j + 2 xyz k .


∴ at the point (1, 0, 3), we have ∇u = 0 i + 9 j + 0 k = 9 j.
The greatest rate of increase of u at the point (1, 0, 3)
du
= the maximum value of at the point (1, 0, 3)
ds
= |∇u |, at the point (1, 0, 3) = | 9 j | = 9.

Example 10: Find the equations of the tangent plane and normal to the surface
2
2 xz − 3 xy − 4 x = 7 at the point (1, − 1, 2).

Solution: The equation of the surface is


f ( x, y, z ) ≡ 2 xz 2 − 3 xy − 4 x = 7.
We have
grad f = (2z 2 − 3 y − 4) i − 3 x j + 4 x z k
= 7i − 3 j + 8k , at the point (1, − 1, 2).
∴ 7 i − 3 j + 8 k is a vector along the normal to the surface at the point (1, − 1, 2).
The position vector of the point (1, − 1, 2) is = r = i – j + 2k .
If R = X i + Y j + Z k is the position vector of any current point ( X , Y , Z ) on the
tangent plane at (1, − 1, 2), then the vector R − r is perpendicular to the vector
grad f .
∴ the equation of the tangent plane is (R − r) • grad f = 0
i. e., {( X i + Y j + Z k ) − (i − j + 2k )} • (7i − 3 j + 8k ) = 0
i. e., {( X − 1) i + (Y + 1) j + (Z − 2) k } • (7i − 3 j + 8k ) = 0
i. e., 7 ( X − 1) − 3 (Y + 1) + 8 (Z − 2) = 0.
The equations of the normal to the surface at the point (1, − 1, 2) are
X −1 Y +1 Z − 2 X −1 Y +1 Z − 2
= = i. e., = = ⋅
∂f ∂f ∂f 7 −3 8
∂x ∂y ∂z
Example 11: Find the equations of the tangent plane and normal to the surface xyz = 4 at
the point (1, 2 , 2). (Meerut 2000)

Solution: The equation of the surface is f ( x , y , z ) ≡ x y z − 4 = 0.


We have grad f = yz i + x z j + x y k = 4i + 2 j + 2k , at the point (1, 2 , 2).
∴ 4i + 2 j + 2k is a vector along the normal to the surface at the point (1, 2 , 2).
The position vector of the point (1, 2 , 2) is = r = i + 2 j + 2k .
If R = X i + Y j + Z k is the position vector of any current point ( X , Y , Z ) on the
tangent plane at (1, 2 , 2), the equation of the tangent plane is
(R − r) • grad f = 0,
V-69

i. e., { ( X i + Y j + Z k ) − (i + 2 j + 2k )} • (4i + 2 j + 2k ) = 0
i. e., { ( X − 1) i + (Y − 2) j + (Z − 2) k} • (4i + 2 j + 2k ) = 0
i. e., 4 ( X − 1) + 2 (Y − 2) + 2 (Z − 2) = 0
i. e., 4 X + 2Y + 2 Z = 12 , i. e., 2 X + Y + Z = 6 .
The equations of the normal to the surface at the point (1, 2 , 2) are
X −1 Y − 2 Z − 2
= =
∂f ∂f ∂f
∂x ∂y ∂z
X −1 Y −2 Z −2 X −1 Y −2 Z −2
i. e., = = , i. e., = = ⋅
4 2 2 2 1 1
Example 12: Find the angle between the surfaces x 2 + y 2 + z 2 = 9, and
2 2
z = x + y − 3 at the point (2 , − 1, 2). (Meerut 2001)
Solution: Angle between two surfaces at a point is the angle between the normals
to the surfaces at that point. Let f1 = x 2 + y 2 + z 2 and f 2 = x 2 + y 2 − z .
Then grad f1 = 2 x i + 2 y j + 2 z k and grad f 2 = 2 x i + 2 y j − k .
Let n1 = grad f1 at the point (2 , − 1, 2) and n 2 = grad f 2 at the point (2 , − 1, 2).
Then n1 = 4i − 2 j + 4k and n 2 = 4i − 2 j − k .
The vectors n1 and n 2 are along normals to the two surfaces at the point (2 , − 1, 2).
If θ is the angle between these vectors, then
n1 • n 2 = |n1||n 2| cos θ
or 16 + 4 − 4 = √ (16 + 4 + 16) √ (16 + 4 + 1) cos θ.
16 8
∴ cos θ = or θ = cos −1 ⋅
6 √ (21) 3 √ (21)

Comprehensive Exercise 2

1. (i) Find the gradient and the unit normal to the level surface
x 2 + y − z = 4 at the point (2 , 0, 0).
(ii) Find the unit normal to the surface z = x 2 + y 2 at the point
(−1, − 2 , 5).
2. (i) Find the unit vector normal to the surface x 2 − y 2 + z = 2 at the point
(1, − 1, 2).
(ii) Find the unit normal to the surface x 4 − 3 xyz + z 2 + 1 = 0 at the point
(1, 1, 1).
(iii) Find a unit normal vector to the surface x 2 y + 2 xz = 4 at the point
(2 , − 2 , 3).
V-70

3. (i) Find the directional derivatives of a scalar point function f in the


direction of coordinate axes.
(ii) Find the directional derivative of φ = xy + yz + zx in the direction of
the vector i + 2 j + 2k at (1, 2 , 0). (Kumaun 2015)
(iii) Find the directional derivative of φ ( x , y , z ) = x 2 yz + 4 xz 2 at the
point (1, − 2 , 1) in the direction of 2 i − j − 2k .
(iv) Find the directional derivative of f ( x , y , z ) = x 2 − 2 y 2 + 4z 2 at the
point (1, 1, − 1) in the direction of 2 i + j − k .
(v) Find the directional derivative of the function f = xy + yz + zx in the
direction of the vector 2i + 3 j + 6k at the point (3 , 1, 2).
(vi) Find the directional derivative of the function xy 2 + yz 2 + zx 2 along
the tangent to the curve x = t , y = t 2 , z = t 3 at the point (1, 1, 1.
)
4. Find the directional derivatives of φ = x y z at the point (2 , 2 , 2), in the
directions (i) i , (ii) j , (iii) i + j + k .
5. For the function f = y / ( x 2 + y 2 ), find the value of the directional
derivative making an angle 30° with the positive x-axis at the point (0, 1).
6. (i) Find the greatest value of the directional derivative of the function
2 x 2 − y − z 4 at the point (2 , − 1, 1).
(ii) In what direction the directional derivative of φ = x 2 y 2 z from (1, 1, 2)
will be maximum and what is its magnitude ? Also find a unit normal
vector to the surface x 2 y 2 z = 2 at the point (1, 1, 2).
(iii) Find the maximum value of the directional derivative of φ = x 2 yz at
the point (1, 4 , 1).
(iv) Calculate the maximum rate of change and the corresponding
direction for the function φ = x 2 y 3 z 4 at the point 2i + 3 j − k .
(v) Find the values of the constants a, b, c so that the directional derivative
of φ = ax 2 + by 2 + c z 2 at (1, 1, 2) has a maximum magnitude 4 in
the direction parallel to y-axis.
7. Find the equation of the tangent plane to the surface yz − zx + x y + 5 = 0,
at the point (1, − 1, 2).
8. (i) Find the equations of the tangent plane and normal to the surface
x 2 + y 2 + z 2 = 25 at the point (4, 0, 3).
(ii) Given the curve x 2 + y 2 + z 2
= 1, x + y + z = 1 (intersection of two
surfaces), find the equations of the tangent line at the point (1, 0, 0).
(iii) Find the equations of the tangent plane and normal to the surface
z = x 2 + y 2 at the point (2 , − 1, 5).
V-71

9. (i) Find the equations of the tangent plane and the normal to the surface
x 2 + 2 y 2 + 3z 2 = 12 at the point (1, 2 , − 1).
(ii) Find the equations of the tangent plane and the normal to the surface
xy + yz + z x = 1, at the point (2, 3, − 1).
(iii) Find the equations of the tangent plane and the normal to the surface
z = x 2 − 2 xy − y 2 at the point (1, 2 , − 7).
(iv) Find the equation of the tangent plane to the surface x 2 + y 2 + z 2 = 9
at (2 , − 1, 2).
(v) Find the equation of the tangent plane to the surface z = x 2 + y 2 at
the point (1, − 1, 2).
10. Show that the directional derivative of a scalar point function at any point
along any tangent line to the level surface at the point is zero.
11. If F and f are point functions, show that the components of the former,
tangential and normal to the level surface f = 0 are
∇f × (F × ∇f ) (F • ∇f ) ∇f
2
and ⋅
(∇f ) (∇f )2

12. Find the angle of intersection at (4, − 3, 2) of spheres x 2 + y 2 + z 2


= 29 and
2 2 2
x + y +z + 4 x − 6 y − 8z − 47 = 0.
13. Find the constants a and b so that the surface ax 2 − byz = (a + 2) x will be
orthogonal to the surface 4 x 2 y + z 3 = 4 at the point (1, − 1, 2).
14. Show that the sum of the squares of the intercepts on the coordinate axes
made by the tangent plane to the surface x 2 /3 + y 2 /3 + z 2 /3 = a2 /3 is
constant.

A nswers 2
1 − (2i + 4 j + k)
1. (i) 4i + j − k ; (4 i + j − k ) (ii)
3 √2 √ (21)
i − 3j − k
(iii)  − , , 
1 1 2 2
2. (i) (2 i + 2 j + k ) (ii)
3 √ (11)  3 3 3
∂f ∂f ∂f
3. (i) , and in the directions of i , j and k
∂x ∂y ∂z
10 13 8
(ii) (iii) − (iv)
3 3 6
45 18
(v) (vi)
7 √ (14)
V-72

4. (i) 4 (ii) 4 (iii) 4 3


5. −1 / 2
6. (i) 9
4i +4 j+ k
(ii) √ (33) in the direction of the vector 4 i + 4 j + k ;
√ (33)
(iii) 9
(iv) 324 √ 2 in the direction of the vector 108 (i + j − 4 k)
(v) a = 0, b = 2, c = 0
7. 3 X − 3Y + 2 Z = 10
x−4 y z −3
8. (i) 4 x + 3z = 25 ; = =
4 0 3
Y Z x−2 y +1 z −5
(ii) X = 1, = (iii) 4 x − 2 y − z = 5 ; = =
−1 1 4 −2 −1
x −1 y − 2 z +1
9. (i) x + 4 y − 3z = 12 ; = =
1 4 −3
x−2 y − 3 z +1
(ii) 2 x + y + 5z = 2 ; = =
2 1 5
x −1 y − 2 z + 7
(iii) 2 x + 6 y + z = 7; = =
2 6 1
(iv) 2 x − y + 2 z = 9 (v) 2 x − 2 y − z = 2
12. cos −1 √ (19 / 29) 13. a = 5 / 2, b = 1

8 Divergence of a Vector Point Function


(Agra 2005)
Definition: Let V be any given differentiable vector point function. Then the divergence of
V, written as, ∇ • V or div V,
 ∂ ∂ ∂
is defined as div V = ∇ • V =  i + j +k  •V
 ∂x ∂y ∂z 
∂V ∂V ∂V ∂V
=i• + j• +k • = ∑i • ⋅
∂x ∂y ∂z ∂x
It should be noted that div V is a scalar quantity. Thus the divergence of a vector point
function is a scalar point function.

Theorem: If V = V1 i + V2 j + V3 k is a differentiable vector point function, then


∂V1 ∂V2 ∂V3
div V = + + ⋅
∂x ∂y ∂z

Proof: We have by definition


V-73

∂V ∂V ∂V
div V = ∇ • V = i • + j• +k • ⋅
∂x ∂y ∂z
Now V = V1 i + V2 j + V3 k .
∂V ∂V1 ∂V2 ∂V3
∴ = i+ j+ k.
∂x ∂x ∂x ∂x
∂V  ∂V ∂V2 ∂V3  ∂V1
∴ i• =i • 1 i + j+ k = ⋅
∂x  ∂x ∂x ∂x  ∂x
∂V ∂V2 ∂V ∂V3
Similarly, j• = and k • = ⋅
∂y ∂y ∂z ∂z
∂V1 ∂V2 ∂V3
Hence, div V = + + ⋅
∂x ∂y ∂z
Solenoidal Vector: A vector V is said to be solenoidal if div V = 0.

9 Curl of a Vector Point Function


(Agra 2005)
Definition: Let f be any given differentiable vector point function. Then the curl or
rotation of f , written as ∇ × f , curl f or rot f is defined as
 ∂ ∂ ∂
curl f = ∇ × f =  i + j +k  ×f
 ∂x ∂y ∂z 
∂f ∂f ∂f ∂f
=i× + j× +k × = ∑i × ⋅
∂x ∂y ∂z ∂x
It should be noted that curl f is a vector quantity. Thus the curl of a vector point function
is a vector point function.
Theorem: If f = f1 i + f 2 j + f 3 k is a differentiable vector point function, then
 ∂f ∂f   ∂f ∂f   ∂f ∂f 
curl f =  3 − 2  i +  1 − 3  j +  2 − 1  k .
 ∂y ∂z   ∂z ∂x   ∂x ∂y 
Proof: We have by definition
∂f ∂f ∂f
curl f = ∇ × f = i × + j× +k ×
∂x ∂y ∂z
∂ ∂
=i× ( f1 i + f 2 j + f 3 k ) + j × ( f1 i + f 2 j + f 3 k )
∂x ∂y

+k× ( f1 i + f 2 j + f 3 k )
∂z
 ∂f ∂f ∂f   ∂f ∂f ∂f 
= i ×  1 i + 2 j + 3 k + j ×  1 i + 2 j + 3 k
 ∂x ∂x ∂x   ∂y ∂y ∂y 
 ∂f ∂f ∂f 
+ k ×  1 i + 2 j + 3 k
 ∂z ∂z ∂z 
V-74

 ∂f ∂f   ∂f ∂f   ∂f ∂f 
=  2 k − 3 j +  − 1 k + 3 i +  1 j − 2 i
 ∂x ∂x   ∂y ∂y   ∂z ∂z 

 ∂f ∂f   ∂f ∂f   ∂f ∂f 
=  3 − 2  i +  1 − 3  j +  2 − 1 k.
 ∂y ∂z   ∂z ∂x   ∂x ∂y 

Note: It should be noted that the expression for curl f can be written immediately
if we treat the operator ∇ as a vector quantity. Thus
 ∂ ∂ ∂
Curl f = ∇ × f =  i + j +k  × ( f1 i + f 2 j + f 3 k )
 ∂ x ∂ y ∂ z

i j k
∂ ∂ ∂ 
= 
∂x ∂y ∂z
 
f f2 f3
 1 
∂ ∂ ∂ ∂ ∂ ∂
= ∂y ∂z i −
∂x ∂z 
 j + ∂x ∂y k
  
 f2 f 3  f1 f 3  f1 f 2

 ∂f ∂f   ∂f ∂f   ∂f ∂f 
=  3 − 2  i +  1 − 3  j +  2 − 1 k .
 ∂y ∂z   ∂z ∂x   ∂x ∂y 
But we must take care that in the expansion of the determinant the operators
∂ ∂ ∂
, , must precede the functions f1 , f 2 , f 3 .
∂x ∂y ∂z

Irrotational vector: A vector f is said to be irrotational if ∇ × f = 0.

10 The Laplacian Operator ∇ 2


∂2 ∂2 ∂2
The Laplacian operator ∇ 2 is defined as ∇ 2 ≡ + + ⋅
∂x 2 ∂y 2 ∂z 2

∂2 f ∂2 f ∂2 f
If f is a scalar point function, then ∇ 2 f = + + ⋅
∂x 2 ∂y 2 ∂z 2
It should be noted that ∇ 2 f is also a scalar quantity.

∂2 f ∂2 f ∂2 f
If f is a vector point function, then ∇ 2 f = + + ⋅
∂x 2 ∂y 2 ∂z 2
It should be noted that ∇ 2 f is also a vector quantity.

Laplace’s equation: The equation ∇ 2 f = 0 is called Laplace’s equation. A function


which satisfies Laplace’s equation is called a harmonic function.
V- 75

Example 13: Prove that div r = 3. (Kumaun 2000; Garhwal 01;


Bundelkhand 01, 04; Meerut 03, 04, 07B, 08;
Agra 08; Lucknow 05; Kanpur 13; Avadh 09; Purvanchal 11)
Solution: We have r = x i + y j + z k .
 ∂ ∂ ∂ ∂r ∂r ∂r
By definition, div r = ∇ • r =  i + j +k  •r = i • + j• +k•
 ∂x ∂y ∂z  ∂x ∂y ∂z
 ∂r ∂r ∂r 
= i •i + j• j + k •k ∵ =i, = j, = k
 ∂x ∂y ∂z 
= 1 + 1 + 1 = 3.

Example 14: Prove that curl r = 0.


(Kumaun 2000; Garhwal 02; Meerut 11; Bundelkhand 06; Agra 08;
Avadh 09; Kashi 13; Purvanchal 11; Rohilkhand 14)
Solution: We have by definition
 ∂ ∂ ∂
curl r = ∇ × r =  i + j +k  ×r
 ∂x ∂y ∂z 
∂r ∂r ∂r
=i× + j× +k × ⋅
∂x ∂y ∂z
Now r = x i + y j + z k.
∂r ∂r ∂r
∴ =i, = j, = k.
∂x ∂y ∂z
∴ curl r = i × i + j × j + k × k
= 0 + 0 + 0 = 0.

Example 15: If f = x 2 y i − 2 xz j + 2 yz k , find


(i) div f , (Kumaun 2008)
(ii) curl f , (Meerut 2012)
(iii) curl curl f . (Meerut 2001; Kanpur 14)
Solution: (i) We have
 ∂ ∂ ∂ 2
div f = ∇ • f =  i + j +k  • ( x y i − 2 xz j + 2 yz k )
 ∂x ∂y ∂z 
∂ 2 ∂ ∂
= ( x y) + (−2 xz ) + (2 yz )
∂x ∂y ∂z
= 2 xy + 0 + 2 y
= 2 y ( x + 1).
V-76

 i j k
 ∂ ∂ ∂
(ii) We have curl f = ∇ × f =  
 ∂x ∂y ∂z 
2 
−2 xz 2 yz
x y 
∂ ∂  ∂ ∂ 2 
=  (2 yz ) − (−2 xz ) i −  (2 yz ) − ( x y) j
 ∂y ∂z   ∂x ∂z 
∂ ∂ 2 
+  (− 2 xz ) − ( x y) k
 ∂x ∂y 
2
= (2z + 2 x) i − 0 j + (−2z − x ) k
= (2 x + 2z ) i − ( x 2 + 2z ) k .
(iii) We have curl curl f = ∇ × ( ∇ × f ) = ∇ × [(2 x + 2z ) i − ( x 2 + 2z ) k ]
 i j k 
 ∂ ∂ ∂ 
= 
 ∂x ∂y ∂z 
 
2 x + 2z 0 − x 2 − 2z

∂  ∂ ∂ 
=  (− x 2 − 2z ) i −  (− x 2 − 2z ) − (2 x + 2z ) j
 ∂ y   ∂ x ∂ z 
 ∂ 
+ 0 − (2 x + 2z ) k
 ∂y 
= 0 i − (−2 x − 2) j + (0 − 0) k = (2 x + 2) j.

Example 16: Determine the constant a so that the vector


V = ( x + 3 y) i + ( y − 2 z ) j + ( x + az ) k is solenoidal.
(Rohilkhand 2009B)
Solution: A vector V is said to be solenoidal if div V = 0.
∂ ∂ ∂
We have div V = ∇ • V = ( x + 3 y) + ( y − 2z ) + ( x + az )
∂x ∂y ∂z
= 1 + 1 + a = 2 + a.
Now div V = 0 if 2 + a = 0 i. e., if a = − 2.
Example 17: Show that the vector V = (sin y + z ) i + ( x cos y − z ) j + ( x − y) k is
irrotational.
Solution: A vector V is said to be irrotational if curl V = 0. We have
curl V = ∇ × V
 i j k 
 ∂ ∂ ∂ 
= 
 ∂x ∂y ∂z 
 
sin y + z x cos y − z x− y
 
V-77

∂ ∂  ∂ ∂ 
=  ( x − y) − ( x cos y − z ) i −  ( x − y) − (sin y + z ) j
 ∂y ∂z   ∂x ∂z 
∂ ∂ 
+  ( x cos y − z ) − (sin y + z ) k
 ∂ x ∂ y 
= (−1 + 1) i − (1 − 1) j + (cos y − cos y) k = 0.
∴ V is irrotational.

Example 18: Prove that ∇ • (r 3 r) = 6r 3 . (Purvanchal 2006, 10)


Solution: We have r = x i + y j + z k .
∴ r 3 r = r 3 ( x i + y j + z k ) = r 3 x i + r 3 y j + r 3 z k.
∴ ∇ • (r 3 r) = div (r 3 r)
∂ 3 ∂ 3 ∂ 3
= (r x) + (r y) + (r z )
∂x ∂y ∂z
∂r ∂r ∂r
= r 3 + 3r 2 x + r 3 + 3r 2 y + r 3 + 3r 2 z
∂x ∂y ∂z
 ∂r ∂r ∂r 
= 3r 3 + 3r 2  x + y +z  …(1)
 ∂x ∂y ∂z 
Now r 2 = x2 + y2 + z 2 .
∂r ∂r x
∴ 2r = 2 x or = ⋅
∂x ∂x r
∂r y ∂r z
Similarly = and = ⋅
∂y r ∂z r
 x y z
∴ from (1), ∇ • (r 3 r) = 3r 3 + 3r 2  x ⋅ + y ⋅ + z ⋅ 
 r r r
 x2 + y2 + z 2 
= 3r 3 + 3r 2  

 r 
r2
= 3r 3 + 3r 2 ⋅ = 3r 3 + 3r 3 = 6r 3 .
r
Example 19: If V is a constant vector, show that
(i) div V = 0, (Rohilkhand 2005; Purvanchal 14)
(ii) curl V = 0. (Purvanchal 2014; Kumaun 15)
Solution: (i) We have
∂V ∂V ∂V
div V = i • + j• +k •
∂x ∂y ∂z
= i • 0 + j • 0 + k • 0 = 0.

(ii) We have
V-78

∂V ∂V ∂V
curl V = i × + j× +k ×
∂x ∂y ∂z
= i × 0 + j × 0 + k × 0 = 0.

Example 20: If a is a constant vector, find


(i) div (r × a) , (Bundelkhand 2006; Kumaun 07, 11)
(ii) curl (r × a). (Bundelkhand 2006; Kanpur 06; Kumaun 07;
Agra 08; Rohilkhand 14)
Solution: We have r = x i + y j + z k .

Let a = a1 i + a2 j + a3 k . Then the scalars a1 , a2 , a3 are all constants.


We have
i j k
r × a = x y z
 
a1 a2 a3

= (a3 y − a2 z ) i + (a1 z − a3 x) j + (a2 x − a1 y) k .


∂ ∂ ∂
(i) div (r × a) = (a3 y − a2 z ) + (a1 z − a3 x) + (a3 x − a1 y)
∂x ∂y ∂z

= 0 + 0 + 0 = 0.
(ii) curl (r × a) = ∇ × (r × a)

 i j k 
 ∂ ∂ ∂ 
= 
 ∂x ∂y ∂z 
 
a3 y − a2 z a1 z − a3 x a2 x − a1 y
 
∂ ∂ 
=  (a2 x − a1 y) − (a1 z − a3 x) i
 ∂y ∂z 
∂ ∂ 
−  (a2 x − a1 y) − (a3 y − a2 z ) j
 ∂ x ∂ z 

∂ ∂ 
+  (a1 z − a3 x) − (a3 y − a2 z ) k
 ∂x ∂y 
= − 2a1 i − 2a2 j − 2a3 k = − 2 (a1 i + a2 j + a3 k ) = − 2a .

Example 21: If V = e xyz (i + j + k), find curl V.


 i j k 
 
∂ ∂ ∂
Solution: We have curl V = 
 ∂x ∂y ∂z 
 xyz 
e e xyz e xyz
 
V-79

∂ ∂ xyz  ∂ ∂ xyz 
=  (e xyz ) − (e ) i −  (e xyz ) − (e ) j
 ∂y ∂z   ∂x ∂z 
∂ ∂ xyz 
+  (e xyz ) − (e ) k
 ∂ x ∂ y 
= e xyz ( xz − x y) i + e xyz ( x y − yz ) j + e xyz ( yz − xz ) k .

Example 22: Evaluate div f where f = 2 x 2 z i − xy 2 z j + 3 y 2 x k .

Solution: We have
 ∂ ∂ ∂ 2 2 2
div f = ∇ • f =  i + j +k  • (2 x z i − xy z j + 3 y x k )
 ∂x ∂y ∂z 
∂ ∂ ∂
= (2 x 2 z ) + (− xy 2 z ) + (3 y 2 x)
∂x ∂y ∂z
= 4 xz − 2 xyz + 0 = 2 xz (2 − y).
Example 23: Show that ∇ 2 ( x / r 3 ) = 0. (Kumaun 2008)

 x ∂ ∂   x
2 2 2

Solution: ∇ 2  3  =  2 + 2 + 2   3  ⋅
 r   ∂x ∂y ∂z   r 
∂2  x  ∂  ∂  x  ∂ 1 3 x ∂r 
Now 2   =   3  =  3 − 4 
∂x  r  ∂x  ∂x  r   ∂x  r
3
r ∂x 
∂ 1 3 x x  2 2 2 2 ∂r x
=  3 − 4  ∵ r = x + y + z gives ∂x = r 
∂x  r r r
∂ 1 3 x2  3 ∂r 6 x 15 x 2 ∂r
=  3 − 5  =− 4 − + 6
∂x  r r  r ∂x r 5 r ∂x

3 x 6 x 15 x 2 x 9 x 15 x 3
=− − + = − + 7 ⋅
r4 r r5 r6 r r5 r
∂2  x  ∂ ∂  x  ∂  3 x ∂r 
Again 2  3
=   3  = − 
∂y  r  ∂y  ∂y  r   ∂y  r 4 ∂y 

∂  3x y   ∂r y 
= − 4  ∵ = 
∂y  r r   ∂y r 
2
∂  3 xy  3 x 15 xy ∂r 3 x 15 x y
= − 5  = − 5 + = − + ⋅
∂y  r  r r 6 ∂y r5 r7
∂2  x  3 x 15 xz 2
Similarly   = − + ⋅
∂z 2  r 3  r5 r7
Therefore, adding we get
 x ∂ ∂2   x 
2
∂2
∇ 2  3  =  2 + 2 + 2   3 
 r   ∂x ∂y ∂z   r 
V-80

9x 15 x 3 3x 15 xy 2 3x 15 xz 2
=− + − + − +
r5 r7 r5 r7 r5 r7
15 x 15 x 15 x 15 x
=− + (x2 + y2 + z 2 ) = − 5 + 7 r 2 = 0.
r5 r7 r r

Comprehensive Exercise 3

1. (i) If F = x 2 z i − 2 y 3 z 2 j + xy 2 z k , find div F, curl F at (1, − 1, 1).

(ii) If f = ( y 2 + z 2 − x 2 ) i + (z 2 + x 2 − y 2 ) j + ( x 2 + y 2 − z 2 ) k ,
find div f and curl f .
(iii) If f = xy 2 i + 2 x 2 yz j − 3 yz 2 k , find div f and curl f . What are
their values at the point (1, − 1, 1) ? (Rohilkhand 2005)
3 3 3
2. Find div F and curl F where F = grad ( x + y +z − 3 xyz ).
(Rohilkhand 2007)
3. Find the divergence and curl of the vector
f = ( x 2 − y 2 ) i + 2 xyj + ( y 2 − xy) k . (Bundelkhand 2004)
4. Given φ = 2 x 3 y 2 z 4 , find div (grad φ).

5. If u = x 2 − y 2 + 4z , show that ∇ 2 u = 0.

6. If u = 3 x 2 y and v = xz 2 − 2 y, then find grad [(grad u) • (grad v)].

7. If f = ( x + y + 1) i + j + (− x − y) k , prove that f • curl f = 0.


8. (i) If f = f1 i + f 2 j + f 3 k , show that
∇ • f = ∇f1 • i + ∇f 2 • j + ∇f 3 • k . (Bundelkhand 2001)
3 3
(ii) Prove that ∇ • (r r) = 6 r .
9. Find the constants a, b, c so that the vector
F = ( x + 2 y + az ) i + (bx − 3 y − z ) j + (4 x + c y + 2 z ) k is irrotational.
(Bundelkhand 2005; Rohilkhand 08B)
10. Show that the vector F = 3 y z i + 4 x 3 z 2 j − 3 x 2 y 2 k is solenoid but not
4 2

irrotational . (Kashi 2014)

A nswers 3
1. (i) − 3 ; − 6i
(ii) − 2 ( x + y + z ) ; 2 ( y − z ) i + 2 (z − x) j + 2 ( x − y) k
V-81

(iii) y 2 + 2 x 2 z − 6 yz ; 9 ; − (3z 2 + 2 x 2 y) i + (4 xyz − 2 xy) k; − i − 2k


2. 6 ( x + y + z ); 0 3. 4 x ; (2 y − x) i + y j + 4 y k
4. 12 xy 2 z 4 + 4 x 3 z 4 + 24 x 3 y 2 z 2
6. (6 yz 2 − 12 x) i + (6 xz 2 ) j + (12 xyz ) k
9. a = 4 , b = 2 , c = −1

11 Vector Identities
1. Prove that div (A + B) = div A + div B or ∇ • (A + B) = ∇ • A + ∇ • B .
Proof: We have
 ∂ ∂ ∂
div (A + B) = ∇ • (A + B) =  i + j +k  • (A + B)
 ∂ x ∂ y ∂ z
∂ ∂ ∂
=i• (A + B) + j • (A + B) + k • (A + B)
∂x ∂y ∂z

 ∂A ∂B   ∂A ∂B   ∂A ∂B 
=i • +  + j• +  +k • + 
 ∂x ∂x   ∂y ∂y   ∂z ∂z 
 ∂A ∂A ∂A   ∂B ∂B ∂B 
= i • + j• +k •  + i • + j• +k • 
 ∂ x ∂ y ∂ z   ∂ x ∂ y ∂z 

= ∇ • A + ∇ • B = div A + div B .
2. Prove that curl (A + B) = curl A + curl B or ∇ × (A + B) = ∇ × A + ∇ × B .
(Rohilkhand 2008)

Proof: We have curl (A + B) = ∇ × (A + B)


 ∂ ∂ ∂
= i + j +k  × (A + B)
 ∂x ∂y ∂z 
∂  ∂A ∂B 
=Σi× (A + B) = Σ i ×  + 
∂x  ∂x ∂x 
∂A ∂B
=Σi× +Σi× = curl A + curl B .
∂x ∂x
3. If A is a differentiable vector function and φ is a differentiable scalar function, then
div (φA) = ( grad φ) • A + φ div A or ∇ • (φA) = (∇φ) • A + φ (∇ • A).
(Garhwal 2001; Agra 05; Kashi 14)
Proof: We have
 ∂ ∂ ∂
div (φA) = ∇ • (φA) =  i + j +k  • (φA)
 ∂x ∂y ∂z 
V-82

∂ ∂ ∂
=i• (φA) + j • (φA) + k • (φA)
∂x ∂y ∂z
 ∂    ∂φ ∂A  
= Σ  i •  (φA)  = Σ  i •  A+φ 
  ∂x     ∂x ∂x  
  ∂φ     ∂A  
= Σ i •  A  + Σ  i •  φ 
  ∂x     ∂x  
  ∂φ     ∂A  
= Σ  i • A + Σ  φ  i • 
  ∂x     ∂x  
[ ∵ a • (mb) = (ma) • b = m (a • b)]
 ∂φ   ∂A 
= Σ i • A + φ Σ  i •  = (∇φ) • A + φ (∇ • A).
 ∂ x   ∂x 
4. Prove that curl (φA) = ( grad φ) × A + φ curl A
or ∇ × (φA) = (∇φ) × A + φ (∇ × A).
(Garhwal 2002, 03; Bundelkhand 06)
 ∂ ∂ ∂
Proof: We have curl (φA) = ∇ × (φA) =  i + j +k  × (φA)
 ∂x ∂y ∂z 

 ∂    ∂φ ∂A  
= Σ i × (φA) = Σ  i ×  A+φ 
 ∂x    ∂x ∂x  
  ∂φ     ∂A  
= Σ i ×  A  + Σ  i ×  φ 
  ∂x     ∂x  
  ∂φ     ∂A  
= Σ  i × A + Σ  φ  i × 
  ∂x     ∂x  
[∵ a × (mb) = (ma) × b = m (a × b)]
  ∂φ   ∂A 
= Σ  i  × A + φ Σ  i ×  = (∇φ) × A + φ (∇ × A).
  ∂x   ∂x 
5. Prove that div (A × B) = B • curl A − A • curl B
or ∇ • (A × B) = B • (∇ × A) − A • (∇ × B).
(Garhwal 2003; Agra 05; Meerut 04, 05B, 08, 09;
Bundelkhand 05, 07; Kashi 13; Avadh 09)
Proof: We have
 ∂    ∂A ∂B 
div (A × B) = Σ  i • (A × B) = Σ  i •  ×B+A× 
 ∂ x    ∂ x ∂x 
  ∂A    ∂B  
= Σ i •  × B  + Σ  i •  A × 
  ∂ x     ∂x  
 ∂A     ∂B 
= Σ  i ×  • B − Σ i •  × A 
  ∂ x     ∂ x 
[∵ a • (b × c) = (a × b) • c and a • (b × c) = – a • (c × b)]
V-83

  ∂A    ∂B  
= Σ  i ×   • B − Σ  i ×  • A
  ∂ x     ∂ x  
  ∂B  
= (curl A) • B −  Σ i ×  • A
  ∂x  
= (curl A) • B − (curl B) • A = B • curl A − A • curl B.
6. Prove that curl (A × B) = (B • ∇) A − B div A − (A • ∇) B + A div B.
(Garhwal 2001; Meerut 06B; Kumaun 09, 11, 12)
Proof: We have curl (A × B) = ∇ × (A × B)
 ∂    ∂B ∂A 
= Σ i × (A × B) = Σ  i ×  A × + × B 
 ∂x    ∂x ∂x 
  ∂B     ∂A 
= Σ i ×  A ×   + Σ i ×  × B 
  ∂x     ∂x 
 ∂B  ∂B   ∂A  ∂A  
= Σ  i •  A − (i • A)  + Σ (i • B) − i •  B
  ∂ x  ∂ x   ∂ x  ∂x  
 ∂B    ∂B 
= Σ  i •  A  − Σ (A • i) 
  ∂ x    ∂x 
 ∂A   ∂A  
+ Σ (B • i)  − Σ  i •  B
 ∂x   ∂x  
  ∂B    ∂  ∂
= Σ  i •   A − A • Σ i  B + B • Σ i  A
  ∂ x    ∂ x   ∂ x
  ∂A  
− Σ  i •  B
  ∂x  
= (div B) A − (A • ∇) B + (B • ∇) A − (div A) B .

7. Prove that grad (A • B) = (B • ∇ ) A + (A • ∇) B + B × curl A + A × curl B .


(Agra 2001; Kumaun 15)

Proof: We have
∂  ∂B ∂A 
grad (A • B) = ∇ (A • B) = Σ i (A • B) = Σ i  A • + • B
∂x  ∂x ∂x 
 ∂B    ∂A  
= Σ  A •  i + Σ   B •  i ⋅ …(1)
  ∂x     ∂x  
Now we know that a × (b × c) = (a • c) b - (a • b) c .
∴ (a • b) c = (a • c) b - a × (b × c).
 ∂B  ∂B  ∂B 
∴ A •  i = (A • i) −A× × i
 ∂x  ∂x  ∂x 
∂B  ∂B 
= (A • i) + A × i × ⋅
∂x  ∂x 
V-84

 ∂B    ∂B    ∂B  
Thus Σ  A •  i = Σ (A • i)  + Σ A ×  i × 
  ∂x    ∂ x    ∂x  
 ∂  ∂B 
= A • Σ i  B + A × Σ  i × 
 ∂x   ∂x 
= (A • ∇) B + A × (∇ × B). …(2)
 ∂A  
Similarly Σ  B •  i = (B • ∇) A + B × (∇ × A). …(3)
 ∂x  
Putting the values from (2) and (3) in (1), we get
grad (A • B) = (A • ∇ ) B + A × (∇ × B) + (B • ∇ ) A + B × (∇ × A).
Note: If we put A in place of B , then
grad (A • A) = 2 (A • ∇) A + 2A × (∇ × A)
1
or grad A2 = (A • ∇) A + A × curl A.
2

8. Prove that div grad φ = ∇ 2 φ i. e., ∇ • (∇φ) = ∇ 2 φ .


Proof: We have
 ∂ ∂ ∂   ∂φ ∂φ ∂φ 
∇ • (∇φ) =  i + j +k  • i+ j+ k
 ∂x ∂y ∂z   ∂x ∂y ∂z 

∂  ∂φ ∂  ∂φ ∂  ∂φ


=   +   +  
∂x  ∂x  ∂y  ∂y  ∂z  ∂z 

∂2 φ ∂2 φ ∂2 φ  ∂2 ∂2 ∂2 
= + + =  2 + 2 + 2  φ = ∇ 2 φ.
∂x 2 ∂y 2 ∂z 2  ∂x ∂y ∂z 

9. Prove that curl of the gradient of φ is zero i. e., ∇ × (∇φ) = 0 , i. e., curl grad φ = 0 .
(Bundelkhand 2014)
∂φ ∂φ ∂φ
Proof: We have grad φ = i+ j+ k.
∂x ∂y ∂z
∴ curl grad φ = ∇ × grad φ
 ∂ ∂ ∂   ∂φ ∂φ ∂φ 
= i + j +k  × i+ j+ k
 ∂x ∂y ∂z   ∂x ∂y ∂z 

 
i j k
 
∂ ∂ ∂
= ∂x ∂y ∂z
 
∂φ ∂φ ∂φ
∂x ∂y ∂z
 
 ∂2 φ ∂2 φ   ∂2 φ ∂2 φ   ∂2 φ ∂2 φ 
= −  i+ −  j+  −  k
 ∂y ∂z ∂z ∂y   ∂z ∂x ∂x ∂z   ∂x ∂y ∂y ∂x 
V-85

= 0 i + 0 j + 0 k = 0,
provided we suppose that φ has continuous second partial derivatives so that the
order of differentiation is immaterial.

10. Prove that div curl A = 0 , i. e., ∇ • (∇ × A) = 0.


(Garhwal 2000; Meerut 02, 06B, 09; Agra 2000;
Rohilkhand 05; Bundelkhand 08; Kumaun 14)
Proof: Let A = A1 i + A2 j + A3 k . Then

 i j k 
 ∂ ∂ ∂ 
curl A = ∇ × A =  
 ∂x ∂y ∂y 
 A1 A2 A3 
 ∂A3 ∂A2   ∂A1 ∂A3   ∂A2 ∂A1 
=  −  i +  −  j+  −  k.
 ∂y ∂z   ∂z ∂x   ∂x ∂y 

Now div curl A = ∇ • (∇ × A)


∂  ∂A3 ∂A2  ∂  ∂A1 ∂A3  ∂  ∂A2 ∂A1 
=  −  +  −  +  − 
∂x  ∂y ∂z  ∂y  ∂z ∂x  ∂z  ∂x ∂y 

∂ 2 A3 ∂ 2 A2 ∂ 2 A1 ∂ 2 A3 ∂ 2 A2 ∂ 2 A1
= − + − + −
∂x ∂y ∂x ∂z ∂y ∂z ∂y ∂x ∂z ∂x ∂z ∂y
= 0 , assuming that A has continuous second partial derivatives.

11. Prove that ∇ × (∇ × A) = ∇ (∇ • A) − ∇ 2 A . (Meerut 2000, 07B, 10B;


Rohilkhand 14; Kumaun 10)
Proof: Let A = A1 i + A2 j + A3 k .

 i j k 
 ∂ ∂ ∂ 
Then ∇ × A = 
 ∂x ∂y ∂z 
 A1 A2 A3 
 ∂A3 ∂A2   ∂A1 ∂A3   ∂A2 ∂A1 
=  −  i +  −  j+  −  k.
 ∂y ∂z   ∂z ∂x   ∂x ∂y 

 
 i j k 
 
∂ ∂ ∂
∴ ∇ × (∇ × A) =  
 ∂x ∂y ∂z 
∂A3 ∂A2 ∂A1 ∂A3 ∂A2 ∂A1 
 − − − 
 ∂y ∂z ∂z ∂x ∂x ∂y 
V-86

 ∂  ∂A2 ∂A1  ∂  ∂A1 ∂A3   


= Σ   −  −  −  i 
 ∂y  ∂x ∂y  ∂z  ∂z ∂x   

  ∂ 2 A ∂ 2 A3   ∂ 2 A1 ∂ 2 A1   
 2
   i 
=Σ  + − +
  ∂y ∂x ∂z ∂x   ∂y 2 ∂z 2   
  
  ∂A2 ∂A3   ∂ 2 A1 ∂ 2 A1   
∂   i 
= Σ   +  − +
 ∂x  ∂y ∂z   ∂y 2 ∂z 2   
  
  ∂A1 ∂A2 ∂A3   ∂ 2 A1 ∂ 2 A1 ∂ 2 A1   
∂  
= Σ   + +  − + + i
 ∂x  ∂x ∂y ∂z   ∂x 2 ∂y 2 ∂z 2   
  
 ∂  
= Σ  (∇ • A) − (∇ 2 A1 ) i 
 ∂ x  
 ∂  
= Σ  (∇ • A) i  − ∇ 2 Σ A1 i = ∇ (∇ • A) − ∇ 2 A.
 ∂x  

Example 24: Taking F = x 2 yi + xzj + 2 yzk , verify that div curl F = 0.


(Garhwal 2002; Bundelkhand 04; Kanpur 10)
 i j k 
∂ ∂ ∂ 
Solution: We have curl F =  
 ∂x ∂y ∂z 
2 
2 yz
x y xz 
∂ ∂  ∂ ∂ 2 
=  (2 yz ) − ( xz ) i −  (2 yz ) − ( x y) j
 ∂y ∂z   ∂x ∂z 
∂ ∂ 2 
+  ( xz ) − ( x y) k
 ∂ x ∂ y 
= (2z − x) i − 0 j + (z − x 2 ) k = (2z − x) i + (z − x 2 ) k .
Now div curl F = div [(2z − x) i + (z − x 2 ) k ]
∂ ∂
= (2z − x) + (z − x 2 ) = − 1 + 1 = 0.
∂x ∂z
Example 25: Prove that div (r n r) = (n + 3) r n . (Bundelkhand 2006)

Solution: We have
div (φA) = φ (div A) + A • grad φ .
Putting A = r and φ = r n in this identity, we get
V- 87

div (r n r) = r n div r + r • grad r n


= 3r n + r • (n r n − 1 grad r)
[∵ div r = 3 and grad f (u) = f ′ (u) grad u]
 ∧ 1 
= 3r n + r • n r n − 1 r
1
 ∵ grad r = r = r
 
r   r 
= 3r n + n r n − 2 (r • r)
= 3 r n + n r n − 2 r2
= (n + 3) r n .
 r
Example 26: Prove that div  3  = 0.
r  (Bundelkhand 2010)
Solution: We have
 1 
div  3 r = div (r −3 r) = r −3 div r + r • grad r −3
r 

= 3r −3 + r • (−3r −4 grad r) = 3r −3 + r •  −3r −4 r


1
 r 
= 3r −3 − 3r −5 (r • r) = 3r −3 − 3r −5 r 2
= 3r −3 − 3r −3 = 0.
∴ the vector r −3 r is solenoidal.

Example 27: Prove that div r = 2 / r. (Bundelkhand 2007; Kanpur 05)

div ( r ) = div  r ⋅ Now proceed as in Example 25.
1
Solution:
r 
Alternative Method:

div r = div  r = div  ( x i + y j + z k )
1 1
r   r 
x y z  ∂  x ∂ y  ∂ z
= div  i + j + k =   +   +  
r r r  ∂x r  ∂y  r  ∂z  r 

1 x ∂r   1 y ∂r   1 z ∂r 
= − 2  + − 2  + − ⋅
 r r ∂x   r r ∂y   r r 2 ∂z 

Now r 2 = x2 + y2 + z 2 .
∂r ∂r x
∴ 2r = 2 x i. e., = ⋅
∂x ∂x r
∂r y ∂r z
Similarly = and = ⋅
∂y r ∂z r
∧ 3  x x y y z z
∴ div r= − 2 + 2 + 2 
r r r r r r r
V-88

2 2 2
3 x + y +z 3 r2 3 1 2
= − = − 3 = − = ⋅
r r3 r r r r r
2
Example 28: Prove that ∇ 2 f (r) = f ′ ′ (r) + f ′ (r).
r (Meerut 2003, 05, 06B)
Solution: We know that if φ is a scalar function, then ∇ 2 φ = ∇ • (∇φ).
∴ ∇ 2 f (r) = ∇ • {∇f (r)} = div { grad f (r) }

= div { f ′ (r) grad r} = div  f ′ (r) r


1
r 

= f ′ (r) div r + r • grad  f ′ (r)


1 1
r r 
d 1 
f ′ (r) + r •   f ′ (r) grad r
3
=
r  dr  r  
3  1 1  1 
= f ′ (r) + r •  − 2 f ′ (r) + f ′ ′ (r) r
r  r r  r 
3 1  1 1 
= f ′ (r) + r  − 2 f ′ (r) + f ′ ′ (r)  (r • r)
r   r r 
3 1  1 1 
= f ′ (r) +   − 2 f ′ (r) + f ′ ′ (r)  r 2
r r  r r 
3 1 2
= f ′ (r) − f ′ (r) + f ′ ′ (r) = f ′ ′ (r) + f ′ (r).
r r r

Prove that ∇ 2   = 0 or div  grad  = 0 .


1 1
Example 29:
 r  r
( Agra 2002; Meerut 07, 13)

Solution: We have

∇ 2   = ∇ • ∇  = div  grad 


1 1 1
 r  r  r
 1   1 1   1 
= div  − 2 grad r = div  − 2 r = div  − 3 r
 r   r r   r 
 1  1 3 d  1 
=  − 3  div r + r • grad  − 3  = − 3 + r •   − 3  grad r
 r   r  r  dr  r  
3 3 1  3 3 3 3
=− +r • 4 r = − 3 + 5 (r • r) = − 3 + 5 r 2 = 0 .
3
r r r  r r r r
∴ 1 / r is a solution of Laplace’s equation.
Example 30: Prove that div grad r n = n (n + 1) r n − 2 , i. e., ∇ 2 r n = n (n + 1) r n − 2 .
(Bundelkhand 2005, 10; Meerut 2000, 08; Avadh 10; Agra 14)
Solution: We have ∇ 2 r n = ∇ • (∇r n ) = div (grad r n )
V-89

= div (nr n − 1 grad r) = div  nr n − 1 r = div (nr n − 2 r)


1
 r 
= (nr n − 2 ) div r + r • (grad nr n − 2 )
= 3nr n − 2 + r • [n (n − 2) r n − 3 grad r]

= 3nr n − 2 + r • n (n − 2) r n − 3 r
1
 r 
= 3nr n − 2 + r • [n (n − 2) r n − 4 r] = 3nr n − 2 + n (n − 2) r n − 4 (r • r)
= 3nr n − 2 + n (n − 2) r n − 4 r 2 = nr n − 2 (3 + n − 2) = n (n + 1) r n − 2 .
−1
Note: If n = − 1, then ∇ 2 (r ) = (−1)(−1 + 1) r −3 = 0.
  1  3
Example 31: Prove that ∇ •  r ∇  3   = 4 or, div [r grad r −3 ] = 3r −4 .
  r   r (Meerut 2009B)
 1
Solution: We have ∇  3  = grad r −3
r 
∂ −3 ∂ −3 ∂ −3
= (r ) i + (r ) j + (r ) k .
∂x ∂y ∂z
∂ −3 ∂r
Now (r ) = − 3r − 4 ⋅
∂x ∂x
But r 2 = x2 + y2 + z 2 .
∂r ∂r x
Therefore 2r = 2x or = ⋅
∂x ∂x r
∂ −3 x
So (r ) = − 3r − 4 = − 3r −5 x .
∂x r
∂ −3 ∂ −3
Similarly (r ) = − 3r −5 y and (r ) = − 3r −5 z .
∂y ∂z
 1
Therefore ∇  3  = − 3r −5 ( x i + yj + zk ).
r 
 1
∴ r ∇  3  = − 3r − 4 ( xi + yj + zk ).
r 
 1 ∂ ∂ ∂
∴ ∇ • r ∇ 3  = (−3r − 4 x) + (− 3r − 4 y) + (−3r − 4 z ).
 r  ∂x ∂y ∂z
∂ ∂r
Now (−3r − 4 x) = 12r −5 x − 3r − 4
∂x ∂x
x
= 12r −5 x − 3r − 4 = 12r −6 x 2 − 3r − 4 .
r

Similarly (−3r y) = 12r y − 3r − 4
−4 −6 2
∂y

and (−3r − 4 z ) = 12r − 6 z 2 − 3r − 4 .
∂z
V-90

 1
Hence ∇ •  r ∇ 3  = 12 r − 6 ( x 2 + y 2 + z 2 ) − 9r − 4
 r 
= 12 r −6 r 2 − 9r − 4 = 12 r − 4 − 9r − 4 = 3r − 4 .
Example 32: If a is a constant vector, prove that div { r n (a × r)} = 0.(Kanpur 2008)

Solution: We have div (φA) = φ div A + A • grad φ .


∴ div { r n (a × r) } = r n
div (a × r) + (a × r) • grad r n
= r n div (a × r) + (a × r) • (nr n − 1 grad r)

= r n (r • curl a − a • curl r) + (a × r) •  nr n −1 r


1
 r 
= r n (r • 0 – a • 0) + nr n − 2 (a × r) • r
[∵ curl of a constant vector is zero and curl r = 0 ]
n−2
= nr [a , r , r]
= 0 , since a sca lar triple product having two equal vec tors is zero.
Example 33: If a and b are constant vectors, prove that
(i) div [(r × a) × b] = − 2 b • a . (Bundelkhand 2010)
(ii) curl [(r × a) × b] = b × a . (Bundelkhand 2010)
Solution: (i) We have (r × a) × b = ( b • r) a − ( b • a) r .
∴ div [(r × a) × b] = div [( b • r) a − ( b • a) r]
= div [( b • r) a] − div [( b • a) r] …(1)
But div (φA) = φ div A + A • grad φ .
Taking φ = b • r and A = a , we get
div [( b • r) a] = ( b • r) div a + a • grad ( b • r).
Since a is a constant vector, therefore div a = 0 .
Also let b = b1 i + b2 j + b3 k .
Then b • r = (b1 i + b2 j + b3 k ) • ( xi + yj + zk )
= b1 x + b2 y + b3 z , where b1 , b2 , b3 are constants.
∴ grad ( b • r) = b1 i + b2 j + b3 k = b .
∴ div [( b • r) a] = a • b . …(2)
Again div [( b • a)] r = ( b • a) div r + r • grad ( b • a).
But div r = 3.
Also grad ( b • a) = 0 because b • a is constant.
∴ div [( b • a) r] = 3 ( b • a). …(3)
Substituting the values from (2) and (3) in (1), we get
div [(r × a) × b] = (a • b) − 3 ( b • a) = − 2 b • a .
(ii) curl [(r × a) × b] = curl [( b • r) a − ( b • a) r]
= curl [( b • r) a] − curl [( b • a) r].
V-91

But curl (φA) = grad φ × A + φ curl A .


∴ curl [( b • r ) a] = [ grad ( b • r )] × a + ( b • r ) curl a = b × a
[∵ curl a = 0 and grad ( b • r ) = b]
Also curl [( b • a ) r ] = [ grad ( b • a )] × r + ( b • a ) curl r = 0
[∵ grad ( b • a ) = 0 and curl r = 0]
∴ curl [( r × a ) × b] = b × a – 0 = b × a .
Example 34: Prove that curl [r × (a × r)] = 3r × a , where a is a constant vector.
(Purvanchal 2009)
Solution: curl [r × (a × r)]
= ∇ × [(r • r) a − (r • a) r] [∵ a × ( b × c) = (a • c) b − (a • b) c]
2
= ∇ × [r a − (r • a) r] [∵ r • r = r2 = r2 ]
= ∇ × (r 2 a) − ∇ × [(r • a) r] [∵ ∇ × (A + B) = ∇ × A + ∇ × B]
2 2
= (∇r ) × a + r (∇ × a) − [∇ (r • a)] × r − (r • a) (∇ × r)
[∵ ∇ × (φA) = (∇φ) × A + φ (∇ × A)]
2
= (2r ∇r) × a + r 0 − [∇ (r • a)] × r − (r • a) 0
[∵ ∇f (r) = f ′ (r) ∇r ; ∇ × a = 0, a
being a constant vector ; and ∇ × r = 0]
 1 
= 2r r × a − [∇ (r • a)] × r
 r 
= 2r × a − a × r [∵ ∇ (r • a) = a, if a is a constant vector]
= 2r × a + r × a = 3r × a.

Example 35: If a is a constant vector, prove that


a×r a 3r
curl 3 = − 3 + 5 (a • r).
r r r (Meerut 2009B; Bundelkhand 09, 11)
a×r  a × r  ∂  a × r
Solution: We have curl = ∇ ×  3  = Σ i ×   ⋅
r 3  r   ∂x  r 3  

∂  a × r 3 ∂r 1  ∂r  1  ∂a 
Now  3  =− 4 (a × r) + 3  a ×  + 3  × r …(1)
∂x  r  r ∂x r  ∂x  r  ∂x 
∂a
Now = 0 because a is a constant vector.
∂x
∂r
Also r = x i + y j+ z k. ∴ =i.
∂x
∂r x
Further = ⋅
∂x r
∂  a × r 3 x 1
∴ (1) becomes  3  =− 4 (a × r) + 3 (a × i)
∂x  r  r r r
V-92

3x 1
=− 5
(a × r) + (a × i).
r r3
∂  a × r 3x 1
∴ i×  3  = − 5 i × (a × r) + 3 i × (a × i)
∂x  r  r r
3x 1
= − 5 [(i • r) a – (i • a) r] + 3 [(i • i) a − (i • a) i]
r r
3x 3x 1 1
= − 5 xa + 5 a1 r + 3 a − 3 a1 i
r r r r

[∵ i • r = x and i • a = a1 if a = a1 i + a2 j + a3 k ]
3 x2 3 1 1
=− 5
a+ 5
a1 xr + 3
a− a1 i .
r r r r3
 ∂  a × r
∴ Σ i ×  
 ∂x  r 3  
 3  3  3 1
=  − 5 Σ x 2  a +  5 Σ a1 x r + 3 a − 3 Σ a1 i
 r  r  r r
3 2 3 3 1
= − 5 r a + 5 (r • a) r + 3 a − 3 a
r r r r
[∵ Σ x 2 = r 2 , Σ a1 x = r • a , Σ a1 i = a]
a 3
=− 3
+ (a • r) r .
r r5

Comprehensive Exercise 4

1. (i) Verify that curl grad f = 0 , where f = x 2 y + 2 xy + z 2 .


(ii) Prove that grad f (u) = f ′(u) grad u.
2
+ y 2 + z 2 )1 /2
(iii) Find ∇φ and |∇φ| when φ = ( x 2 + y 2 + z 2 ) e − ( x .
(Kumaun 2012)
2. (i) Prove that curl (ψ∇φ) = ∇ψ × ∇φ = − curl (φ∇ψ).
(ii) Prove that ∇ 2 (φψ) = φ∇ 2 ψ + 2∇φ • ∇ψ + ψ∇ 2 φ. (Kumaun 2014)
(iii) Prove that div (∇φ × ∇ψ) = 0.
(iv) If A and B are irrotational, prove that A × B is solenoidal.
(v) Prove that curl (φ grad φ) = 0.
3. (i) Prove that a • {∇( v • a) − ∇ × ( v × a)} = div v, where a is a constant
unit vector.
(ii) Prove that vector f (r) r is irrotational.
(iii) If f and g are two scalar point functions, prove that
div ( f ∇ g) = f ∇ 2 g + ∇f • ∇g.
V-93

(iv) Show that curl (a • r) a = 0 , where a is a constant vector.


(Kumaun 2007, 09)
[ Hint: Use identity 4. Note that ∇ (a • r) = a , if a is a constant vector.]
4. If a is a constant vector, then prove that
(i) ∇ (a • u) = (a • ∇) u + a × curl u,
(ii) ∇ • (a × u) = − a • curl u,
(iii) ∇ × (a × u) = a div u − (a • ∇) u ,

5. (i) Given that ρ F = ∇p, where ρ, p, F are point functions, prove that
F • curl F = 0.
(ii) A vector function f is the product of a scalar function and the gradient
of a scalar function. Show that f • curl f = 0.
1
(iii) Show that curl a φ (r) = φ′ (r) r × a , where a is a constant vector.
r
n−2
6. (i) Prove that curl [r n
(a × r)] = (n + 2) r n
a − nr (r • a) r,
where a is a constant vector. (Kumaun 2008)
2 n−2
(ii) Prove that ∇ (r r) = n (n + 3) r
n
r.
7. (i) Prove that curl grad r n = 0. (Avadh 2010; Kanpur 11)
2 c1
(ii) If ∇ f (r) = 0 , show that f (r) = + c2 ,
r
where r 2 = x 2 + y 2 + z 2
and c1 , c 2 are arbitrary constants.

8. If r is the position vector of the point ( x , y , z ) show that curl (r n r) = 0, where


r is the module of r.
9. Prove that r n r is an irrotational vector for any value of n but is solenoidal
only if n + 3 = 0.
10. (i) If u = (1 / r) r, show that ∇ × u = 0. (Avadh 2010)
(ii) Prove that div (A × r) = r • curl A .
11. (i) If ∇ 2 f (r) = 0 show that f (r) = c1 log r + c 2 where r 2 = x 2 + y 2 and
c1 , c 2 are arbitrary constants.
(ii) If a and b are constant vectors, then show that ∇ • (a • br) = 3a • b .
12. If u = (1 / r) r find grad (div u).
  r 
13. (i) Prove that ∇ 2 ∇ •  2   = 2r − 4 .
  r  (Kumaun 2011, 13)
a•r
(ii) Prove that a • ∇  = − 3 ⋅
1
 r r

(iii) Prove that ∇ • (U ∇V − V ∇U ) = U ∇ 2 V − V ∇ 2 U.


V-94

1 3 (a • r) (b • r) a • b
(iv) Prove that b • ∇ a • ∇  = − ,
 r r5 r3
where a and b are constant vectors. (Kumaun 2015)
14. Evaluate div {a × (r × a)}, where a is a constant vector. (Kanpur 2007)
 f (r) r  1 d 2
15. (i) Prove that div   = (r f ).
 r  r 2 dr (Kumaun 2007, 14)
1 2
(ii) Prove that ∇ a = (a • ∇) a + a × curl a .
2
16. Prove that curl [r × (a × r)] = 3r × a , where a is a constant vector.
17. Prove that ∇ × (F × r) = 2F − (∇ • F) r + (r • ∇) F. (Kumaun 2015)
18. If a and b are constant vectors, prove that
grad [(r × a) • (r × b)] = (b × r) × a + (a × r) × b. (Kumaun 2015)

A nswers 4
2
1. (iii) (2 − r) e − r r ; (2 − r) e − r r 12. − r
r3
14. 2 a2

O bjective T ype Q uestions

Multiple Choice Questions


Indicate the correct answer for each question by writing the corresponding letter from
(a), (b), (c) and (d).
1. If the vector V = ( x + 3 y) i + ( y − 2z ) j + ( x + az ) k is solenoidal, then the
constant a is
(a) 0 (b) 1
(c) –2 (d) 2
2. The directional derivative of φ ( x, y, z ) = x 2 yz + 4 xz 2 at (1, − 2 , − 1) in the
direction of the vector 2 i − j − 2 k is
(a) 37/3 (b) 3/37
(c) 3 (d) none of these
3. If r = |r | where r = x i + y j + z k , then ∇ 2 r n
=
n −1
(a) n (n + 1) r n
(b) n (n + 1) r
n−2
(c) n (n + 1) r (d) none of these (Agra 2007)
V-95

∇ 2   =
1
4.
 r
(a) −2 / r 3 (b) 0
3
(c) 2 / r (d) none of these
(Kumaun 2014)
5. If r = xi + yj + zk and a is a constant vector, then curl (r × a) is
(a) − a (b) − 3a
(c) − 2 a (d) none of these

6. The value of div ^


r is
(a) 2 / r (b) 0
(c) 1 / r (d) none of these (Agra 2014)
7. The V is a constant vector, then div V is :
(a) 3 (b) 3V
(c) 0 (d) none of these
(Kumaun 2007, 11)
2
8. If f = x yi − 2 xzj + 2 yzk , then div f is equal to
(a) 2 x ( x + 1) (b) 2 y ( x + 1)
(c) y ( y + 1) (d) none of these
(Kumaun 2008)
9. ∇ × (∇ f ) is equal to
(a) ∇ 2 f (b) 0
(c) 0 (d) none of these
(Kumaun 2010)

Fill in the Blank(s)


Fill in the blanks “……”, so that the following statements are complete and correct.
1. If F = ( x 2 + y 2 ) i − 2 xy j , then F • dr = …… .
∂P
2. If P = e xy i + ( x − 2 y) j + ( x sin y) k , then = …… .
∂x
3. If a is a constant vector then grad (a • r) = …… .
(Rohilkhand 2007; Kumaun 12)
4. If a is a constant vector, then ∇ • (a × r) = …… .
5. If r = xi + yj + zk , then the value of div r = …… .
(Agra 2008; Kumaun 15)

6. If A = x 2 zi − 2 y 3 z 2 j + xy 2 zk , then div A at (1, − 1, 1) = …… .


7. If r = xi + yj + zk , then the value of curl r = …… .
(Agra 2008; Rohilkhand 14)
V-96

8. For any vector A, div curl A = …… .


9. A vector V is said to be solenoidal if …… . (Rohilkhand 2005; Agra 06)
10. A vector F is said to be irrotational if …… .
11. If φ = x 2 y + 2 xy + z 2 , then curl grad φ = …… . (Bundelkhand 2007)
12. If f = x 2 yi + 2 xzj + 2 yzk , then div (curl f ) = …… . (Kumaun 2010, 12)
13. Value of div grad φ is …… . (Kumaun 2011)
14. If a is a constant vector, then curl (r × a) = ...... . (Kumaun 2013)

True or False
Write ‘T’ for true and ‘F’ for false statement.
1. If r = xi + yj + zk , then r is solenoidal.
2. If V is a constant vector, then div V = 0. (Rohilkhand 2005)
3. If F = 2 xyz i + y 2 zj − 2 yz 2 k , then F is irrotational.
4. If φ is a differentiable scalar function, then curl grad φ = 0.
5. If φ is a differentiable scalar function then div grad φ = ∇ 2 φ .
6. ∇ • (A × B) = A • (∇ × B) − B • (∇ × A).
7. A function which satisfies Laplace’s equation is called a harmonic function.

A nswers

Multiple Choice Questions


1. (c) 2. (a) 3. (c)
4. (b) 5. (c) 6. (a)
7. (c) 8. (b) 9. (b)

Fill in the Blank(s)


1. ( x 2 + y 2 ) dx − 2 xydy 2. y e xy i + j + sin y k 3. a
4. 0 5. 3 6. –3
7. 0 8. 0 9. div V = 0
10. curl F = 0 11. 0 12. 0
13. ∇ 2 φ 14. −2a

True or False
1. F 2. T 3. F
4. T 5. T 6. F
7. T

¨
V-97

I ntegration of V ectors

1 Integration of Vector Functions


e shall define integration as the reverse process of differentiation. Let f (t) and F (t)
W be two vector functions of the scalar t such that
d
F (t) = f (t).
dt
Then F (t) is called the indefinite integral of f (t) with respect to t and symbolically
we write
∫ f (t) dt = F (t). …(1)
The function f (t) to be integrated is called the integrand.
If c is any arbitrary constant vector independent of t, then
d
{F (t) + c } = f (t).
dt
This is equivalent to
∫ f (t) dt = F (t) + c . …(2)
From (2) it is obvious that the integral F (t) of f (t) is indefinite to the extent of an
additive arbitrary constant c .Therefore F (t) is called the indefinite integral of f (t).
V-98

The constant vector c is called the constant of integration. It can be determined if we


are given some initial conditions.
d
If F (t) = f (t) for all t in the interval [a , b], then the definite integral between the
dt
limits t = a and t = b can, in such case, be written as
b d 
∫ a f (t) dt = ∫ a  dt F (t) dt = [F (t) + c] a = F (b) − F (a).
b b

Theorem: If f (t) = f1 (t) i + f 2 (t) j + f 3 (t) k , then

∫ f (t) dt = i ∫ f1 (t) dt + j ∫ f 2 (t) dt + k ∫ f 3 (t) dt.


d
Proof: Let F (t) = f (t). …(1)
dt
Then ∫ f (t) dt = F (t). …(2)
Let F (t) = F1 (t) i + F2 (t) j + F3 (t) k .
Then from (1), we have
d
{ F1 (t) i + F2 (t) j + F3 (t) k } = f (t)
dt
 d F (t) i +  d F (t) j +  d F (t) k
or  1   2   3 
 dt   dt   dt 
= f1 (t) i + f 2 (t) j + f 3 (t) k .
Equating the coefficients of i , j, k , we get
d d d
F1 (t) = f1 (t), F2 (t) = f 2 (t), F3 (t) = f 3 (t).
dt dt dt
∴ F1 (t) = ∫ f1 (t) dt, F2 (t) = ∫ f 2 (t) dt, F3 (t) = ∫ f 3 (t) dt.
∴ F (t) = {∫ f1 (t) dt} i + {∫ f 2 (t) dt} j + {∫ f 3 (t) dt} k .

So from (2), we get

∫ f (t) dt = i ∫ f1 (t) dt + j ∫ f 2 (t) dt + k ∫ f 3 (t) dt .

Note: From this theorem we conclude that the definition of the integral of a
vector function implies the definition of integrals of three scalar functions which
are the components of that vector function. Thus in order to integrate a vector
function we should integrate its components.

2 Some Standard Results


We have already obtained some standard results for differentiation. With the help
of these results we can obtain some standard results for integration.
d dr ds
1. We have (r • s) = •s+r • ⋅
dt dt dt
V-99

 dr • s + r • ds  dt = r • s + c ,
Therefore ∫  dt dt 
where c is the constant of integration. It should be noted that c is here a scalar
quantity since the integrand is also scalar.
d 2 dr 2 r • dr  dt = r 2 + c .
2. We have
dt
(r ) = 2r •
dt
⋅ Therefore ∫  dt 
Here the constant of integration c is a scalar quantity.
d  dr  2 dr d2 r
3. We have   =2 • ⋅
dt  dt  dt dt 2
 dr d2 r  dr 2
Therefore we have ∫ 2 • 2  dt =   + c .
 dt dt   dt 

Here the constant of integration c is a scalar quantity.


2
Also  dr dr dr
• = ⋅
 dt  dt dt
d  d2 r d2 r
 r ×  =
dr dr dr
4. We have × +r× 2 =r× 2 ⋅
dt  dt  dt dt dt dt
 d r
2
dr
∴ ∫  r × dt 2  dt = r × dt + c .
Here the constant of integration c is a vector quantity since the integrand
d2 r
r × 2 is also a vector quantity.
dt
5. If a is a constant vector, we have
d da dr dr
(a × r) = ×r+a× =a× ⋅
dt dt dt dt

Therefore ∫  a ×  dt = a × r + c .
dr
 dt 
Here the constant of integration c is a vector quantity.

6. If r = |r | and ^
r is a unit vector in the direction of r, then
d ^ d 1  1 d r 1 dr
(r)=  r = − 2 r.
dt dt  r  r dt r dt
 1 dr 1 dr 
Therefore ∫  − 2 r dt = ^
r + c.
 r dt r dt 
7. If c is a constant scalar and r a vector function of a scalar t ,then obviously
∫ cr dt = c ∫ r dt .
8. If r and s are two vector functions of the scalar t , then obviously

∫ (r + s) dt = ∫ r dt + ∫ s dt.
V-100

2 d2 r
Example 1: Evaluate ∫ r× dt, where r = 2t 2 i + t j − 3t 3 k .
1 dt 2
(Bundelkhand 2004; Kumaun 08; Kanpur 13)

Solution: Given r = 2t 2 i + t j − 3t 3 k .
dr d2 r
∴ = 4t i + j − 9t 2 k and = 4i + 0 j − 18tk .
dt dt 2
d2 r
∴ r× 2
= (2t 2 i + tj − 3t 3 k ) × (4i + 0 j − 18tk )
dt
 i j k 
= 2t 2 t − 3t  3

 

4 0 −18t 

= − 18t 2 i − (− 36t 3 + 12t 3 ) j − 4t k


= − 18 t 2 i + 24t 3 j − 4tk.
2 d2 r 2
∴ ∫1 r×
dt 2
dt = ∫1 (−18t 2 i + 24t 3 j − 4tk ) dt

2 2 2
= − 18i ∫1 t 2 dt + 24 j ∫
1
t 3 dt − 4k ∫
1
t dt
2 2 2
 t3   t4   t2 
= − 18i   + 24 j   − 4k  
 3 1  4 1  2 1
= − 6 (8 − 1) i + 6 (16 − 1) j − 2 (4 − 1) k
= − 42i + 90 j − 6k .
d2 r
Example 2: Find the value of r satisfying the equation = ta + b, where a and b are
dt 2
constant vectors.
d2 r dr 1 2
Solution: Integrating the equation 2
= ta + b, we get = t a + tb + c, where
dt dt 2
c is constant.
Again integrating, we get
1 1
r = t 3 + t 2 b + tc + d , where d is constant.
6 2
Example 3: If r (t) = 5t 2 i + tj − t 3 k , prove that
2  d2 r 
∫1  r × 2  dt = − 14i + 75 j − 15k .
 dt  (Kumaun 2000; Meerut 01, 04, 05, 07, 10, 10B;
Kanpur 10; Rohilkhand 13)
V-101

 d2 r  dr
Solution: We have ∫  r × 2  dt = r × + c.
 dt  dt
 d2 r  dr 2
 r × 2  dt = r ×  ⋅
2
∴ ∫1  dt   dt 1
dr dr
Let us now find r × ⋅ We have = 10 t i + j − 3t 2 k .
dt dt
dr
∴ r× = (5t 2 i + t j − t 3 k ) × (10 t i + j − 3t 2 k )
dt
 i j k 
 2 
= 5t t − t 3 = − 2t 3 i + 5t 4 j − 5t 2 k .

 

10 t 1 −3t 2

 d2 r 
[ ]
2 2
∴ ∫1  r × 2  dt = −2t 3 i + 5t 4 j − 5t 2 k
 dt  1

[ ] [ ] [ ]
2 2 2
= − 2t 3 i + 5t 4 j − 5t 2 k
1 1 1

= − 14i + 75 j − 15k.
 2i − j + 2k , when t = 2
Example 4: Given that r (t) = 
 4i − 2 j + 3k , when t = 3,

show that ∫
3  r • dr  dt = 10.
2  dt  (Meerut 2003, 13B; Bundelkhand 08;
Kanpur 09, 11; Agra 06; Avadh 10; Purvanchal 13)
3
 r • dr  dt =  1 r 2  ⋅
Solution: We have ∫  dt  2  2

When t = 3, r = 4i − 2 j + 3k .
∴ when t = 3, r 2 = (4i − 2 j + 3k ) • (4i − 2 j + 3k ) = 16 + 4 + 9 = 29.
When t = 2 , r = 2i − j + 2k .
∴ when t = 2 , r 2 = 4 + 1 + 4 = 9.
3  r • dr  dt = 1 [29 − 9] = 10.
∴ ∫2  dt  2
Example 5: The acceleration of a particle at any time t ≥ 0 is given by
dv
a= = 12 cos 2t i − 8 sin 2t j + 16 t k .
dt
If the velocity v and displacement r are zero at t = 0, find v and r at any time.
(Agra 2007)
dv
Solution: We have = 12 cos 2t i − 8 sin 2t j + 16t k .
dt
Integrating, we get
V-102

v = i ∫ 12 cos 2t dt + j ∫ −8 sin 2t dt + k ∫ 16t dt

or v = 6 sin 2t i + 4 cos 2t j + 8t 2 k + c.
When t = 0, v = 0.
∴ 0 = 0 i + 4 j + 0 k + c or c = − 4 j.
dr
∴ v= = 6 sin 2t i + (4 cos 2t − 4) j + 8t 2 k .
dt
Integrating, we get

r = i ∫ 6 sin 2t dt + j ∫ (4 cos 2t − 4) dt + k ∫ 8t
2
dt
8 3
= − 3 cos 2t i + (2 sin 2t − 4t) j + t k + d,
3
where d is constant.
When t = 0, r = 0.
∴ 0 = − 3i + 0 j + 0 k + d . ∴ d = 3i .
8 3
∴ r = − 3 cos 2t i + (2 sin 2t − 4t) j + t k + 3i
3
8
= (3 − 3 cos 2t) i + (2 sin 2t − 4t) j + t 3 k .
3

Comprehensive Exercise 1

2
1. If f (t) = (t − t 2 ) i + 2 t 3 j − 3k , find (i) ∫ f (t) dt (ii) ∫1 f (t) dt.
1
2. Evaluate ∫ e t i + e −2 t j + t k) dt . (Garhwal 2001, 02)
0
1
3. If f (t) = t i + (t 2 − 2 t) j + (3t 2 + 3t 3 ) k , find ∫ f (t) dt.
0

(Garhwal 2003; Bundelkhand 07)


4. If r = t i − t 2 j + (t − 1) k and s = 2t 2 i + 6t k , evaluate
2 2
(i) ∫0 r • s dt, (ii) ∫0 r × s dt
(Rohilkhand 2008)
d2 r
5. (i) Find the value of r satisfying the equation = a , where a is a
dt 2
dr
constant vector. Also it is given that when t = 0, r = 0 and = u.
dt
d2 r
(ii) Solve the equation = a where a is a constant vector ; given that
dt 2
dr
r = 0 and = 0 when t = 0.
dt (Bundelkhand 2008)
V-103

d2 r
6. Find the value of r satisfying the equation 2
= 6ti − 24t 2 j + 4 sin t k ,
dt
given that r = 2i + j and dr / dt = − i − 3 k at t = 0.
(Agra 2001; Meerut 11)
7. The acceleration of a particle at any time t is e i + e2 t j + k .
t

Find v, given that v = i + j at t = 0.


2
8. Evaluate ∫ (a • b × c) dt , where
1
a = ti − 3 j + 2tk , b = i − 2 j + 2k , c = 3i + t j − k . (Meerut 2013)

d2 r
9. Integrate 2
= − nr 2 .
dt (Kumaun 2009)

A nswers 1
t 2 t 3  t4 5 15
1. (i)  +  i+ j − 3t k + c (ii) −i+ j−3k
2 3 2 6 2
1 1 1 2 7
2. (e − 1) i − (e −2 − 1) j + k 3. i − j+ k
2 2 2 3 4
40 64
4. (i) 12 (ii) − 24 i − j+ k
3 5
1 2 1 2
5. (i) t a + tu (ii) t a
2 2
6. r = (t 3 − t + 2) i + (1 − 2t 4 ) j + (t − 4 sin t) k
1 2t
7. et i + (e + 1) j + t k 8. 0
2
9. − n2 r 2 + c

O bjective T ype Q uestions

Multiple Choice Questions


Indicate the correct answer for each question by writing the corresponding letter from
(a), (b), (c) and (d).
1
1. If F (t) = t i + (t 2 − 2 t) j + (3t 2 + 3t 3 ) k , then the value of ∫ F (t) dt is
0

1 2 7 1 2 7
(a) i + j+ k (b) i − j+ k
2 3 4 2 3 4
1 2 7
(c) − i − j + k (d) None of these
2 3 4
(Bundelkhand 2001)
V-104

1
2. If r = t i − t 2 j + (t − 1) k , and s = 2t 2 i + 6t k , then the value of ∫ r • s dt is
0

(a) 10 (b) 12
(c) 15 (d) None of these
(Kumaun 2007, 10)

Fill in the Blank(s)


Fill in the blanks “……”, so that the following statements are complete and correct.
1
1. ∫0 [t i + (t 2 − 2t) j ] dt = …… .

2. If F (t) = 3t 2 i + t j + 2 k and G (t) = 6t 2 i + (t − 1) j + 3t k , then


1  dF • G + F • dG dt = …… .
∫0  dt dt  (Meerut 2011)
2
2 d r
3. If r (t) = 5t 2 i + t j − t 3 k , then ∫ r× dt = …… .
1 dt 2 (Kumaun 2013)
1 dr
4. If r = t 2 i + j − t k , then ∫ r• dt = …… .
0 dt (Kumaun 2014)

True or False
Write ‘T’ for true and ‘F’ for false statement.
 dr d2 r  dr 2
1. ∫ 2 • 2  dt =   + c .
 dt dt   dt 
1
2. The value of ∫ (e t i + e − 2 t j + t k ) dt is (e − 1) i − (e − 2 − 1) j + k .
0

A nswers

Multiple Choice Questions


1. (b) 2. (d)

Fill in the Blank(s)


1 2
1. i− j 2. 24
2 3
3. −14 i + 75 j − 15 k 4. 1

True or False
1. T 2. F

¨
Line Integrals
V-105

Line Integrals

1 Some Preliminary Concepts


riented curve. Suppose C is a
O curve in space. Let us orient C by
taking one of the two directions
along C as the positive direction; the
opposite direction along C is then called
the negative direction . Suppose A is the
initial point and B the terminal point of
C under the chosen orientation. In case these two points coincide, the curve C is
called a closed curve.
Smooth curve. Let r (t) = x (t) i + y (t) j + z (t) k , where r (t) is the position
vector of ( x, y, z ), be the parametric representation of a curve C joining the points A
and B, where t = t1 and t = t2 respectively. We know that dr / dt is a tangent vector
V-106

to this curve at the point r. Suppose the function r (t) is continuous and has a
continuous first derivative not equal to zero vector for all values of t under
consideration. Then the curve C possesses a unique tangent at each of its points. A
curve satisfying these assumptions is called a smooth curve.
A curve C is said to be
piecewise smooth if it is
composed of a finite number
of smooth curves. The curve
C in the adjoining figure is
piecewise smooth as it is
composed of three smooth curves C1 , C2 and C3 . The circle is a smooth closed
curve while the curve consisting of the four sides of a rectangle is a piecewise
smooth closed curve.
Smooth surface. Suppose S is a surface which has a unique normal at each of its
points and the direction of this normal depends continuously on the points of S.
Then S is called a smooth surface.
If a surface S is not smooth but can be subdivided into a finite number of smooth
surfaces, then it is called a piecewise smooth surface. The surface of a sphere is
smooth while the surface of a cube is piecewise
smooth.
Classification of regions. A region R in which
every closed curve can be contracted to a point
without passing out of the region is called a
simply connected region. Otherwise the region
R is multiply-connected. The region interior to a
circle is a simply-connected plane region. The
region interior to a sphere is a simply-connected
region in space. The region between two
concentric circles lying in the same plane is a
multiply-connected plane region.
If we take a closed curve in this region surrounding the inner circle, then it cannot
be contracted to a point without passing out of the region. Therefore the region is
not simply-connected. However the region between two concentric spheres is a
simply-connected region in space. The region between two infinitely long coaxial
cylinders is a multiply-connected region in space.

2 Line Integrals
(Avadh 2014)
Any integral which is to be evaluated along a curve is called a line integral.
V-107

Suppose r (t) = x (t) i + y (t) j + z (t) k ,where r (t) is the position vector of ( x, y, z )
i. e., r (t) = x i + y j + z k , defines a piecewise smooth curve joining two points A
and B. Let t = t1 at A and t = t2 at B. Suppose F( x, y, z ) = F1 i + F2 j + F3 k is a vector
point function defined and continuous along C. If s denotes the arc length of the
dr
curve C, then = t is a unit vector along the tangent to the curve C at the point r.
ds
dr dr
The component of the vector F along this tangent is F • . The integral of F •
ds ds
along C from A to B written as
B  dr  B
∫ A F • ds  ds = ∫ A F • dr = ∫ C F • dr
is an example of a line integral. It is called the tangent line integral of F along C.
Since r = x i + y j + z k , therefore, dr = dx i + dy j + dz k .
∴ F • dr = ( F1 i + F2 j + F3 k ) • (dx i + dy j + dz k )
= F1 dx + F2 dy + F3 dz .
Therefore in components form the above line integral is written as
∫C F • dr = ∫C ( F1 dx + F2 dy + F3 dz ).

The parametric equations of the curve C are x = x(t), y = y(t) and z = z (t).
Therefore we may write
t2  dx dy dz 
∫C F • dr = ∫t 1  F1 dt + F2 dt + F3 dt  dt.

Circulation: If C is a simple closed curve ( i.e. a curve which does not intersect itself
anywhere), then the tangent line integral of F around C is called the circulation of F about C.
It is often denoted by
∫ F • dr = ∫ ( F1 dx + F2 dy + F3 dz ).

Work done by a Force. Suppose a force F acts upon a particle. Let the particle be
displaced along a given path C in space. If r denotes the position vector of a point
dr
on C,then is a unit vector along the tangent to C at the point r in the direction of s
ds
dr
increasing. The component of force F along tangent to C is F • . Therefore the
ds
work done by F during a small displacement ds of the particle along C is F •  ds
dr
 ds 
i. e., F • dr. The total work W done by F in this displacement along C, is given by the
line integral
W= ∫C F • dr,

the integration being taken in the sense of the displacement.


V-108

Example 1: Evaluate ∫C F • dr, where F = x 2 i + y 3 j and curve C is the arc of the

parabola y = x 2 in the x-y plane from (0, 0) to (1, 1).

Solution: We shall illustrate two methods for the solution of such a problem.
Method 1. The curve C is the parabola y = x 2 from (0, 0) to (1, 1).
Let x = t ; then y = t 2 . If r is the position vector of any point ( x, y) on C, then
dr
r (t) = xi + yj = ti + t 2 j . ∴ = i + 2tj.
dt
Also in terms of t, F = t 2 i + t 6 j.
At the point (0, 0), t = x = 0. At the point (1, 1), t = 1.
 dr  1 2
∫ C F • dr = ∫ C  F • dt  dt = ∫0 (t i + t j) • (i + 2t j) dt
6

1
1 2  t3 2t 8  1 1 7

7
= t + 2t ) dt =  +  = + = .
0
3 8  0 3 4 12
Method 2: In the xy-plane we have r = xi + yj .
∴ dr = dxi + dyj.

Therefore, F • dr = ( x 2 i + y 3 j) • (dx i + dy j) = x 2 dx + y 3 dy.


∴ ∫C F • dr = ∫C ( x 2 dx + y 3 dy).

Now along the curve C, y = x 2 . Therefore dy = 2 x dx.


1
∴ ∫C F • dr = ∫ x =0 [ x 2 dx + x 6 (2 x) dx]
1
1  x3 2 x8  7
= ∫0 ( x 2 + 2 x 7 ) dx = 
3
+
8
 =
12
.
 0

Example 2: If F = 3 xy i − y 2 j , evaluate ∫ F • dr, where C is the curve in the x y-plane,


C

y = 2 x 2 , from (0, 0) to (1, 2). (Garhwal 2001, 02; Kumaun 07; Rohilkhand 12)
Solution: The parametric equations of the parabola y = 2 x 2 can be taken as
x = t, y = 2t 2 .
At the point (0, 0), x = 0 and so t = 0. Again at the point (1, 2), x = 1and so t = 1.
Now ∫C F • dr = ∫C (3 xyi − y 2 j) • (dx i + dy j)

[∵ r = xi + yj, so that dr = dxi + d y j]


V-109

1  dx dy 
= ∫C (3 xy dx − y 2 dy) = ∫t =0 3 xy
 dt
− y2  dt
dt 
1
= ∫0 (3 . t . 2 t 2 . 1 − 4 t 4 . 4t) dt

[∵ x = t, y = 2t 2 so that dx / dt = 1 and dy / dt = 4t]


1
1  t4 t6 
∫0 (6t − 16t ) dt = 6 . 4 − 16 . 6 
3 5
=
 0
6 16 3 8 7
= − = − =− .
4 6 2 3 6
Example 3: Evaluate ∫ F • dr along the curve x 2 + y 2 = 1, z = 1in the positive direction
from (0, 1, 1) to (1, 0, 1) where F = (2 x + yz ) i + xzj + ( xy + 2z ) k .
Solution: Let the given curve be denoted by C and let A and B be points (0, 1, 1) and
(1, 0, 1) respectively.
Along the given curve C, we have r = xi + yj + zk .
∴ dr = dx i + dy j + dz k .
B
∴ ∫C F • dr = ∫A [(2 x + yz ) i + xz j + ( xy + 2z ) k ] • (dx i + dy j + dz k )
B
= ∫A [(2 x + yz ) dx + xz dy + ( xy + 2z ) dz ]. ...(1)

In moving from A to B, x varies from 0 to 1 , y varies from 1 to 0 and z remains


constant. We have z = 1 and so dz = 0.
Hence from (1)
1 0
∫C F • dr = ∫0 (2 x + y) dx + ∫
1
x dy + 0
1 1 1
= ∫0 [2 x + √ (1 − x 2 )] dx − ∫
0
√ (1 − y 2 ) dy = [ x 2 ] 0 = 1,

the last two integrals cancel by a property of definite integrals.


Example 4: Evaluate ∫ ( x dy − y dx) around the circle x 2 + y 2 = 1.
(Meerut 2002)
2 2
Solution: Let C denote the circle x + y = 1. The parametric equations of this
circle are x = cos t, y = sin t.
To integrate around the circle C we should vary t from 0 to 2π.
2π  dy dx 
∴ ∫C ( x dy − y dx) = ∫0 x
 dt
− y  dt
dt 
2π 2π
= ∫0 (cos 2 t + sin2 t) dt = ∫0 dt = 2π.
Example 5: If F = (2 x + y) i + (3 y − x) j , evaluate ∫ F • dr where C is the curve in
C
the xy-plane consisting of the straight lines from (0, 0) to (2, 0) and then to (3, 2).
(Agra 2007; Meerut 11; Purvanchal 14)
V-110

Solution: The path of integration C has been shown in the figure.


It consists of the straight lines OA and AB.
We have ∫C F • dr

= ∫C [(2 x + y) i + (3 y − x) j] • (dx i + dy j)

= ∫C [(2 x + y) dx + (3 y − x) dy].

Now along the straight line OA, y = 0, dy = 0 and x


varies from 0 to 2. The equation of the straight line AB is
2−0
y−0 = ( x − 2) i. e., y = 2 x − 4.
3−2
∴ along AB, y = 2 x − 4, dy = 2 dx and x varies from 2 to 3.
2 3
∴ ∫C F • dr = ∫0 [(2 x + 0) dx + 0] + ∫
2
[(2 x + 2 x − 4) dx

+ (6 x − 12 − x) 2 dx]

[x ]
2 3
= 2
+∫ (14 x − 28) dx
0 2
3
3 ( x − 2)2 
= 4 + 14∫ ( x − 2) dx = 4 + 14   = 4 + 7 = 11.
2 2
  2
Example 6: Evaluate ∫ F • dr where F = ( x 2 + y 2 ) i − 2 xyj , curve C is the rectangle in
C

the x y-plane bounded by y = 0, x = a, y = b, x = 0.


(Meerut 2000, 06B, 07B, 09B, 12, 13; Kanpur 10; Bundelkhand 09;
Purvanchal 09)
Solution: In the xy-plane z = 0.
Therefore
r = x i + y j and dr = dx i + dy j .
The path of integration C has been shown in
the figure. It consists of the straight lines
OA, AB, BD and DO.
We have
∫C F • dr = ∫C [( x 2 + y 2 ) i − 2 xy j] • (dx i + dy j)

= ∫C [( x 2 + y 2 ) dx − 2 xy dy].

Now on OA, y = 0, dy = 0 and x varies from 0 to a ; on AB, x = a, dx = 0 and y


varies from 0 to b ; on BD, y = b, dy = 0 and x varies from a to 0; on
DO, x = 0, dx = 0 and y varies from b to 0.
a b 0 0
∴ ∫C F • dr = ∫0 x 2 dx − ∫
0
2a y dy + ∫
a
( x 2 + b 2 ) dx + ∫
b
0 dy
V-111

a b 0
 x3   y2   x3 
=   − 2a   + + b 2 x + 0 = − 2ab 2 .
 3 0  2  0  3 a
Example 7: Find the total work done in moving a particle in a force field given by
F = 3 xyi − 5zj + 10 xk
along the curve x = t + 1, y = 2t 2 , z = t 3 from t = 1 to t = 2.
2

Solution: Let C denote the arc of the given curve from t = 1to t = 2. Then the total
work done
=∫C F • dr = ∫C (3 x yi − 5z j + 10 x k ) • (dx i + dy j + dz k )

=∫ (3 xy dx − 5z dy + 10 x dz )
C
2  dx dy dz 
= ∫1 3 x y
 dt
− 5z
dt
+ 10 x  dt
dt 
2
= ∫1 [3 (t 2 + 1) (2t)2 (2t) − (5t 3 )(4t) + 10 (t 2 + 1)(3t 2 )] dt
2
= ∫1 (12t 5 + 12t 3 − 20 t 4 + 30 t 4 + 30 t 2 ) dt
2
= ∫1 (12t 5 + 10 t 4 + 12t 3 + 30 t 2 ) dt = 303.

Example 8: Evaluate ∫ F • dr, where F = yz i + zx j + x y k and C is the portion of the


C

curve r = a cos t i + b sin t j + ct k , from t = 0 to t = π / 2. (Avadh 2010)


Solution: Along the curve C,
r = x i + y j + z k = a cos t i + b sin t j + ct k .
∴ x = a cos t, y = b sin t, z = ct.
Now ∫C F • dr = ∫C ( yz i + z x j + x y k ) • (dx i + dy j + dz k )

= ∫C ( yz dx + z x dy + x y dz ) = ∫C d ( x yz )
t = π /2
= [ x yz ] t =0 = [(a cos t) . (b sin t) . (ct)] 0
π /2

= abc [t cos t sin t ] 0π /2 = abc (0 − 0) = 0.

Comprehensive Exercise 1

1. Find ∫ t • dr where t is the unit tangent vector and C is the unit circle,
C

in xy-plane, with centre at the origin. (Bundelkhand 2008)

2. (i) Integrate the function F = x 2 i − xyj from the point (0, 0) to (1, 1) along
parabola y 2 = x.
V-112

(ii) Evaluate ∫ x y 3 ds, where C is the segment of the line y = 2 x in the


C

xy-plane from (−1, − 2) to (1, 2).


3. Evaluate ∫ F • dr, where F = i cos y − j x sin y and C is the curve
C
2
y = √ (1 − x ) in the xy-plane from (1, 0) to (0, 1). (Agra 2001)

∫C
2 2 2
4. (i) Evaluate F • dr where F is x y i + yj and C is y = 4 x in the

xy-plane from (0, 0) to (4, 4).


(Garhwal 2003; Meerut 04, 06;
Kanpur 09, 11, 12; Rohilkhand 11)
(ii) Evaluate ∫C F • dr, where F = x y i + yz j + z x k and C is the curve

r = ti + t 2 j + t 3 k , t varying from −1 to + 1.
(iii) Evaluate ∫C F • dr, where F = (2 x + y) i + (3 y − x) j + yzx k and C is

the curve x = 2t 2 , y = t, z = t 3 from t = 0 to t = 1. (Kumaun 2011)


5. Evaluate ∫ F • dr where
C

F = c [−3a sin2 t cos t i + a (2 sin t − 3 sin3 t) j + b sin 2t k ]


and C is given by r = a cos t i + a sin t j + bt k from t = π / 4 to π / 2.
6. Evaluate ∫ F • dr where F = z i + x j + y k and C is the arc of the curve
C

r = cos t i + sin t j + t k from t = 0 to t = 2π .


7. Evaluate ∫ F • dr where F = x y i + ( x 2 + y 2 ) j and C is the x-axis from
C

x = 2 to x = 4 and the straight line x = 4 from y = 0 to y = 12.


8. Find the work done in moving a particle in a force field
F = 3 x 2 i + (2 xz − y) j + z k along the line joining (0, 0, 0) to (2, 1, 3).
9. Calculate ∫ [( x 2 + y 2 ) i + ( x 2 − y 2 ) j] • dr where C is the curve :
C

(i) y 2 = x joining (0, 0) to (1, 1). (Meerut 2005B)


(ii) x 7 = y joining (0, 0) to (1, 1).
(iii) consisting of two straight lines joining (0, 0) to (1, 0) and (1, 0) to (1, 1).
(iv) consisting of three straight lines joining (0, 0) to (2, − 2), (2, − 2) to
(0, − 1) and (0, − 1) to (1, 1).
10. Find the circulation of F round the curve C, where
F=e sin y i + e x cos y j and C is the rectangle whose vertices are
x

1 1
(0, 0), (1, 0), (1, π), (0, π).
2 2
11. Find the circulation of F round the curve C, where F = ( x − y) i + ( x + y) j
and C is the circle x 2 + y 2 = 4, z = 0.
V-113

12. (i) If F = (2 x 2 + y 2 ) i + (3 y − 4 x) j , evaluate ∫ F • dr around the triangle

ABC whose vertices are A (0, 0), B (2, 0) and C (2, 1).
(Kumaun 2008)


2 2
(ii) If F = (3 x + 6 y) i − 14 yz j + 20 xz k , then evaluate F • dr from

(0, 0, 0) to (1, 1, 1) along the curve x = t, y = t 2 , z = t 3 .


(Kumaun 2012)

A nswers 1
1 16
1. 2π 2. (i) (ii) 3. − 1
12 √5
10 1
4. (i) 264 (ii) 5. c (a2 + b 2 ) 6. 2 π + π = 3π
7 2
7. 768 8. 16
7 38 7
9. (i) (ii) (iii) 1 (iv) − 10. 0
10 45 3
11. 8π 12. (i) −14 / 3 (ii) 5

3 Surface Integrals
(Avadh 2014)
Any integral which is to be evaluated over a surface is called a surface integral.

Suppose S is a surface of finite area. Suppose


f ( x, y, z ) is a single valued function of position
defined over S. Subdivide the area S into n elements
of areas δS1 , δS2 , …, δS n . In each part δS k we
choose an arbitrary point Pk whose coordinates are
( x k , y k , z k ).
We define f ( Pk ) = f ( x k , y k , z k ).
n
Form the sum ∑ f ( Pk ) δS k .
k =1

Now take the limit of this sum as n → ∞ in such a way that the largest of the areas
δS k approaches zero. This limit if it exists, is called the surface integral of f ( x, y, z )
over S and is denoted by ∫ ∫ f ( x, y, z ) dS.
S

It can be shown that if the surface S is piecewise smooth and the function f ( x, y, z )
is continuous over S, then the above limit exists i. e., is independent of the choice of
sub-divisions and points Pk .
V-114

Flux: Suppose S is a piecewise smooth surface


and F( x, y, z ) is a vector function of position
defined and continuous over S. Let P be any point
on the surface S and let n be the unit vector at P in
the direction of outward drawn normal to the
surface S at P. Then F • n is the normal
component of F at P. The integral of F • n over S is
∫ ∫ F • n dS.
S

It is called the flux of F over S.


Let us associate with the differential of surface area dS a vector dS (called vector area)
whose magnitude is dS and whose direction is that of n. Then dS = n dS. Therefore
we can write ∫ ∫ F • n dS = ∫ ∫ F • dS.
S S

Suppose the outward drawn normal to the surface S at P makes angles α, β, γ with
the positive directions of x, y and z axes respectively. If l, m, n are the direction
cosines of the outward drawn normal, then
l = cos α, m = cos β, n = cos γ .
Also n = cos α i + cos β j + cos γ k = l i + m j + n k .
Let F( x, y, z ) = F1 i + F2 j + F3 k . Then
F • n = F1 cos α + F2 cos β + F3 cos γ = F1 l + F2 m + F3 n.
Therefore, we can write
∫ ∫S F • n dS = ∫ ∫S ( F1 cos α + F2 cos β + F3 cos γ ) dS

= ∫ ∫S ( F1 dy dz + F2 dz dx + F3 dx dy),

if we define ∫ ∫S F1 cos α dS = ∫ ∫S F1 dy dz ,

∫ ∫S F2 cos β dS = ∫ ∫S F2 dz dx, ∫ ∫S F3 cos γ dS = ∫ ∫


S
F3 dx dy.

Note 1: Other examples of surface integrals are ∫ ∫ f n dS, ∫ ∫ F × dS


S S
where f ( x, y, z ) is a scalar function of position.
Note 2: Important. In order to evaluate surface
integrals it is convenient to express them as
double integrals taken over the orthogonal
projection of the surface S on one of the
coordinate planes. But this is possible only if any
line perpendicular to the co-ordinate plane
chosen meets the surface S in no more than one
point. If the surface S does not satisfy this
condition, then it can be sub-divided into surfaces which do satisfy this
condition.
V-115

Suppose the surface S is such that any line perpendicular to the xy-plane meets S in
no more than one point. Then the equation of the surface S can be written in the
form z = h ( x, y).
Let R be the orthogonal projection of S on the x y-plane. If γ is the acute angle
which the undirected normal n at P( x, y, z ) to the surface S makes with
z-axis, then it can be shown that cos γ dS = dx dy,
where dS is the small element of area of surface S at the point P.
dx dy dx dy
Therefore dS = = , where k is the unit vector along z-axis.
cos γ |n • k |
dx dy
Hence ∫ ∫S F • n dS = ∫ ∫R F •n
|n • k|
.

Thus the surface integral on S can be evaluated with the help of a double integral
integrated over R.

Example 9: Evaluate ∫ ∫ F • n dS, where F = yz i + zx j + xy k and S is that part of the


S
surface of the sphere x + y 2 + z 2 = 1 which lies in the first octant.
2

(Garhwal 2003; kumaun 14))


Solution: A vector normal to the surface S is given by
∇ ( x 2 + y 2 + z 2 ) = 2 x i + 2 y j + 2z k .
Therefore n = a unit normal to any point ( x, y, z ) of S
2 x i + 2 y j + 2z k
= = xi + yj + zk ,
√ (4 x 2 + 4 y 2 + 4z 2 )
since x 2 + y 2 + z 2 = 1 on the surface S.
dx dy
We have ∫ ∫S F • n dS = ∫ ∫R F•n
|n • k |
,

where R is the projection of S on the xy-plane.


The region R is bounded by x-axis, y-axis and the circle x 2 + y 2 = 1, z = 0.
We have F • n = ( yz i + zx j + x y k ) • ( x i + y j + z k ) = 3 x yz .
Also n • k = ( x i + y j + z k ) • k = z . ∴ |n • k | = z .
3 x yz
Hence ∫ ∫ S F • n dS = ∫ ∫ R z dx dy = 3∫ ∫ R x y dx dy
π/2 1
= 3∫ ∫r =0 (r cos θ) (r sin θ) r dθ dr, on changing to polars
θ =0
1
π/2 r4  3  1 = 3 .
= 3∫   cos θ sin θ dθ =
0  2 8
 4 0 4
V-116

Example 10: Evaluate ∫ ∫ F • n dS,where F = z i + x j − 3 y 2 zk and S is the surface of


S

the cylinder x 2 + y 2 = 16 included in the first octant between z = 0 and z = 5.

Solution: A vector normal to the surface S is given by


∇ ( x 2 + y 2 ) = 2 x i + 2 y j.
Therefore n = a unit normal to any point of S
2x i + 2 y j xi+ y j
= 2 2
= ,
√ (4 x + 4 y ) 4
since x 2 + y 2 = 16, on the surface S.
dx dz
We have ∫ ∫ F • n dS = ∫ ∫R F•n ,where R is the projection of S on the x-z
S | n • j|
plane. It should be noted that in this case we cannot take the projection of S on the
x-y plane as the surface S is perpendicular to the x-y plane.
 x i + y j 1
Now F • n = ( z i + x j − 3 y2 z k) •   = ( xz + x y ),
 4  4
 x i + y j y
n•j=   •j= ⋅
 4  4
Therefore the required surface integral is
xz + x y dx dz
= ∫∫
R 4 y /4
5 4  xz 
=∫ ∫  + x  dx dz , since y = √ (16 − x 2 ) on S
z =0 x =0 2
 √ (16 − x ) 
5
= ∫0 (4z + 8) dz = 90.

4 Volume Integrals
(Avadh 2014)
Suppose V is a volume bounded by a
surface S. Suppose f ( x, y, z ) is a single
valued function of position defined over V.
Subdivide the volume V into n elements of
volumes δV1 , δV2 , …, δVn . In each part δVk
we choose an arbitrary point Pk whose
co-ordinates are ( x k , y k , z k ). We define
f ( Pk ) = f ( x k , y k , z k ).
Form the sum
n
∑ f ( Pk ) δVk .
k =1
V-117

Now take the limit of this sum as n → ∞ in such a way that the largest of the
volumes δVk approaches zero. This limit, if it exists, is called the volume integral of
f ( x, y, z ) over V and is denoted by ∫ ∫ ∫ f ( x, y, z ) dV .
V

It can be shown that if the surface is piecewise smooth and the function f ( x, y, z )
is continuous over V, then the above limit exists i. e., is independent of the choice of
sub-divisions and points Pk .
If we subdivide the volume V into small cuboids by drawing lines parallel to the
three co-ordinates axes, then dV = dx dy dz and the above volume integral
becomes ∫ ∫ ∫ f ( x, y, z) dx dy dz.
V

If F ( x, y, z ) is a vector function, then ∫ ∫ ∫V F dV


is also an example of a volume integral.

Example 11: If F = (2 x 2 − 3z ) i − 2 x y j − 4 x k , then evaluate ∫ ∫ ∫V ∇ • F dV

where V is the closed region bounded by the planes x = 0, y = 0, z = 0 and


2 x + 2 y + z = 4. Also Evaluate ∫ ∫ ∫ ∇ × F dV .
V

Solution: We have F = (2 x 2 − 3z ) i − 2 x y j − 4 x k .
 ∂ ∂ ∂ 2
∴ ∇ • F = i + j +k  • [(2 x − 3z ) i − 2 x y j − 4 x k ]
 ∂ x ∂ y ∂ z 
∂ ∂ ∂
= (2 x 2 − 3z ) + (− 2 x y ) + (− 4 x ) = 4 x − 2 x = 2 x.
∂x ∂y ∂z
∴ ∫ ∫ ∫V ∇ • F dV = ∫ ∫ ∫V 2 x dx dy dz [∵ dV = dx dy dz ]
2 2−x 4 −2 x −2 y
= 2∫ ∫ y =0 ∫z =0 x dx dy dz.
x =0

[Note that we have taken a thin column parallel to z-axis as the elementary volume.
It cuts the boundary at z = 0 and z = 4 − 2 x − 2 y. Also the projection of the plane
2 x + 2 y + z = 4 on the xy-plane is bounded by the axes y = 0, x = 0 and the line
x + y = 2.Hence the limits for y are from 0 to 2 − x and those for x are from 0 to 2]

[z ]
2 2− x 4 −2 x −2 y
∴ ∫ ∫ ∫V ∇ • F dV = 2∫
x =0 ∫ y =0 x
z =0
dx dy
2 2− x
= 2∫ ∫ y= 0 x (4 − 2 x − 2 y ) dx dy
x =0

[4 x y − 2 x ]
2 2− x
= 2∫ 2
y − x y2 dx
x =0 y =0
2
= 2∫ [4 x (2 − x ) − 2 x 2 (2 − x ) − x (2 − x )2 ] dx
0
2
= 2∫ [ x 3 − 4 x 2 + 4 x ] dx, on simplifying
0
2
= 2  x 4 − x 3 + 2 x 2  = 2 4 − + 8 = ⋅
1 4 32 8
4 3  0  3  3
V-118

Second part: We have


 i j k 
 ∂ ∂ ∂ 
∇×F= 
 ∂x ∂y ∂z 
 2 
2 x − 3z − 2x y − 4x
 
∂ ∂  ∂ ∂ 
=  (− 4 x ) − (− 2 x y ) i − (− 4 x ) − (2 x 2 − 3z ) j
∂ y ∂ z  ∂x ∂ z 
 
∂ ∂ 
+  (− 2 x y ) − (2 x 2 − 3z ) k
 ∂x ∂y 
= 0 i − (− 4 + 3) j + (− 2 y ) k = j − 2 y k .
∴ ∫ ∫ ∫V ∇ × F dV = ∫ ∫ ∫V ( j − 2 y k ) dx dy dz
2 2− x 4 −2 x −2 y
= ∫ x =0 ∫ y =0 ∫z =0 ( j − 2 y k ) dx dy dz
2 2−x
= ∫ x =0 ∫ y =0 ( j − 2 y k ) (4 − 2 x − 2 y ) dx dy
2−x
2  j (4 y − 2 x y − y 2 ) − 2 k (2 y 2 − x y 2 − 2 y 3 )
= ∫ x =0  3  y = 0
dx

∫ x = 0 [j (2 − x ) (4 − 2 x − 2 + x )
2
=

− 2 k (2 − x )2 2 − x − (2 − x )  dx
2
 3 
2 (2 − x )2 j − 2 (2 − x )3 k  dx
= ∫0  3 
2 ( x − 2)2 j + 2 ( x − 2)3 k  dx
= ∫0  3 
2 2
( x − 2)3  2 ( x − 2)4  8 8 8
=  j+   k = j − k = ( j − k ).
 3  0 3 4  0 3 3 3

Comprehensive Exercise 2

1. (i) Evaluate ∫ ∫ F • n dS, where F = 18z i − 12 j + 3 y k and S is the


S

surface of the plane 2 x + 3 y + 6z = 12 in the first octant.


(Bundelkhand 2005)
(ii) Evaluate ∫ ∫ ∫ φ dV , where φ = 45 x 2 y and V is the closed region
V

bounded by the planes 4 x + 2 y + z = 8, x = 0, y = 0, z = 0.


V-119

2. Evaluate ∫ ∫ F • n dS, where F = ( x + y 2 ) i − 2 x j + 2 yz k and S is the


S

surface of the plane 2 x + y + 2z = 6 in the first octant.


3. Evaluate ∫ ∫ A • n dS, where A = x y i − x 2 j + ( x + z ) k , S is the portion
S
of the plane 2 x + 2 y + z = 6 included in the first octant and n is a unit
normal to S.
4. If F = 2 yi − 3 j + x 2 k and S is the surface of the parabolic cylinder y 2 = 8 x
in the first octant bounded by the planes y = 4 and z = 6, then evaluate
∫ ∫ F • n dS.
S

5. Evaluate ∫ ∫ F • n dS, where F = yi + 2 xj − z k and S is the surface of the


S
plane 2 x + y = 6 in the first octant cut off by the plane z = 4.
6. Evaluate ∫ ∫ ∫ F dV where F = x i + y j + z k and V is the region bounded
V

by the surfaces x = 0, x = 2, y = 0, y = 6, z = 4 and z = x 2 .

A nswers 2
1. (i) 24 (ii) 128
27
2. 81 3._ 4._132
4
384
5. 108 6. 24 i + 96 j + k
5

O bjective T ype Q uestions

Multiple Choice Questions


Indicate the correct answer for each question by writing the corresponding letter
from (a), (b), (c) and (d).
1. If F = x 2 i + y 3 j and curve C is the arc of the parabola y = x 2 in the
) then ∫
x y-plane from (0, 0) to (1, 1, F • d r is
C

(a) 7 / 12 (b) 5 / 12 (c) 11 / 12 (d) None of these


2. The work done in moving a particle in a force field
F = 3 x 2 i + (2 x z − y) j + 3k
along the line joining (0, 0, 0) to (2 , 1, 3) is
(a) 12 (b) 16 (c) 0 (d) 20
V-120

3. For a closed surface S, the value of ∫ ∫S r • n dS is

(a) V (b) 2 V (c) 3V (d) 0


where V is the volume enclosed by S. (Kumaun 2013)

Fill in the Blank(s)


Fill in the blanks “……”, so that the following statements are complete and correct.
1. Any integral which is to be evaluated along a curve is called a …… .
2. Any integral which is to be evaluated over a surface is called a …… .
dr
3. If r denotes the position vector of a point on a curve C , then is a unit
ds
vector along the …… to C at the point r in the direction of s increasing.
4. ∫ ∫S F • n dS is called the …… of F over S.

5. If t is the unit tangent vector and C is the unit circle in x y-plane, with centre
at the origin, then ∫C t • dr = …… .

6. The value of ∫ ( x dy − y dx) around the circle x 2 + y 2 = 1 is …… .

7. If F = (3 x 2 + 6 y) i − 14 y z j + 20 x z 2
k and C is a straight line joining
) then ∫
(0, 0, 0) to (1, 1, 1, F • d r = …… .
C

True or False
Write ‘T’ for true and ‘F’ for false statement.
1. If C is a simple closed curve, then ∫ F • d r is called the circulation of F
C
about C .
2. If F = ax i + by j + cz k , a, b, c are constants, then
2
∫ ∫ S F • n dS = 3 π (a + b + c), where S is the surface of a unit sphere.

A nswers
Multiple Choice Questions
1. (a) 2. (b) 3. (c)
Fill in the Blank(s)
1. line integral 2. surface integral
3. tangent 4. flux
5. 2π 6. 2π 7. 13 / 3

True or False
1. T 2. F

¨
V-121

6
G reen's, G auss's and
S toke's T heorems

1 Green’s Theorem in the Plane


et R be a closed bounded region in the x-y plane whose boundary C consists of finitely
L many smooth curves. Let M and N be continuous functions of x and y having continuous
∂M ∂N
partial derivatives and in R. Then
∂y ∂x
 ∂N ∂M 
∫ ∫R 
 ∂x
−  dx dy =
∂y  ∫C ( M dx + N dy ),

the line integral being taken along the entire boundary C of R such that R is on the left as one
advances in the direction of integration.
(Meerut 2004, 05, 07, 10; Avadh 10; Purvanchal 08;
Rohilkhand 09B, 12; Kashi 13)
Proof: We shall first prove the theorem for a special region R bounded by a closed
curve C and having the property that any straight line parallel to any one of the
coordinate axes and intersecting R has only one segment (or a single point) in
common with R. This means that R can be represented in both of the forms
a ≤ x ≤ b, f ( x) ≤ y ≤ g ( x)
and c ≤ y ≤ d, p( y) ≤ x ≤ q ( y).
V-122

In the adjoining figure, the equations of the curves AEB and BFA are y = f ( x) and
y = g ( x) respectively. Similarly the
equations of the curves FAE and EBF are
x = p ( y) and x = q ( y) respectively.
We have
∂M
∫ ∫R ∂y
dx dy

b  g ( x) ∂M 
= ∫ x = a ∫ y = f ( x) ∂y
dy  dx

y = g ( x)
b  
= ∫x=a  M ( x, y )  dx
  y = f ( x)

∫ x = a [M [ x, g ( x )] − M [ x, f ( x)]] dx
b
=
b a
=− ∫a M [ x, f ( x)] dx − ∫b M [ x, g ( x)] dx

=− ∫C M ( x, y) dx, since y = f ( x) represents the curve AEB

and y = g ( x) represents the curve BFA.


If portions of C are segments parallel to y-axis
such as GH and PQ in the adjoining figure,
then the above result is not affected. The line
integral ∫ M dx over GH is zero because on

GH, we have x = constant implies


dx = 0. Similarly the line integral over PQ is
zero. The equations of QG and HP are y = f ( x)
and y = g ( x) respectively. Hence we have

∂M
−∫ ∫ dx dy = ∫C M ( x, y ) dx. ... (1)
R ∂y
∂N d  q ( y) ∂N 
Similarly, ∫ ∫R ∂x
dx dy = ∫ y =c ∫ x = p ( y) ∂x
dx  dy

x = q ( y)
d  
= ∫y=c  N ( x, y)  dy
  x = p ( y)

[N [ q ( y ), y ] − N [ p ( y ), y]] dy
d
= ∫ y=c
d c
= ∫ c N [ q ( y ), y ] dy + ∫ d N [ p ( y ), y ] dy

= ∫ N ( x, y ) dy. ... (2)


C
V-123

 ∂N ∂M 
From (1) and (2), we get on adding ∫ ∫R 
 ∂x
−  dx dy =
∂y  ∫C ( M dx + N dy) .

The proof of the theorem can now be extended to a region R which can be
subdivided into finitely many special
regions of the above type by drawing
lines (TS in the adjoining figure). In this
case we apply the theorem to each
subregion (R1 and R2 in the figure) and
then add the results. The sum of the left
hand members will be equal to the
integral over R. The sum of the right
hand members will be equal to the
integral over C plus the line integrals over
the curves introduced for subdividing R.
Each of the latter integrals comes twice,
taken once in each direction (as ST and TS in the figure). Therefore these two
integrals cancel each other and thus the sum of the right hand members will be
equal to the line integral over C.
Note: Extension of Green’s theorem in plane to multiply
connected regions.
Green’s theorem in the plane is also
valid for a multiply-connected
region R such as shown in the
adjoining figure. Here the boundary
C of R consists of two parts; the
exterior boundary C1 is traversed in
the anticlockwise sense so that R is
on the left, while the interior
boundary C2 is traversed in the
clockwise sense so that R is on the
left.
In order to establish the theorem, we construct a line such as AD (called a cross cut)
connecting the exterior and interior boundaries. The region bounded by
ADEFGDAPQLHA is simply-connected and so Green’s theorem is valid for it.
Therefore
 ∂N ∂M 
∫C M dx + N dy = ∫ ∫ 
R  ∂x
−  dx dy.
∂y 
ADEFGDAPQLHA

The integral on the left hand side leaving out the integrand is equal to
∫AD + ∫ C2 + ∫ DA + ∫ C1 = ∫ C2 +∫
C1
, since ∫
AD
=−∫
DA

= ∫C ( M dx + N dy ).

Hence the theorem.


V-124

2 Green’s Theorem in the Plane in Vector Notation


We have r = x i + y j so that dr = dx i + dy j.
Let F = Mi + Nj .
Then M dx + N dy = ( M i + N j) • (dx i + dy j) = F • dr.
Also curlF = ∇ × F
i j k
∂ ∂ ∂ ∂N ∂M  ∂N ∂M 
= = − i+ j+  −  k.
 ∂x ∂y ∂z ∂z ∂z  ∂x ∂y 
 
M N 0
∂N ∂M
∴ (∇ × F) • k = − ⋅
∂x ∂y
Hence Green’s theorem in plane can be written as
∫ ∫R (∇ × F) • k dR = ∫C F • dr

where dR = dx dy and k is unit vector perpendicular to the x y-plane.


If s denotes the arc length of C and t is the unit tangent vector to C, then
dr
dr = ds = t ds. Therefore the above result can also be written as
ds
∫ ∫R (∇ × F) • k dR = ∫C F • t ds.

Example 1: Verify Green’s theorem in the plane for ∫C ( xy + y 2 ) dx + x 2 dy , where C

is the closed curve of the region bounded by y = x and y = x 2 . (Rohilkhand 2011)


Solution: By Green’s theorem in plane, we have
 ∂N ∂M 
∫ ∫ R  ∂x − ∂y  dx dy = ∫ C ( M dx + N dy ).
Here M = x y + y 2 , N = x 2 .
The curves y = x and y = x 2 intersect at (0, 0) and
(1, 1). The positive direction in traversing C is as
shown in the figure.
 ∂N ∂M 
We have ∫ ∫R  ∂x − ∂y  dx dy
∂ ∂ 
= ∫ ∫R  (x )− ( xy + y 2 ) dx dy
2

 ∂ x ∂ y 
= ∫ ∫R (2 x − x − 2 y ) dx dy = ∫ ∫R ( x − 2 y ) dx dy
V-125

x
1 x 1  2
= ∫ x = 0 ∫ y = x2 ( x − 2 y) dy dx = ∫x =0  xy − y  dx
  y= x 2

1 1
= ∫0 [ x 2 − x 2 − x 3 + x 4 ] dx = ∫0 ( x 4 − x 3 ) dx
1
 x5 x4  1 1 1
= −  = − =− ⋅
 5 4 0 5 4 20
Now let us evaluate the line integral along C. Along y = x 2 , dy = 2 x dx. Therefore
along y = x 2 , the line integral equals
1 1 19
∫0 [{( x )( x 2 ) + x 4 } dx + x 2 (2 x) dx ] = ∫0 (3 x 3 + x 4 ) dx =
20

Along y = x, dy = dx. Therefore along y = x, the line integral equals


0 0
∫1 [{( x) ( x) + x 2 } dx + x 2 dx ] = ∫1 3 x 4 dx = − 1.

19 1
Therefore the required line integral = −1= − ⋅ Hence the theorem is
20 20
verified.

Example 2: Evaluate by Green’s theorem ∫ ( x 2 − cosh y ) dx + ( y + sin x) dy, where


C

C is the rectangle with vertices (0, 0), (π, 0), (π, 1), (0, 1).
(Meerut 2002, 05B, 06, 13B; Rohilkhand 14)
Solution: By Green’s theorem in plane, we have
 ∂N ∂M 
∫ ∫R  ∂x − ∂y  dx dy = ∫ C ( M dx + N dy ).
Here M = x 2 − cosh y, N = y + sin x.
∂N ∂M
∴ = cos x, = − sinh y.
∂x ∂y
Hence the given line integral is equal to
∫ ∫R (cos x + sinh y ) dx dy
π 1
= ∫ x =0 ∫ y =0 (cos x + sinh y ) dy dx
π 1
 y cos x + cosh y 
= ∫ x =0   y =0
dx

π
π  
= ∫ x =0 [cos x + cosh 1 − 1] dx =  sin x + x cosh 1 − x  = (cosh 1 − 1).
 0
Example 3: Show that the area bounded by a simple closed curve C is given by
1
2 ∫C
( x dy − y dx). Hence find the area of the ellipse x = a cos θ, y = b sin θ.
V-126

Solution: By Green’s theorem in plane, if R is a plane region bounded by a simple


closed curve C, then
 ∂N ∂M 
∫ ∫R 
 ∂x
−  dx dy =
∂y  ∫C M dx + N dy.

Putting M = − y, N = x, we get
∂ ∂ 
∫C ( x dy − y dx) = ∫ ∫R  ( x) −
 ∂x ∂y
(− y) dx dy

=2 ∫ ∫R dx dy

= 2 A, where A is the area bounded by C.


1
Hence A=
2 ∫C ( x dy − y dx).

The area of the ellipse


1 1 2π  dy dx 
= ∫ ( x dy − y dx) = ∫  a cos θ − b sin θ  dθ
2 C 2 θ = 0  dθ dθ
1 2 π 1 2 π
= ∫ (ab cos 2 θ + ab sin2 θ) dθ = ab ∫ dθ = πab.
2 0 2 0

Comprehensive Exercise 1

1. Verify Green’s theorem in the plane for


∫C [(2 xy − x 2 ) dx + ( x 2 + y 2 ) dy ],
where C is the boundary of the region enclosed by y = x 2 and y 2 = x
described in the positive sense.
2. Verify Green’s theorem in the plane for
∫ [(3 x − 8 y ) dx + (4 y − 6 xy) dy],
2 2
C
where C is the boundary of the region defined by y = √ x, y = x 2 .
3. Apply Green’s theorem in the plane to evaluate
∫C {( y − sin x ) dx + cos x dy},
where C is the triangle enclosed by the lines y = 0, x = 2π, πy = 2 x.
(Avadh 2010)
−x −x
4. Evaluate by Green’s theorem in plane ∫C (e sin y dx + e cos y dy ),

where C is the rectangle with vertices (0, 0), (π , 0),  π , π , 0, π .
1 1
 2   2 
5. Evaluate by Green’s theorem ∫C (cos x sin y − xy) dx + sin x cos y dy,
where C is the circle x 2 + y 2 = 1. (Kumaun 2012)
6. If F = ( x 2 − y 2 ) i + 2 x y j and r = xi + yj , find the value of ∫ F • dr
around the rectangular boundary x = 0, x = a, y = 0, y = b.
V-127

7. Verify Green’s theorem in the plane for

∫C ( x 2 − xy 3 ) dx + ( y 2 − 2 xy) dy,
where C is the square with vertices (0, 0), (2, 0), (2, 2), (0, 2).
(Meerut 2001)
8. Apply Green’s theorem in the plane to evaluate
∫C [(2 x 2 − y 2 ) dx + ( x 2 + y 2 ) dy], where C is the boundary of the
surface enclosed by the x-axis and the semi-circle y = (1 − x 2 )1 /2 .
9. If C is the simple closed curve in the xy-plane not enclosing the origin,
−i y+ jx
show that ∫ F • dr = 0, where F = .
C x2 + y2

A nswers 1
π 2 −π
3. − − 4. 2 (e − 1) 5. 0 6. 2ab 2
4 π

3 The Divergence Theorem of Gauss


Suppose V is the volume bounded by a closed piecewise smooth surface S. Suppose F ( x, y, z ) is
a vector function of position which is continuous and has continuous first partial derivatives in
V. Then ∫ ∫ ∫ ∇ • F dV = ∫ ∫ F • n dS,
V S

where n is the outward drawn unit normal vector to S.


(Meerut 2000, 01, 06, 10B, 12, 12B;
Bundelkhand 09, 11; Avadh 14; Kashi 14)
Since F • n is the normal component of vector F, therefore divergence theorem may
also be stated as follows :
The surface integral of the normal component of a vector F taken over a closed surface is equal to
the integral of the divergence of F taken over the volume enclosed by the surface.
Cartesian equivalent of Divergence Theorem:
Let F = F1 i + F2 j + F3 k .
∂F ∂F ∂F
Then ∇ • F = div F = 1 + 2 + 3 .
∂x ∂y ∂z
If α, β, γ are the angles which outward drawn unit normal n makes with positive
directions of x, y, z -axes, then cos α, cos β, cos γ are direction cosines of n and we
have n = cos α i + cos β j + cos γ k .
∴ F • n = ( F1 i + F2 j + F3 k ) • (cos α i + cos β j + cos γ k )
= F1 cos α + F2 cos β + F3 cos γ .
V-128

Therefore the divergence theorem can be written as


 ∂F1 ∂F ∂F 
∫ ∫ ∫V 
 ∂x
+ 2 + 3  dx dy dz
∂y ∂z 
= ∫ ∫S ( F1 cos α + F2 cos β + F3 cos γ ) dS

= ∫∫ F1 dy dz + F2 dz dx + F3 dx dy).

The significance of divergence theorem lies in the fact that a surface integral
may be expressed as a volume integral and vice versa.
Proof of the divergence theorem:
We shall first prove the theorem for a special
region V which is bounded by a piecewise
smooth closed surface S and has the property
that any straight line parallel to any one of the
coordinate axes and intersecting V has only
one segment (or a single point) in common
with V. If R is the orthogonal projection of S
on the xy-plane, then V can be represented in
the form f ( x, y) ≤ z ≤ g ( x, y) where ( x, y)
varies in R.
Obviously z = g ( x, y) represents the upper portion S1 of S, z = f ( x, y) represents
the lower portion S2 of S and there may be a remaining vertical portion S3 of S.
We have
∂F3 ∂F3
∫ ∫ ∫V ∂z
dV = ∫ ∫ ∫V ∂z
dx dy dz

 g ( x, y) ∂F3 
= ∫ ∫ R ∫ z = f ( x, y) ∂z
dz  dx dy

∫ ∫ R [F3 ( x, y, z)] z = f ( x, y) dx dy
g ( x, y)
=

= ∫ ∫ [ F3 [ x, y, g ( x, y)] − F3 [ x, y, f ( x, y)]] dx dy
R

= ∫ ∫R F3 [ x, y, g ( x, y)] dx dy

− ∫ ∫R F3 [ x, y, f ( x, y)] dx dy ...(1)

Now for the vertical portion S3 of S, the normal n 3 to S3 makes a right angle γ with
k. Therefore ∫ ∫ S3 F3 k • n 3 dS3 = 0, since k • n 3 = 0.

For the upper portion S1 of S, the normal n1 to S1 makes an acute angle γ 1 with k.
Therefore k • n1 dS1 = cos γ 1 dS1 = dx dy.
Hence ∫ ∫ S1 F3 k • n1 dS1 = ∫ ∫R F3 [ x, y, g ( x, y)] dx dy.
V-129

For the lower portion S2 of S,the normal n 2 to S2 makes an obtuse angle γ 2 with k.
Therefore k • n 2 dS2 = cos γ 2 dS2 = − dx dy.
Hence ∫ ∫ S1 F3 k • n 2 dS2 = − ∫ ∫R F3 [ x y, f ( x, y)] dx dy.

∴ ∫ ∫S F3 k • n 3 dS3 + ∫ ∫ S1 F3 k • n1 dS1 + ∫ ∫ S2 F3 k • n 2 dS2

=0 + ∫ ∫R F3 [ x, y, g ( x, y)] dx dy

− ∫ ∫R F3 [ x, y, f ( x, y)] dx dy

or with the help of (1), we get


∂F3
∫ ∫S F3 k • n dS = ∫ ∫ ∫V ∂z
dV . ...(2)

Similarly, by projecting S on the other co-ordinate planes, we get


∂F2
∫ ∫S F2 j • n dS = ∫ ∫ ∫V ∂y
dV ...(3)

∂F1
and ∫ ∫S F1 i • n dS = ∫ ∫ ∫V ∂x
dV ...(4)

Adding (2), (3) and (4), we get


 ∂F1 ∂F ∂F 
∫ ∫S ( F1 i + F2 j + F3 k ) • n dS = ∫ ∫ ∫V 
 ∂x
+ 2 + 3  dV
∂y ∂z 

or ∫ ∫ ∫V ∇ • F dV = ∫ ∫S F • n dS.

The proof of the theorem can now be extended to a region V which can be
subdivided into finitely many special regions of the above type by drawing
auxiliary surfaces. In this case we apply the theorem to each sub-region and then
add the results. The sum of the volume integrals over parts of V will be equal to the
volume integral over V. The surface integrals over auxiliary surfaces cancel in pairs,
while the sum of the remaining surface integrals is equal to the surface integral over
the whole boundary S of V.

Note: The divergence theorem is applicable for a


region V if it is bounded by two closed surfaces
S1 and S2 one of which lies within the other.
Here outward drawn normals will have the
directions as shown in the figure.
V-130

4 Some Deductions from Divergence Theorem


1. Green’s theorem: Let φ and ψ be scalar point functions which together with their
derivatives in any direction are uniform and continuous within the region V bounded by a
closed surface S, then
∫ ∫ ∫V (φ∇ 2 ψ − ψ∇ 2 φ) dV = ∫ ∫S (φ∇ ψ − ψ∇ φ) • n dS.

Proof: By divergence theorem, we have ∫ ∫ ∫V ∇ • F dV = ∫ ∫S F • n dS.

Putting F = φ∇ψ, we get ∇ • F = ∇ • (φ∇ψ)


= φ (∇ • ∇ψ) + (∇φ) • (∇ψ) = φ∇ 2 ψ + (∇φ) • (∇ψ).
Also F • n = (φ∇ψ) • n.
∴ divergence theorem gives
∫ ∫ ∫V [φ∇ 2 ψ + (∇φ) • (∇ψ)] dV = ∫ ∫S (φ∇ψ) • n dS ...(1)

This is called Green’s first identity or theorem.


Interchanging φ and ψ in (1), we get
∫ ∫ ∫V [ψ∇ 2 φ + (∇ψ) • (∇φ)] dV = ∫ ∫S [ψ∇φ] • n dS ...(2)

Subtracting (2) from (1), we get


∫ ∫ ∫V (φ∇ 2 ψ − ψ∇ 2 φ) dV = ∫ ∫S (φ∇ ψ − ψ∇φ) • n dS ...(3)

This is called Green’s second identity or Green’s theorem in symmetrical form.


∂ψ ∂φ
Since ∇ψ = n and ∇φ = n, therefore
∂n ∂n
 ∂ψ ∂φ 
(φ∇ψ − ψ∇φ) • n =  φ n−ψ n • n
 ∂n ∂n 
∂ψ ∂φ
=φ −ψ .
∂n ∂n
Hence (3) can also be written as
 ∂ψ ∂φ
∫ ∫ ∫V (φ∇ 2 ψ − ψ∇ 2 φ) dV = ∫ ∫S φ
 ∂n
− ψ  dS.
∂n
Note: Harmonic function: If a scalar point function φ satisfies Laplace’s equation
∇ 2 φ = 0, then φ is called harmonic function. If φ and ψ are both harmonic functions,
then ∇ 2 φ = 0, ∇ 2 ψ = 0.
 ∂ψ ∂φ
Hence from Green’s second identity, we get ∫ ∫S φ
 ∂n
− ψ  dS = 0.
∂n
2. Prove that ∫ ∫ ∫ ∇φ dV = ∫ ∫S φn dS.
V

Proof: By divergence theorem, we have ∫ ∫ ∫ ∇ • F dV = ∫ ∫S F • n dS.


V
V-131

Taking F = φ C where C is an arbitrary constant non-zero vector, we get


∫ ∫ ∫V ∇ • (φC) dV = ∫ ∫S (φC) • n dS. ...(1)

Now ∇ • (φ C) = (∇φ) • C + φ(∇ • C) = (∇φ) • C, since ∇ • C = 0.


Also (φ C) • n = C • (φ n).
∴ (1) becomes
∫ ∫ ∫V C • (∇φ) dV = ∫ ∫S C • (φn) dS

or C•∫∫∫ ∇φ dV = C • ∫ (φn) dS
V S

or C • ∫ ∫ ∫ ∇φ dV − ∫ ∫S φn dS = 0.
 V 
Since C is an arbitrary vector, therefore we must have
∫ ∫ ∫V ∇φ dV = ∫ ∫S φn dS.

3. Prove that ∫ ∫ ∫ ∇ × B dV = ∫ ∫S n × B dS.


V

Proof: In divergence theorem taking F = B × C, where C is an arbitrary constant


vector, we get
∫ ∫ ∫V ∇ • (B × C) dV = ∫ ∫S (B × C) • n dS. ...(1)

Now ∇ • (B × C) = C • curl B − B • curl C = C • curl B, since curl C = 0.


Also (B × C) • n = [B, C, n] = [C, n, B] = C • (n × B).
∴ (1) becomes
∫ ∫ ∫V (C • curl B) dV = ∫ ∫ S C • (n × B) dS
or C • ∫ ∫ ∫ (∇ × B) dV = C • ∫ ∫ (n × B) dS
V S

or C • ∫ ∫ ∫ (∇ × B) dV − ∫ ∫S (n × B) dS = 0.
 V 
Since C is an arbitrary vector therefore we can take C as a non-zero vector which is
not perpendicular to the vector
∫ ∫ ∫V (∇ × B) dV − ∫ ∫S (n × B) dS.
Hence we must have
∫ ∫ ∫V (∇ × B) dV − ∫ ∫S (n × B) dS = 0

or ∫ ∫ ∫V (∇ × B) dV = ∫ ∫
S
(n × B) dS.

Example 4: (i) For any closed surface S, prove that ∫ ∫ curl F • n dS = 0.


S
(Meerut 2009B; Purvanchal 14; Avadh 14)
V-132

(ii) Evaluate ∫ ∫ r • n dS, where S is a closed surface.


S
(Agra 2006; Kumaun 07; Bundelkhand 09; Purvanchal 14)
(iii) If F = ax i + by j + cz k , a, b, c are constants show that
4
∫ ∫S F • n dS = 3 π (a + b + c),
where S is the surface of a unit sphere.
(Bundelkhand 2001, 07, 08; Rohilkhand 07; Kashi 13)
Solution: (i) By divergence theorem, we have
∫ ∫S curl F • n dS = ∫ ∫ ∫V (div curl F) dV ,

where V is the volume enclosed by S


= 0, since div curl F = 0.
(ii) By the divergence theorem, we have
∫ ∫S r • n dS = ∫ ∫ ∫V ∇ • r dV = ∫ ∫ ∫V 3 dV ,
since ∇ • r = div r = 3
= 3V, where V is the volume enclosed by S.
(iii) By the divergence theorem, we have
∫ ∫S F • n dS = ∫ ∫ ∫V (∇ • F) dV ,
where V is the volume enclosed by S
= ∫ ∫ ∫V [∇ • (a x i + by j + cz k )] dV

∂ ∂ ∂ 
= ∫ ∫ ∫V  (a x) +
 ∂x ∂y
(by) +
∂z
(cz ) dV

= ∫ ∫ ∫V (a + b + c ) dV
4
= (a + b + c ) V = (a + b + c ) π,
3
4 4
since the volume V enclosed by a sphere of unit radius is equal to π(1)3 i. e., π.
3 3
Example 5(i): Show that ∫ ∫ n dS = 0 for any closed surface S.
S (Purvanchal 14)
(ii) Prove that ∫ ∫ r × n dS = 0 for any closed surface S.
S (Agra 2007)
(iii) Prove that ∫ ∫ n × (a × r) dS = 2Va, where a is a constant vector and V is the volume
S

enclosed by the closed surface S. (Avadh 2011, 12)


Solution: (i) Let C be any arbitrary constant vector.
Then C• ∫ ∫ n dS = ∫ ∫S C • n dS
S

= ∫ ∫ ∫V (∇ • C) dV , by divergence theorem
= 0, since div C = 0.
V-133

Thus C•∫ ∫ n dS = 0, where C is an arbitrary vector.


S

Therefore we must have ∫ ∫ n dS = 0.


S
(ii) Let C be any arbitrary constant vector. Then
C•∫∫ r × n dS = ∫ ∫S C • [(r × n)] dS = ∫ ∫S (C × r) • n dS
S

= ∫ ∫ ∫V [∇ • (C × r)] dV , by divergence theorem

= ∫ ∫ ∫V [r • curl C − C • curl r] dV = 0,
since curl C = 0 and curl r = 0.
Thus C•∫∫ r × n dS = 0, where C is an arbitrary vector.
S

Therefore, we must have ∫ ∫ r × n dS = 0.


S

(iii) We know that


∫ ∫ ∫V ∇ × B dV = ∫ ∫S n × B dS. [See article 4, part 3]

Putting B = a × r, we get
∫ ∫S n × (a × r) dS = ∫ ∫ ∫V ∇ × (a × r) dV

= ∫ ∫ ∫V curl (a × r) dV

= ∫ ∫ ∫V 2a dV , since curl (a × r) = 2a

= 2a ∫ ∫ ∫ dV = 2Va .
V

Example 6: Using the divergence theorem, show that the volume V of a region T bounded by
a surface S is
V = ∫∫ x dy dz = ∫ ∫ y dz dx = ∫ ∫ z dx dy
S S S
1
= ∫ ∫ ( x dy dz + y dz dx + z dx dy).
3 S (Meerut 2010)
Solution: By divergence theorem, we have
∂ 
∫ ∫ S x dy dz = ∫ ∫ ∫V  ∂x ( x) dV = ∫ ∫ ∫V dV = V
∂ 
∫ ∫S y dz dx = ∫ ∫ ∫V  ( y) dV =
 ∂y 
∫ ∫ ∫V dV = V

∂ 
∫ ∫S z dx dy = ∫ ∫ ∫V  (z ) dV =
 ∂z  ∫ ∫ ∫V dV = V .

Adding these results, we get


3V = ∫ ∫ ( x dy dz + y dz dx + z dx dy )
S
1
or V = ∫ ∫ ( x dy dz + y dz dx + z dx dy).
3 S
V-134

Example 7: Verify divergence theorem for


F = ( x 2 − yz ) i + ( y 2 − zx ) j + ( z 2 − x y ) k
taken over the rectangular parallelopiped 0 ≤ x ≤ a, 0 ≤ y ≤ b, 0 ≤ z ≤ c .
(Meerut 2006B; Avadh 09; Rohilkhand 13)
Solution: We have
div F = ∇ • F
∂ ∂ ∂
= ( x 2 − yz ) + ( y 2 − zx ) + ( z2 − x y )
∂x ∂y ∂z
= 2 x + 2 y + 2z.
∴ volume integer = ∫ ∫ ∫ ∇ • F dV = ∫ ∫ ∫V 2( x + y + z ) dV
V
c b a
= 2∫ ∫ y =0 ∫ x =0 ( x + y + z ) dx dy dz
z= 0
a
c b  x2 
=2 ∫z =0 ∫ y =0  + yx + zx dy dz
2  x =0
c b  a2 
=2 ∫z =0 ∫ y =0  + ay + az  dy dz
2 
b
c  a2 y2 
= 2∫  y+a + azy dz
z =0
 2 2 
y =0

c  a b ab
2 2
=2 ∫z =0  + + abz  dz
 2 2 
c
 a2 b ab 2 z 2
=2 z + z + ab 
 2 2 2 0
= [a2 bc + ab 2 c + abc 2 ] = abc (a + b + c ).

Surface Integral: We shall now calculate ∫ ∫ F • n dS over the six faces of the
S
rectangular parallelopiped.
Over the face DEFG, n = i, x = a.
Therefore, ∫ ∫ DEFG F • n dS
c b
= ∫z =0 ∫ y =0 [(a2 − y z ) i

+ ( y 2 − za) j + (z 2 − ay) k ] • i dy dz
c b
= ∫z =0 ∫ y =0 (a2 − yz ) dy dz
b
c  2 y2 
= ∫z =0 a y − z
 2


dz
y =0
V-135

c
c  2 zb 2   2 z2 2  c 2 b2
= ∫z =0 a b −
2 
 dz = a bz −
4
b  = a2 bc −
4

  0
Over the face ABCO, n = − i, x = 0. Therefore
∫ ∫ABCO F • n dS = ∫∫ [(0 − yz ) i + ... + ... ] • (− i) dy dz
b
c b c  y2 
= ∫z =0 ∫ y =0 y z dy dz = ∫z =0  z
 2 
dz
y =0
2 2 2
c b b c
= ∫z =0 2
z dz =
4

Over the face ABEF, n = j , y = b. Therefore


c a
∫ ∫ ABEF F • n dS = ∫z =0 ∫ x =0 [( x 2 − bz ) i + (b 2 − zx) j

+ ( z 2 − bx ) k ] • j dx dz
c a a2 c 2
= ∫z =0 ∫ x =0 (b 2 − zx) dx dz = b 2 ca −
4

Over the face OGDC, n = − j, y = 0. Therefore


c a c 2 a2
∫ ∫ OGDC F • n dS = ∫z=0 ∫ x =0 zx dx dz =
4

Over the face BCDE, n = k , z = c . Therefore


b a a2 b 2
∫ ∫ BCDE F • n dS = ∫ y =0 ∫ x =0 (c 2 − xy ) dx dy = c 2 ab −
4

Over the face AFGO, n = − k , z = 0. Therefore


b a a2 b 2
∫ ∫ AFGO F • n dS = ∫ y =0 ∫ x =0 x y dx dy =
4

Adding the six surface integrals, we get


 c 2 b2 c 2 b2   2 a2 c 2 a2 c 2 
∫ ∫S F • n dS =  a2 bc −
 4
+  +  b ca −
4   4
+
4 

 a2 b 2 a2 b 2 
+  c 2 ab − + 
 4 4 
= abc (a + b + c ).
Hence the theorem is verified.

Example 8: If F = x i − y j + (z 2 − 1)k , find the value of ∫ ∫S F • n dS where S is the

closed surface bounded by the planes z = 0, z = 1 and the cylinder x 2 + y 2 = 4.


(Garhwal 2000; Kanpur 05; Avadh 13; Kumaun 15)

Solution: By divergence theorem, we have ∫ ∫S F • n dS = ∫ ∫ ∫V div F dV .


V-136

∂ ∂ ∂ 2
Here div F = ( x) + (− y) + (z − 1) = 1 − 1 + 2z = 2 z.
∂x ∂y ∂z
1 2 √(4 − y 2 )
∴ ∫ ∫ ∫V div F dV = ∫ z = 0 ∫ y =−2 ∫ x = −√(4 − y 2 ) 2z dx dy dz

1 2 √(4 − y 2 )
= ∫ z = 0 ∫ y = −2 [2zx]
x = −√(4 − y 2 )
dy dz

1 2
= ∫z = 0 ∫ y = −2 4z √ (4 − y 2 ) dy dz
1
2  z2 
= ∫ y = −2 4 √ (4 − y 2 ) dy
 2  z =0
2 2
= 2∫ √ (4 − y 2 ) dy = 4∫ √ (4 − y 2 ) dy
y = −2 0

y y 2
=4 √ (4 − y 2 ) + 2 sin −1 
2 2 0
π
= 4 [2 sin −1 1] = 4 (2) = 4π.
2

Comprehensive Exercise 2

1. (i) Verify divergence theorem for F = (2 x − z )i + x 2 yj − xz 2 k taken over


the region bounded by x = 0, x = 2, y = 0, y = 2, z = 0, z = 2.

(ii) Verify divergence theorem for F = (2 x − z ) i + x 2 y j − xz 2 k taken


over the region bounded by x = 0, x = 1, y = 0, y = 1, z = 0, z = 1.
(Garhwal 2001; Kumaun 14)
2. (i) If F = 4 xz i − y j + yz k and S is the surface bounded by
2

x = 0, y = 0, z = 0, x = 1, y = 1, z = 1, evaluate ∫ ∫S F • n dS.

(ii) Evaluate ∫ ∫ x 2 dy dz + y 2 dz dx + 2z ( xy − x − y) dx dy
S

where S is the surface of the cube 0 ≤ x ≤ 1, 0 ≤ y ≤ 1, 0 ≤ z ≤ 1.


(Kumaun 2015)

3. (i) Evaluate ∫ ∫S [4 xz dy dz − y 2 dz dx + yz dx dy] where S is the

surface of the cube bounded by the planes x = 0, y = 0, z = 0, x = 1,


y = 1 and z = 1. (Meerut 2005, 06B, 10B, 11)
V-137

(ii) Apply Gauss’s divergence theorem to evaluate


∫ ∫S [( x 3 − yz ) dy dz − 2 x 2 y dz dx + z dx dy ]
over the surface of a cube bounded by the coordinate planes and the
planes x = y = z = a. (Rohilkhand 2011)
4. (i) State divergence theorem of Gauss.
(ii) Use Gauss divergence theorem to show that
1 5
∫ ∫S {( x 3 − yz ) i − 2 x 2 y j + 2 k } • n dS =
3
a ,

where S denotes the surface of the cube bounded by the planes


x = 0, x = a, y = 0, y = a, z = 0, z = a. (Bundelkhand 2005, 06)
5. Evaluate ∫ ∫ ( x i + y j + z k ) • n dS where S denotes the surface of the cube
S

bounded by the planes x = 0, y = 0, z = 0, x = a, y = a, z = a by the


application of Gauss divergence theorem. Verify your answer by evaluating the
integral directly. (Garhwal 2003)
6. (i) Evaluate by divergence theorem the integral
∫ ∫ xz dy dz + ( x y − z ) dz dx + (2 xy + y z) dx dy,
2 2 3 2
S
where S is the entire surface of the hemispherical region bounded by
z = √ (a2 − x 2 − y 2 ) and z = 0.
(ii) Evaluate ∫ ∫S ( y 2 z 2 i + z 2 x 2 j + z 2 y 2 k ) • n dS

where S is the part of the sphere x 2 + y 2 + z 2 = 1 above the xy-plane


and bounded by this plane. (Bundelkhand 2006)
7. (i) If F = ax i + by j + cz k , where a,b,c are constants, show that

∫ ∫ S (n • F) dS = 3 (a + b + c),
S being the surface of the sphere ( x − 1)2 + ( y − 2)2 + ( z − 3)2 = 1.
(ii) If S is any closed surface enclosing a volume V and
F = x i + 2 y j + 3 z k , prove that ∫ ∫ F • n dS = 6V .
S
(Rohilkhand 2009B)
8. Verify the divergence theorem for F = 4 xi − 2 y j + z 2 k 2

taken over the region bounded by the surfaces x 2 + y 2 = 4, z = 0, z = 3.


(Garhwal 2002; Bundelkhand 08)
9. Use Gauss divergence theorem to find ∫ ∫ F • n dS, where
S
F = 2 x 2 y i − y 2 j + 4 xz 2 k and S is the closed surface in the first octant
bounded by y 2 + z 2 = 9 and x = 2.
10. If F = y i + ( x − 2 xz ) j − x y k , evaluate ∫ ∫S (∇ × F) • n dS where S is the

surface of the sphere x 2 + y 2 + z 2 = a2 above the xy-plane.


V-138

11. Evaluate ∫ ∫S (∇ × F) • n dS,


where F = ( x 2 + y − 4) i + 3 x y j + (2 xz + z 2 ) k and S is the surface of the
paraboloid z = 4 − ( x 2 + y 2 ) above the xy-plane.
12. Compute
(i) ∫ ∫S (a2 x 2 + b 2 y 2 + c 2 z 2 )1 /2 dS, and

(ii) ∫ ∫S (a2 x 2 + b 2 y 2 + c 2 z 2 ) −1 /2 dS
over the ellipsoid ax 2 + by 2 + cz 2 = 1.

13. Evaluate ∫ ∫S ( x 2 + y 2 ) dS, where S is the surface of the cone

z 2 = 3 ( x 2 + y 2 ) bounded by z = 0 and z = 3.
14. Show that ∫ ∫S ( x 2 i + y 2 j + z 2 k ) • n dS vanishes where S denotes the

x2 y2 z2
surface of the ellipsoid + + = 1.
a2 b2 c2
(Meerut 2005, 07; Kumaun 11, 13)
15. If n is the unit outward drawn normal to any closed surface S, show that
∫ ∫ ∫V div n dV = S.

A nswers 2
3 1 3  a3 
2. (i) (ii) 3. (i) (ii) a2  + a
2 2 2  3 
2πa5 π
5. 3a 3 6. (i) (ii)
5 12
9. 180 10. 0
4 4π
11. − 4π 12.(i) π abc. (ii)
3 √ (abc )
13. 9π

5 Stoke’s Theorem
Let S be a piecewise smooth open surface bounded by a piecewise smooth simple closed curve C.
Let F ( x, y, z ) be a continuous vector function which has continuous first partial derivatives
in a region of space which contains S in its interior. Then

∫C F • dr = ∫ ∫S (∇ × F) • n dS = ∫ ∫S (curl F) • dS
V-139

where C is traversed in the positive direction. The direction of C is called positive if an observer,
walking on the boundary of S in this direction, with his head pointing in the direction of
outward drawn normal n to S, has the surface on the left.
(Meerut 2009; Bundelkhand 10)
 F • dr  ds =
Note: ∫C F • dr = ∫C 


ds  ∫C (F • t) ds, where t is unit tangent vector

to C. Therefore F • t is the component of F in the direction of the tangent vector of


C. Also (∇ × F) • n is the component of curl F in the direction of outward drawn
normal vector n of S.Therefore in words Stoke’s theorem may be stated as follows:
The line integral of the tangential component of vector F taken around a simple closed curve C is
equal to the surface integral of the normal component of the curl of F taken over any surface S
having C as its boundary.

Cartesian equivalent of Stoke’s theorem:


Let F = F1 i + F2 j + F3 k . Let outward drawn normal vector n of S make angles
α, β, γ with positive directions of x, y, z axes.
Then n = cos α i + cos β j + cos γ k .
 i j k 
 ∂ ∂ ∂ 
Also ∇ × F =
 ∂x ∂y ∂z 

 F1 F2 F3 
 ∂F ∂F   ∂F ∂F   ∂F ∂F 
=  3 − 2  i +  1 − 3  j +  2 − 1  k.
 ∂y ∂z   ∂z ∂x   ∂x ∂y 
 ∂F ∂F   ∂F ∂F 
∴ (∇ × F) • n =  3 − 2  cos α +  1 − 3  cos β
 ∂y ∂z   ∂z ∂x 
 ∂F ∂F 
+  2 − 1  cos γ.
 ∂x ∂y 
Also F • dr = ( F1 i + F2 j + F3 k ) • (dx i + dy j + dz k )
= F1 dx + F2 dy + F3 dz .
∴ Stoke’s theorem can be written as
∫C F1 dx + F2 dy + F3 dz

 ∂F3 ∂F   ∂F1 ∂F 
= ∫ ∫S 
 ∂y
− 2  cos α +
∂z 

 ∂z
− 3  cos β
∂x 

 ∂F ∂F  
+  2 − 1  cos γ  dS.
 ∂x ∂y  
V-140

Proof of Stoke’s theorem: Let S be a


surface which is such that its
projections on the xy, yz and zx planes
are regions bounded by simple closed
curves. Suppose S can be represented
simultaneously in the forms
z = f ( x, y ), y = g ( x, z ),
x = h ( z, y ) where f , g, h are
continuous functions and have
continuous first partial derivatives.
Consider the integral
∫ ∫S [∇ × ( F1 i)] • n dS.

 i j k 
 ∂ ∂ ∂  ∂F1 ∂F
We have ∇ × ( F1 i) =   = j − 1 k.
 ∂x ∂y ∂z  ∂z ∂y
 F1 0 0 
 ∂F ∂F  ∂F ∂F
∴ [∇ × ( F1 i)] • n =  1 j • n − 1 k • n = 1 cos β − 1 cos γ .
 ∂z ∂y  ∂z ∂y
 ∂F1 ∂F 
∴ ∫ ∫S [∇ × ( F1 i)] • n dS = ∫∫ S 
 ∂ z
cos β − 1 cos γ  dS.
∂ y 
We shall prove that
 ∂F1 ∂F 
∫ ∫S 
 ∂z
cos β − 1 cos γ  dS =
∂y 
∫C F1 dx.

Let R be the orthogonal projection of S on the xy-plane and let Γ be its boundary
which is oriented as shown in the figure. Using the representation z = f ( x, y ) of S,
we may write the line integral over C as a line integral over Γ. Thus
∫C F1 ( x, y, z ) dx = ∫ Γ F1 [ x, y, f ( x, y )] dx

= ∫ Γ {F1 [ x, y, f ( x, y )] dx + 0 dy}
∂F1
= −∫∫ dx dy,
∂y
R

by Green’s theorem in plane for the region R.


∂F1 [ x, y, f ( x, y )] ∂F1 ( x, y, z ) ∂F1 ( x, y, z ) ∂f
But = + ⋅
∂y ∂y ∂z ∂y
[∵ z = f ( x, y)]
 ∂F1 ∂F ∂f 
∴ ∫C F1 ( x, y, z ) dx = − ∫ ∫R 
 ∂y
+ 1  dx dy
∂z ∂y 
...(1)

Now the equation z = f ( x, y ) of the surface S can be written as


φ ( x, y, z ) ≡ z − f ( x, y ) = 0.
V-141

∂f ∂f
We have grad φ = − i− j + k.
∂x ∂y
Let | grad φ | = a.
grad φ
Since grad φ is normal to S, therefore, we get n = ± ⋅
a
But the components of both n and grad φ in positive direction of z-axis are positive.
Therefore
grad φ
n=+
a
1 ∂f 1 ∂f 1
or cos α i + cos β j + cos γ k = − i− j + k.
a ∂x a ∂y a
1 ∂f 1 ∂f 1
∴ cos α = − , cos β = − , cos γ = ⋅
a ∂x a ∂y a
dx dy
Now dS = = a dx dy.
cos γ
 ∂F1 ∂F 
∴ ∫ ∫S 
 ∂z
cos β − 1 cos γ  dS
∂y 
 ∂F1  1 ∂f  ∂F1 1
= ∫ ∫R  −  −
 ∂z  a ∂y 
 a dx dy
∂y a
 ∂F1 ∂F ∂f 
=− ∫ ∫R 
 ∂ y
+ 1  dx dy.
∂z ∂y 
...(2)

From (1) and (2), we get


 ∂F1 ∂F 
∫C F1 dx = ∫ ∫S 
 ∂z
cos β − 1 cos γ  dS
∂y 
= ∫ ∫S [∇ × ( F1 i)] • n dS ...(3)

Similarly, by projections on the other coordinate planes, we get


∫C F2 dy = ∫ ∫S [∇ × ( F2 j)] • n dS ...(4)

∫C F3 dz = ∫ ∫S [∇ × ( F3 k )] • n dS ...(5)

Adding (3), (4), (5), we get


∫C ( F1 dx + F2 dy + F3 dz ) = ∫ ∫S [∇ × ( F1 i + F2 j + F3 k )] • n dS

or ∫C F • dr = ∫ ∫S (∇ × F) • n dS.

If the surface S does not satisfy the restrictions imposed above, even then Stoke’s
theorem will be true provided S can be subdivided into surfaces S1 , S2 , ... , S k
with boundaries C1 , C2 , ... , Ck which do satisfy the restrictions. Stoke’s theorem
V-142

holds for each such surface. The sum of surface integrals over S1 , S2 , ... , S k will
give us surface integral over S while the sum of the integrals over C1 , C2 , ... , Ck
will give us line integral over C.
Note: Green’s theorem in plane is a special case of Stoke’s theorem. If R is a
region in the xy-plane bounded by a closed curve C, then in vector form Green’s
theorem in plane can be written as
∫ ∫R (∇ × F) • k dR = ∫C F • dr.

This is nothing but a special case of Stoke’s theorem because here k = n = outward
drawn unit normal to the surface of region R.

Example 9: Prove that ∫ r • dr = 0. (Meerut 2010)


C

Solution: By Stoke’s theorem ∫C r • dr = ∫ ∫S (curl r) • n dS = 0,since curl r = 0.

Example 10: By Stoke’s theorem prove that div curl F = 0.

Solution: Let V be any volume enclosed by a closed surface.


Then by divergence theorem
∫ ∫ ∫V ∇ • (curl F) dV = ∫ ∫S (curl F) • n dS.
Divide the surface S into two portions S1 and S2
by a closed curve C. Then
∫ ∫S (curl F) • n dS = ∫ ∫ S1 (curl F) • n dS1

+ ∫ ∫ S2 (curl F) • n dS2 . ... (1)

By Stoke’s theorem right hand side of (1) is = ∫C F • dr − ∫C F • dr = 0.

Negative sign has been taken in the second integral because the positive directions
about the boundaries of the two surfaces are opposite.
∴ ∫ ∫ ∫V ∇ • (curl F) dV = 0.
Now this equation is true for all volume elements V. Therefore we have
∇ • (curl F) = 0 or div curl F = 0.
Example 11: Verify Stoke’s theorem for F = y i + z j + x k where S is the upper half
surface of the sphere x 2 + y 2 + z 2 = 1 and C is its boundary.
(Agra 2000, 06; Kanpur 09; Kumaun 07, 10, 13)
Solution: The boundary C of S is a circle in the xy-plane of radius unity and centre
origin. The equations of the curve C are x 2 + y 2 = 1, z = 0. Suppose
x = cos t, y = sin t, z = 0, 0 ≤ t < 2 π are parametric equation of C. Then
V-143

∫C F • dr = ∫C ( y i + z j + x k ) • (dx i + dy j + dz k )

= ∫C ( y dx + z dy + x dz )

= ∫C y dx, since on C, z = 0 and dz = 0


2π dx 2π
= ∫0 sin t
dt
dt = ∫0 − sin2 t dt

1 2π 1  sin 2t  2 π
=−
2 ∫ 0
(1 − cos 2t ) dt = −
2 t −
2  0

= − π. ...(1)
Now let us evaluate ∫ ∫ curl F • n dS. We have
S
 i j k 
 ∂ ∂ ∂ 
curl F = ∇ × F =  = – i – j – k.
 ∂x ∂y ∂z 

 y z x 
If S1 is the plane region bounded by the circle C, then by an application of
divergence theorem, we have
∫ ∫S curl F • n dS = ∫ ∫ S1 curl F • k dS

[See example 4 after article 4]


= ∫ ∫ S1 (− i − j − k ) • k dS
= ∫ ∫S1 (− 1) dS = − ∫ ∫S1 dS = − S1 .

But S1 = area of a circle of radius 1 = π (1)2 = π.


∴ ∫ ∫S curl F • n dS = − π. ...(2)

Hence from (1) and (2), the theorem is verified.

Example 12: Verify Stoke’s theorem for F = (2 x − y) i − yz 2 j − y 2 z k , where S is the


upper half surface of the sphere x 2 + y 2 + z 2 = 1 and C is its boundary.
(Kumaun 2003; Kanpur 10, 14; Avadh 09)
Solution: The boundary C of S is a circle in the xy-plane of radius unity and centre
origin. Suppose x = cos t, y = sin t, z = 0,0 ≤ t < 2 π are parametric equations of
C. Then
∫ F • dr = ∫ [(2 x − y )i − yz j − y z k ] • (dx i + dy j + dz k )
2 2
C C
=∫C [(2 x − y) dx − yz 2 dy − y 2 z dz ]

=∫ (2 x − y) dx, since z = 0 and dz = 0


C
2π dx
= ∫0 (2 cos t − sin t)
dt
dt
V-144


=−∫ (2 cos t − sin t) sin t dt
0
2π 1
=−∫ [sin 2t −
(1 − cos 2t)] dt
0 2

 cos 2t 1 1 sin 2t 
= − − − t+
 2 2 2 2  0
1 1 1 1
= − [(− + ) − (π − 0) + (0 − 0)] = π. ...(1)
2 2 2 4
 i j k 
 ∂ ∂ ∂ 
And (∇ × F) =  
 ∂x ∂y ∂z 
 2 x − y − yz 2 − y2 z 
= (− 2 yz + 2 yz ) i − (0 − 0) j + (0 + 1) k = k .
Let S1 be the plane region bounded by the circle C. If S′ is the surface consisting of
the surfaces S and S1 , then S′ is a closed surface.
∴ by an application of Gauss divergence theorem, we have
∫ ∫ S′ curl F • n dS = 0 [See example 4(i) after article 4 ]

or ∫ ∫S curl F • n dS + ∫ ∫ S1 curl F • n dS = 0
[∵ S ′ consists of S and S1 ]
or ∫ ∫S curl F • n dS − ∫ ∫ S1 curl F • k dS = 0 [∵ on S1 , n = − k ]

or ∫ ∫S curl F • n dS = ∫ ∫ S1 curl F • k dS.

∴ ∫ ∫S curl F • n dS = ∫ ∫ S1 curl F • k dS

= ∫ ∫ S1 k • k dS = ∫ ∫ S1 dS = S1 = π . ...(2)

Note that S1 = area of a circle of radius 1 = π (1)2 = π .


Hence from (1) and (2) Stoke’s theorem is verified.

Example 13: Verify Stoke’s theorem for F = ( x 2 + y 2 ) i − 2 x yj taken round the


rectangle bounded by x = ± a, y = 0, y = b. (Bundelkhand 2007; Agra 08;
Kumaun 15)
 i j k 
 ∂ ∂ ∂ 
Solution: We have curl F =  
 ∂x ∂y ∂z 
2
x + y2 −2 xy 0 
= (− 2 y − 2 y ) k = − 4 y k .
Also n = k.
b a
∴ ∫ ∫S (curl F) • n dS = ∫ y = 0 ∫ x = −a (− 4 y k ) • k dx dy
V-145

b a
=−4 ∫ y = 0 ∫ x = −a y dx dy
b
= − 4∫
a
[ xy] x = − a dy
y= 0
b
= − 4∫ 2ay dy
y=0
b
= − 4 [ay 2 ] 0 = − 4ab 2 .

Also o∫ F • dr = ∫ C [( x + y ) i − 2 xy j ] • (dx i + dy j)
2 2
C

= o∫ [( x 2 + y 2 ) dx − 2 xy dy ]
C

=∫ [( x 2 + y 2 ) dx − 2 xy dy ] + ∫ +∫ +∫ .
DA AB BE ED

Along DA , y = 0 and dy = 0. Along AB, x = a and dx = 0.


Along BE, y = b and dy = 0. Along ED, x = − a and dx = 0.
a b
∴ ∫C F • dr = ∫ x = −a x 2 dx + ∫
y =0
− 2ay dy
−a 0
+ ∫x=a ( x 2 + b 2 ) dx + ∫ y=b 2ay dy
a a b
= ∫ −a x 2 dx − ∫
−a
( x 2 + b 2 ) dx − 4a ∫
0
y dy
b
a b  y2 
∫ −a ∫0 − 4a   = − 4ab 2 .
2 2
=− x dx − 4a y dy = − 2ab
 2 
0
Thus ∫C F • dr = ∫ ∫S (curl F) • n dS.
Hence the theorem is verified.

Example 14: Evaluate ∫C F • dr by Stoke’s theorem where

F = y 2 i + x 2 j − (x + z) k
and C is the boundary of the triangle with vertices at (0, 0, 0), (1, 0, 0), (1, 1, 0).
(Avadh 2013)
Solution: We have
Y
i j k
∂ ∂ ∂ B (1, 1)
Curl F =
∂x ∂y ∂z
y2 x2 (x + z)
= 0 i + j + 2 ( x − y ) k.
Also we note that z co-ordinate of each vertex O
A (1, 0) X
of the triangle is zero. Therefore the triangle lies
in the xy-plane. So n = k .
∴ Curl F • n = [ j + 2 ( x − y)k ] • k = 2 ( x − y).
V-146

In the figure, we have only considered the x y plane.


The equation of the line OB is y = x.
By Stoke’s theorem
∫C F • dr = ∫ ∫S (curl F ) • n dS

∫ ∫
1 x
= x =0 y =0
2 ( x − y ) dx dy
x
 y2 
=2∫ x y −
1
x =0  dx
 2 
 y =0
1  2 x 2 1 x2
= 2∫  x −  dx = 2∫ dx
0 2 0 2

1 1
= ∫ x 2 dx = ⋅
0 3

Comprehensive Exercise 3

1. (i) State Stoke’s theorem.


(ii) By Stoke’s theorem prove that curl grad φ = 0. (Kumaun 2014)
(iii) Verify Stoke’s theorem for the function F = zi + xj + y k where curve
is the unit circle in the xy-plane bounding the hemisphere
z = √ (1 − x 2 − y 2 ). (Garhwal 2003; Agra 07)
2. Verify Stoke’s theorem for the vector F = z i + x j + y k taken over the half
of the sphere x 2 + y 2 + z 2 = a2 lying above the xy-plane.
3. (i) Verify Stoke’s theorem for the function F = x 2 i + xy j integrated along
the rectangle, in the plane z = 0, whose sides are along the lines
x = 0, y = 0, x = a and y = b.
(ii) Verify Stoke’s theorem for the function F = ( x 2 − y 2 ) i + 2xy j in
the rectangular region in the xy-plane bounded by the lines
x = 0, x = a, y = 0 and y = b . (Kanpur 2008)
2
(iii) Verify Stoke’s theorem for the function F = x i + xy j, integrated
round the square, in the plane z = 0, whose sides are along the lines
x = 0, y = 0, x = a, y = a. (Agra 2002)
4. Verify Stoke’s theorem for the vector A = 3 y i − xz j + yz 2 k , where S is
the surface of the paraboloid 2z = x 2 + y 2 bounded by z = 2 and C is its
boundary.
V-147

5. (i) By converting into a line integral evaluate


∫ ∫S (∇ × A) • n dS, where A = ( x − z ) i + ( x 3 + yz ) j − 3 xy 2 k

and S is the surface of the cone z = 2 − √ ( x 2 + y 2 ) above the


xy-plane.
(ii) By converting into a line integral evaluate ∫ ∫S (∇ × F) • n dS

where F = ( x 2 + y − 4) i + 3 xy j + (2 xy + z 2 ) k and S is the surface of


the paraboloid z = 4 − ( x 2 + y 2 ) above the xy-plane.
6. (i) Evaluate by Stoke’s theorem ∫C (e x dx + 2 y dy − dz )

where C is the curve x 2 + y 2 = 4, z = 2. (Garhwal 2001; Meerut 09B)


(ii) Evaluate by Stoke’s theorem ∫C ( yz dx + xz dy + xy dz )

where C is the curve x 2 + y 2 = 1, z = y 2 . (Meerut 2006B)


7. (i) Evaluate ∫ ∫ (∇ × F) • n dS, where F = ( y − z + 2) i +
S

( yz + 4) j − xz k and S is the surface of the cube x = y = z = 0,


x = y = z = 2 above the xy-plane.
(ii) Evaluate by Stoke’s theorem ∫ (sin z dx − cos x dy + sin y dz ) where
C

C is the boundary of the rectangle 0 ≤ x ≤ π, 0 ≤ y ≤ 1, z = 3.


(Meerut 2009)
8. If f = ∇ φ and g = ∇ψ are two vector point functions, such that ∇ 2 φ = 0,
∇ 2 ψ = 0, show that

∫ ∫S (g • ∇) f • dS = ∫C (f × g) • dr + ∫ ∫
S
(f • ∇)g • dS.

9. Prove that a necessary and sufficient condition that ∫ F • dr = 0 for every


C

closed curve C lying in a simply connected region R is that ∇ × F = 0


identically.
10. Apply Stoke’s theorem to prove that

∫C ( y dx + z dy + x dz ) = − 2 √ 2 πa2
where C is the curve given by x 2 + y 2 + z 2 − 2ax − 2ay = 0, x + y = 2a
and begins at the point (2 a, 0, 0) and goes at first below the z-plane.
(Meerut 2005, 06B)
11. Use Stoke’s theorem to evaluate ∫ ∫ (∇ × F) • n dS,
S

where F = y i + ( x − 2 xz ) j − x y k and S is the surface of sphere


x 2 + y 2 + z 2 = a2 , above the xy-plane. (Kumaun 2012)
V-148

A nswers 3
5. (i) 12π (ii) − 4π 7. (i) − 4 (ii) 2
11. 0

6 Line Integrals Independent of Path


Let F ( x, y, z ) = f ( x, y, z ) i + g ( x, y, z ) j + h ( x, y, z ) k be a vector point function
defined and continuous in a region R of space. Let P and Q be two points in R and
let C be a path joining P to Q. Then

∫C F • dr = ∫ ( f dx + g dy + h dz ) ... (1)

is called the line integral of F along C. In general the value of this line integral
depends not only on the end points P and Q of the path C but also on C.
In other words, if we integrate from P to Q along different paths, we shall, in
general, get different values of the integral. The line integral (1) is said to be
independent of path in R, if for every pair of end points P and Q in R the value of the
integral is the same for all paths C in R starting from P and ending at Q.
In this case the value of this line integral will depend on the choice of P and Q and
not on the choice of the path joining P to Q.

Definition: The expression f dx + g dy + h dz is said to be an exact differential if there


exists a single valued scalar point function φ ( x, y, z ), having continuous first partial
derivatives such that dφ = f dx + g dy + h dz .
It can be easily seen that f dx + g dy + h dz is an exact differential if and only if the
vector function F = f i + g j + h k is the gradient of a single valued scalar function
φ ( x, y, z ).
Because F = grad φ
∂φ ∂φ ∂φ
if, and only if f i + g j+ hk = i+ j+ k
∂x ∂y ∂z
∂φ ∂φ ∂φ
if, and only if f = , g= , h=
∂x ∂y ∂z
∂φ ∂φ ∂φ
if, and only if f dx + g dy + h dz = dx + dy + dz
∂x ∂y ∂z
if, and only if f dx + g dy + h dz = d φ.

Thus F = grad φ if, and only if f dx + g dy + h dz is an exact differential dφ.


V-149

Theorem 1: Let f (x, y, z), g (x, y, z) and h (x, y, z) be continuous in a region R of space.
Then the line integral
∫ ( f dx + g dy + h dz )

is independent of path in R if and only if the differential form under the integral sign is exact in
R.
Or
Let F ( x, y, z ) be continuous in region R of space. Then the line integral ∫C F • dr

is independent of the path C in R joining P and Q if and only if F = grad φ where φ ( x, y, z )


is a single-valued scalar function having continuous first partial derivatives in R.
Proof: Suppose F = grad φ in R. Let P and Q be any two points in R and let C be
any path from P to Q in R.
Then ∫C F • dr = ∫C ∇φ • dr
 ∂φ ∂φ ∂φ 
= ∫C 
 ∂x
i+
∂y
j+ k  • (dx i + dy j + dz k)
∂z 
 ∂φ ∂φ ∂φ 
= ∫C 
 ∂x
dx +
∂y
dy +
∂z
dz 

= ∫C dφ
Q
= ∫P dφ = [ φ ] Q
P

= φ (Q) − φ ( P).

Thus the line integral depends only on points P and Q and not on the path joining
them. This is true, of course, only if φ ( x, y, z ) is single valued at all points P and Q.
Conversely, suppose the line integral ∫C F • dr is independent of the path C

joining any two points P and Q in R. Let P be a fixed point ( x0 , y0 , z 0 ) in R and


let Q be any point ( x, y, z ) in R.
( x, y, z)
Let φ ( x, y, z ) = ∫( x0 , y0 , z0 ) F • dr
( x, y, z)  dr 
= ∫ x0 , y0 , z0  F • ds  ds.
Differentiating both sides with respect to s, we get
dφ dr
=F• .
ds ds
dφ ∂φ dx ∂φ dy ∂φ dz
But = + +
ds ∂x ds ∂y ds ∂z ds
 ∂φ ∂φ ∂φ   dx dy dz 
= i+ j+ k  • i + j+ k 
 ∂ x ∂ y ∂ z   ds ds ds 
V-150

dr
=∇φ• .
ds
dr dr
∴ F• =∇φ•
ds ds
dr
or (∇ φ – F) • = 0.
ds
Now this result is true irrespective of the path joining P to Q i. e. this result is true
dr
irrespective of the direction of which is tangent vector to C. Therefore we must
ds
have ∇φ – F = 0 i.e., ∇ φ = F.
This completes the proof of the theorem.

Definition: A vector field F ( x, y, z ) defined and continuous in a region R of space is said to


be a conservative vector field if the line integral ∫ F • dr is independent of the path C in R
C

joining P and Q where P and Q are any two points in R.


By theorem 1, vector field F ( x, y, z ) is conservative if and only if F = ∇ φ where
φ ( x, y, z ) is a single valued scalar function having continuous first partial
derivatives in R. The function φ ( x, y, z ) is called the scalar potential of the vector
field F.

Theorem 2: Let F ( x, y, z ) be a vector function defined and continuous in a region R of


Q
space. Then the line integral ∫ F • dr is independent of the path joining any two points P
P

and Q in R if and only if ∫C F • dr = 0 for every simple closed path in R.

Proof: Let C be any simple closed path in R and


let the line integral be independent of path in
R. Take two points P and Q on C and subdivide
C into two arcs PBQ and QAP. Then
∫C F • dr = ∫ PBQAP F • dr

= ∫ PBQ F • dr + ∫
QAP
F • dr

= ∫ PBQ F • dr − ∫
PAQ
F • dr

= 0,
since the integral from P to Q along a path through B is equal to the integral from P
to Q along a path through A.
Conversely, suppose that the integral under consideration is zero on every simple
closed path in R. Let P and Q be any two points in R which join P to Q and do not
cross. Then
V-151

∫ PBQAP F • dr = ∫ PBQ F • dr + ∫ QAP F • dr

= ∫ PBQ F • dr − ∫
PAQ
F • dr.

But as given, we have


∫ PBQAP F • dr = 0.

∴ ∫ PBQ F • dr − ∫ PAQ F • dr = 0

or ∫ PBQ F • dr = ∫ PAQ F • dr.

This completes the proof of the theorem.

Theorem 3: Let F ( x, y, z ) = f i + g j + h k be a continuous vector function having


continuous first partial derivatives in a region R of space. If ∫ f dx + g dy + h dz is
independent of path in R and consequently f dx + g dy + h dz is an exact differential in R,
then curl F = 0 everywhere in R. Conversely, if R is simply connected and curl F = 0
everywhere in R, then f dx + g dy + h dz is an exact differential in R or
∫ f dx + g dy + h dz is independent of path in R.
Proof: Suppose ∫ ( f dx + g dy + h dz ) is independent of path in R. Then
f dx + g dy + h dz is an exact differential in R. Therefore
F = f i + g j + h k = grad φ.
∴ curl F = curl ( grad φ ) = 0.
Converse. Suppose R is simply connected and curl F = 0 everywhere in R. Let C be
any simple closed path in R. Since R is simply connected, therefore we can find a
surface S in R having C as its boundary. Therefore by Stoke’s theorem

∫C F • dr = ∫ ∫S (curl F ) • n dS = 0.

Thus ∫C F • dr is zero for every simple closed path C in R.

Therefore ∫ F • dr is independent of path in R.

Therefore F = ∇ φ and consequently f dx + g dy + h dz is an exact differential dφ.

Note: The assumption that R be simply connected is essential and cannot be


omitted. It is obvious from the following illustration.
y x
Illustration: Let F=− 2 2
i+ 2
j.
x + y x + y2
Here F is not defined at origin. In every region R of the xy-plane not containing the
origin, we have
V-152

 
 i j k 
 
∂ ∂ ∂ 
Curl F = 
 ∂x ∂y ∂z 
 y 
x
− 2 2
0 
 x + y
2
x + y2 
 ∂  x  ∂  y  
= 0i + 0 j +     
 x 2 + y 2  + ∂y  x 2 + y 2   k
 ∂x     
 x 2 + y 2 − 2 x 2 x 2 + y 2 − 2 y 2 
= +  k
 ( x 2 + y 2 )2 ( x 2 + y 2 )2 
=0k
= 0.
Suppose R is simply connected. For example let R be the region enclosed by a
simple closed curve C not enclosing the origin. Then
 y x 
∫C F • dr = ∫C  − x 2 + y 2 dx + x 2 + y 2 dy 
∂  x  ∂  y 
= ∫ ∫R 
 ∂x
   
 x 2 + y 2  − ∂y  − x 2 + y 2   dx dy,
    
by Green’s theorem in plane
= 0.
Suppose R is not simply connected. Let R be the region of the xy-plane contained
1 3
between concentric circles of radii and and having centre at origin. Obviously
2 2
R is not simply connected. We have z = 0, everywhere in R. Let C be a closed curve
in R. The parametric equations of C can be taken as x = cos t, y = sin t, z = 0,
0 ≤ t < 2π.
 y y 
We have ∫C F • dr = ∫C − 
 x 2 + y 2 dx + x 2 + y 2 dy
 
2π  sin t dx cos t dy 
= ∫t=0 − 2 2
+ 2 2  dt
 cos t + sin t dt cos t + sin t dt 

= ∫0 (sin2 t + cos 2 t) dt

= 2 π.
Thus we see that ∫C F • dr ≠ 0.

Definition: Irrotational vector field: A vector field F is said to be irrotational if curl


F = 0.
V-153

We see that an irrotational field F is characterized by any one of the three


conditions :
(i) F = ∇ φ,
(ii) ∇ × F = 0,
(iii) ∫C F • dr = 0 for every closed path.

Any one of these conditions implies the other two.

Example 15: Are the following forms exact ?


(i)__e dx + e dy + e z dz .________
y x

(ii) yz dx + xz dy + xy dz .
Solution:
(i) Here F=e y
i + e x j + e z k.
We have
i j k
∂ ∂ ∂
Curl F =
∂x ∂y ∂z
e y
ex ez
= 0 i + 0 j + (e x − e y ) k .
Since curl F ≠ 0, therefore the given form is not exact.

(ii) Here F = yz i + xz j + xy k .
We have
i j k
∂ ∂ ∂
Curl F =
∂x ∂y ∂z
yz xz xy
= ( x − x)i − ( y − y) j + (z − z )k
= 0.
Since curl F = 0, therefore the given form is exact.

Example 16: In each of following cases show that the given differential form is exact and
find a function φ such that the form equals dφ :
(i) cos x dx − 2 yz dy − y 2 dz .
(ii) (z 2 − 2 xy) dx − x 2 dy + 2 xz dz .

Solution: (i) Here F = cos x i − 2 yz j − y 2 k .


V-154

We have
 i j k 
 ∂ ∂ ∂ 
Curl F =  
 ∂x ∂y ∂z 
 
cos x − 2 yz − y 2

= (− 2 y + 2 y) i + 0 j + 0 k = 0.
∴ the given form is exact.
Let F = ∇ φ,
∂φ ∂φ ∂φ
or cos x i − 2 yz j − y 2 k = i+ j+ k.
∂x ∂y ∂z
Then
∂φ
= cos x whence φ = sin x + f1 ( y, z ) ... (1)
∂x
∂φ
= − 2 yz whence φ = − y 2 z + f 2 ( x, z ) ... (2)
∂y
∂φ
= − y 2 whence φ = − y 2 z + f 3 ( x, y). ... (3)
∂z
(1), (2), (3) each represents φ. These agree if we choose
f1 ( y, z ) = − y 2 z , f 2 ( x, z ) = sin x, f 3 ( x, y) = sin x.
∴ φ = sin x − y 2 z to which may be added any constant.
∴ φ = sin x − y 2 z + C.
(ii) Here F = (z 2 − 2 x y) i − x 2 j + 2 xz k . We have

i j k
∂ ∂ ∂
Curl F =
∂x ∂y ∂z
z2 − 2 xy − x 2 2 xz
= 0 i + 0 j + 0 k = 0.
∴ the given form is exact.
Let F =∇ φ
∂φ ∂φ ∂φ
or (z 2 − 2 xy) i − x 2 j + 2 xz k = i+ j+ k.
∂x ∂y ∂z
∂φ
Then = z 2 − 2 xy whence φ = z 2 x − x 2 y + f1 ( y, z ) ... (1)
∂x
∂φ
= − x2 whence φ = − x 2 y + f 2 ( x, z ) ... (2)
∂y
∂φ
= 2 xz whence φ = xz 2 + f 3 ( x, y). ... (3)
∂z
V-155

(1), (2), (3) each represents φ. These agree if we choose


f1 ( y, z ) = 0, f 2 ( x, z ) = xz 2 , f 3 ( x, y) = − x 2 y.
∴ φ = z 2 x − x 2 y to which may be added any constant.
∴ φ = z 2 x − x 2 y + C.

Example 17: Show that the vector field F given by


F = ( x 2 − yz ) i + ( y 2 − zx) j + (z 2 − xy) k
is irrotational. Find a scalar φ such that F = ∇φ.
Solution: We have
i j k
∂ ∂ ∂
Curl F =
∂x ∂y ∂z
x 2 − yz y 2 − zx z 2 − xy
= (− x + x) i − (− y + y) j + (− z + z ) k = 0.
∴ The vector field F is irrotational.
Let F = ∇φ
∂φ ∂φ ∂φ
or ( x 2 − yz ) i + ( y 2 − zx) j + (z 2
− xy) k = i+ j+ k.
∂x ∂y ∂z
∂φ x3
Then = x 2 − yz whence φ = − xyz + f1 ( y, z ) ... (1)
∂x 3
∂φ y3
= y 2 − zx whence φ = − xyz + f 2 ( x, z ) ... (2)
∂y 3
∂φ z3
= z 2 − x y whence φ = − x y z + f 3 ( x, y). ... (3)
∂z 3
(1), (2), (3) each represents φ. These agree if we choose
y3 z3 x3 + z 3 x3 + y3
f1 ( y, z ) = + , f 2 ( x, z ) = , f 3 ( x, y) = .
3 3 3 3
3 3
x + y + z3
Therefore φ= − xyz + C.
3

7 Physical Interpretation of Divergence and Curl


Physical interpretation of divergence: Suppose that there is a fluid motion whose
velocity at any point is v ( x, y, z). Then the loss of fluid per unit volume per unit time in a small
parallelopiped having centre at P (x, y, z) and edges parallel to the co-ordinate axes and having
lengths δx, δy, δz respectively, is given approximately by
div v = ∇ • v.
V-156

Let v = v1 i + v2 j + v3 k .
x-component of velocity v at P = v1 ( x, y, z ).
∴ x-component of v at centre of face AFED
which is perpendicular to x-axis and is nearer
to origin
 δx 
= v1  x − , y, z 
 2 
δx ∂v1
= v1 ( x, y, z ) − +…
2 ∂x
by Taylor’s theorem
δx ∂v1
= v1 ( x, y, z ) − approximately.
2 ∂x
Similarly x-component of v at centre of opposite face GHCB
δx ∂v1
= v1 + approximately.
2 ∂x
∴ volume of fluid entering the parallelopiped across AFED per unit time
 δx ∂v1 
=  v1 −  δy δz .
 2 ∂x 
Also volume of fluid going out the parallelopiped across GHCB per unit time
 δx ∂v1 
=  v1 +  δy δz .
 2 ∂x 
∴ loss in volume per unit time in the direction of x-axis
 δx ∂v1   δx ∂v1 
=  v1 +  δy δz −  v1 −  δy δz
 2 ∂x   2 ∂x 
∂v1
= δx δy δz .
∂x
Similarly, loss in volume per unit time in y direction
∂v
= 2 δx δy δz ,
∂y
and loss in volume per unit time in z direction
∂v
= 3 δx δy δz .
∂z
∴ total loss of the fluid per unit volume per unit time symbol ·
 ∂v1 ∂v ∂v 
 + 2 + 3  δx δy δz
 ∂x ∂y ∂z 
=
δx δy δz
∂v1 ∂v2 ∂v3
= + + = ∇ • v = div v.
∂x ∂y ∂z
V-157

Physical interpretation of curl: Let S be a circular disc of small radius r and centre
P bounded by the circle C. Let F ( x, y, z ) be a continuously differentiable vector
function in S. Then by Stoke’s theorem

∫C F • dr = ∫ ∫S (curl F ) • n dS = (curl F ) • n ∫ ∫S dS,

by mean value theorem of integral calculus where (curl F ) • n is some value


intermediate between the maximum and minimum values of (curl F ) • n over S.
∴ ∫C F • dr = (curl F ) • n S.

 F • dr
∫ C 
∴ (curl F ) • n =  .
S
Taking limit as r → 0, we get at P,
∫ C F • dr
(curl F ) • n = lim .
r→0 S
Now (curl F ) • n is normal component of curl F at P and ∫ F • dr is circulation of
C

F about C. Therefore the normal component of the curl can be interpreted


physically as the limit of the circulation per unit area.

Comprehensive Exercise 4

1. Are the following forms exact ?


(i) x dx − y dy + z dz .
(ii) y 2 z 3 dx + 2 xyz 3 dy + 3 xy 2 z 2 dz .
2. In each of following cases show that the given differential form is exact
and find a function φ such that the form equals dφ :
(i) x dx − y dy − z dz .
(ii) dx + z dy + y dz .
3. (i) Show that
( y 2 z 3 cos x − 4 x 3 z ) dx + 2z 3 y sin x dy + (3 y 2 z 2 sin x − x 4 ) dz
is an exact differential of some function φ and find this function.
(ii) Show that F = (2 x y + z 3 ) i + x 2 j + 3 xz 2 k is a conservative force
field. Find the scalar potential. Find also the work done in moving an
object in this field from (1, − 2, 1) to (3, 1, 4).
V-158

4. (i) Show that the vector field


F = (2 xy 2 + yz ) i + (2 x 2 y + xz + 2 yz 2 ) j + (2 y 2 z + xy) k
is conservative.
(ii) Show that F = xi + yj + zk is conservative and find φ such that F = ∇φ.
5. (i) Show that F = (sin y + z ) i + ( x cos y − z ) j + ( x − y) k is a
conservative vector field and find a function φ such that F = ∇φ.
(ii) Evaluate ∫ 2 xyz 2 dx + ( x 2 z 2 + z cos yz ) dy + (2 x 2 yz + y cos yz ) dz
C

where C is any path from (0, 0, 1) to (1, π / 4 , 2).


6. Show that the following vector functions F are irrotational and find the
corresponding scalar φ such that F = ∇φ .
(i) F = (sin y + z cos x) i + ( x cos y + sin z ) j + ( y cos z + sin x) k .
(ii) F = ( y sin z − sin x) i + ( x sin z + 2 yz ) j + ( xy cos z + y 2 ) k .
(iii) F = x 3 i + y 3 j + z 3 k .
7. Find a, b, c if F = (3 x − 3 y + az ) i + (bx + 2 y − 4z ) j + (2x + cy + z ) k
is irrotational.
8. Evaluate ∫C yz dx + ( xz + 1) dy + x y dz , where C is any path from

(1, 0, 0) to (2, 1, 4).


9. Show that the form under the integral sign is exact and evaluate
(2 , 0,1)
∫(0, 2 ,1) [ze x dx + 3 yz dy + (e x + y 2 ) dz ].

A nswers 4
1. (i) Exact (ii) Exact
2
x − y2 − z 2
2. (i) Exact; φ = +C
2
(ii) Exact; φ = x + yz + C
3. (i) φ = y 2 z 3 sin x − x 4 z + C
(ii) φ = x 2 y − xz 3 + C; 202
1
4. (ii) φ = ( x 2 + y 2 + z 2 ) + C.
2
5. (i) φ = x sin y + x z − yz + C.
(ii) π +1
V-159

6. (i) φ = x sin y + z sin x + y sin z + C


(ii) φ = xy sin z + cos x + y 2 z + C
1
(iii) φ = ( x 4 + y 4 + z 4 ) + C
4
7. a = 2, b = − 3, c = − 4.
8. 9 9. e2 − 5

O bjective T ype Q uestions

Multiple Choice Questions


Indicate the correct answer for each question by writing the corresponding letter from
(a), (b), (c) and (d).

1. If C is the curve x 2 + y 2 = 1, z = y 2 , then by Stoke’s theorem


∫ ( yz dx + zx dy + xy dz) is
C

(a) 0 (b) 3
(c) 5 (d) None of these
2. If S denotes the surface of the cube bounded by the planes x = 0, x = a, y = 0,
y = a , z = 0, z = a then by the application of Gauss divergence theorem the
value of ∫ ∫ ( x i + y j + z k ) • n dS is
S

(a) a3 (b) 2a3


(c) 3a3 (d) 0

Fill in the Blank(s)


Fill in the blanks “……”, so that the following statements are complete and correct.

1. For any closed surface S, ∫ ∫ curl F • n dS = …… .


S
2. If n is the unit outward drawn normal to any closed surface S, then

∫ ∫ ∫V div n dV = …… .
(Bundelkhand 2008)
3. The value of ∫C r • d r = …… .
(Agra 2008)
4. A necessary and sufficient condition that ∫ F • d r = 0 for every closed
C
curve C lying in a simply connected region R is that ∇ × F = ……
identically.
V-160

5. By Stoke’s theorem, ∫C F • d r = …… .

6. ∫ ∫S r • n dS = …… .
(Kumaun 2009)

True or False
Write ‘T’ for true and ‘F’ for false statement.

1. ∫ ∫S n dS = 0 for any surface S.


2. Green’s theorem in plane is a special case of Stoke’s theorem.
3. Green’s theorem states that “the surface integral of the normal component of
a vector F taken over a closed surface is equal to the integral of the divergence
of F taken over the volume enclosed by the surface”.

A nswers
Multiple Choice Questions
1. (a) 2. (c)

Fill in the Blank(s)


1. 0 2. S 3. 0 4. 0
5. ∫ ∫ (∇ × F) • n dS 6. 3V
S

True or False
1. F 2. T 3. F

You might also like